CAT Previous Year Paper 2016

CAT 2016 Session 1

Question :- 1 – 4

Section I – Verbal Ability:-

Arrange the sentences A, B, C and D to form a logical sequence between sentences 1 and 6.

Q. 1 Amount of published information available varies widely by industry.

A. Unfortunately for the researcher, many industries do not meet these criteria, and there may be little published information available.

B. Generally, the problem the researcher will face in using published data for analysing an economically meaningful industry is that they are too broad or too arranged to fit the industry.

C. However, it is always possible to gain some important information about an industry from published sources and these sources should be aggressively pursued.

D. Larger the industry, the older it is, and the slower the rate of technological change, better is the available published information.

 

If a researcher starts a searching for data with this reality in mind, the uselessness of broad data will be better recognized and the tendency to give up will be avoided.

A. ACBD

B. BDAC

C. ADBC

D. BDCA

 

Q. 2 1. The main source of power in industrial undertaking is electricity.

A. Electricity from water also requires enormous river valley projects involving huge expenditure.

 

B. In contrast, electricity from atomic power stations will result in a tremendous saving in expenditure.

C. Besides, the mineral resources of the world required for generation of electricity are being rapidly depleted.

D. But the production of electricity needs huge quantities of coal.

6. The installation of atomic plants will help in meeting the shortage of these resources.

A. DACB

B. BADC

C. DABC

D. ADBC

 

Q. 3 1. Intensity of competition in an industry is neither a matter of coincidence nor bad luck.

A. The collective strength of these forces determines the ultimate profit potential in the industry where profit potential is measured in terms of long run returns on invested capital.

B. Rather, competition in an industry is rooted in its underlying economic structure and goes well beyond the behavior of current competitors.

C. Not all industries have the same potential.

D. The state of competition in an industry depends on five basic competitive forces.

6. They differ fundamentally in their ultimate profit potential as the collective strength of the forces differ.

A. BDAC

B. DBAC

C. ADBC

D. BDCA

 

Q. 4 1. The New Economic Policy comprises the various policy measures and changes introduced since July 1991.

A. There is a common thread running through all these measures.

B. The objective is simple to improve the efficiency of the system.

C. The regulator mechanism involving multitude of controls has fragmented the capacity and reduced competition even in the private sector.

 

D. The thrust of the new policy is towards creating a more competitive environment as a means to improving the productivity and efficiency of the economy.

6. This is to be achieved by removing the banners and restrictions on the entry and growth of firms.

A. DCAB

B. ABCD

C. BACD

D. ABDC

 

Questions: 5 – 7

A number of sentences are given below which, when properly sequenced, form a coherent paragraph, Each sentence is labelled with a letter. However, One of the statement is illogically placed. Choose the illogically placed out of sentences from among the five given choices and construct a coherent paragraph from the remaining four sentences.

 

Q. 5 A. It is turning off the tap.

B. And with no consensus of the exit policy, the government is damned if it supports loss making units and damned if it doesn’t.

C. The private sector did the same in the past because securing legal sanction for closure was virtually impossible.

D. After years of funding the losses of public sector companies, the government is doing the unthinkable.

E. Private sector and public sector companies act in a similar fashion when in crisis.

A. A

B. B

C. C

D. D

E. E

 

Q. 6 A. Trade protocols were signed, the dollar as the medium of exchange was ignored, trade was denominated in rupees and the exchange rate between the two countries was to be fixed outside the ambit of free markets.

B. A young India, some years after independence fashioning her foreign policy of nonalignment, found it prudent to stay close to the former Soviet Union.

 

C. Therefore it lead to the escalation of cold war between Soviet Union and the U.S.A.

D. Once upon a time there was a super power named Soviet Union that attracted nations apprehensive of the global aspirations of the other superpower, the U.S.A.

E. One way of doing this was to evolve a bilateral relations in trade that could be called upon provide a buffer against the arm-twisting by the U.S.A.

A. A

B. B

C. C

D. D

E. E

 

Q. 7 A. I had heard that sort of thing before.

B. He said that his generation was the first to believe that it had no future.

C. A young American made earthling stopped by my house the other day to talk about some book of mine he had read.

 

D. He was the son of a Boston man who had died an alcoholic vagrant.

E. Alcohol had a devastating effect on this Boston based American family

A. A

B. B

C. C

D. D

E. E

 

Question 8

Evidence has been presented that the unconscious mind, still viewed by many psychological scientists as the shadow of a ‘‘real’’ conscious mind, is identifiably more deliberate, action oriented and complex than its conscious counterpart. Furthermore researchers have proven that the mind is incredibly efficient at extracting meaning from stimuli of which one is not consciously aware. The claims above are made on conclusive experimentations in which test subjects who were allowed to sleep during a decision making process made more optimal decisions when compared to the subjects who were given the exact same amount of information but were not allowed to sleep, leading researchers to believe that rationality comes on the way of making a rational decision. Researchers explain that while the conscious mind can only follow strict rules, unconscious mind can handle and integrate a larger amount of information, explaining why it can make better decisions. Even more surprising than this unconscious mind’s ability is that the mental processes that drive such decision making are necessarily minimal and unsophisticated and do not require humongous amount of calories to make us arrive at the best decision whereas using conscious mind for complex decisions burns up a lot of energy, setting in fatigue, forcing the conscious mind to give up and leading to subconscious decisions. Overall, researchers agree that there is no need to have sleepless night pondering over a complex issue to resolve it when it can actually be solved more efficiently by snoring the night away.

 

Q. 8 The purpose of the passage is to:

A. highlight the differences among psychologists regarding the importance of the unconscious mind in making complex decisions.

B. contrast and compare the workings of the rationality with that of deliberate, action oriented and complex decision making processes.

C. prove by citing results of experiments that one decision making process is better than the other.

D. show that unconscious mind is not that undependable as previously thought.

E. ascertain that using conscious and unconscious minds together yield second-to-none judgments.

 

Question 9

French cuisine is highly regarded all over the world. Yet in Paris there are more American restaurants selling burgers and fries (which many people now class as junk food) than there are in any other European capital city. Obviously the French are very fond of junk food, and are not too proud to eat it. 

 

Q. 9 Which of the following, if true, would most weaken the author’s contention?

A. There are also a larger number of Lebanese restaurants in Paris than there are in

other European capital cities

B. French Cordon Bleu cuisine is very expensive

C. The number of French tourists eating in New York burger restaurants is very low

D. Junk food is actually has high nutritional value when eaten in moderation

E. There are an unusually large number of American tourists in Paris who eat at burger joints

 

Question 10

In research designed to investigate the possibility of animals developing friendship with other, unrelated, members of their species, a group of 29 chimpanzees were reared together for 15 years. At the end of that time the chimps were presented with two options for obtaining food: press a lever and feed themselves, or press another identical lever and feed themselves, and at the same time deliver food to the chimp next door. (The chimps were able to see each other). The researchers found that the chimps were no more likely to choose the lever that fed a neighbour. The researchers concluded that the chimps had no concept of friendship. However, one critic has suggested that the animals were in an artificial environment from which little can be concluded, and that, at the least, the test ought to have involved the animals being able to touch. 

 

Q. 10 What role do the parts in boldface play in the argument above?

A. The first is a position that the critic opposes. The second is a position that the critic supports.

 

B. The first is an observation that supports the researcher’s position. The second is an observation that opposes the researcher’s position.

 

C. The first is a finding on which the researchers base their conclusion. The second is a suggestion that might cast doubt on that finding.

D. The first is an observation that supports the critic’s conclusion. The second is the critic’s conclusion.

 

E. The first is part of the evidence that the critic disputes. The second is a suggestion that the researchers do not accept.

 

 

Questions: 11 – 16

Reading Comprehension:

A conservation problem equally as important as that of soil erosion is the loss of soil fertility. Most agriculture was originally supported by the natural fertility of the soil; and, in areas in which soils were deep and rich in minerals, farming could be carried on for many years without the return of any nutrients to the soil other than those supplied through the natural breakdown of plant and animal wastes. In river basins, such as that of the Nile, annual flooding deposited a rich layer of silt over the soil, thus restoring its fertility. In areas of active volcanism, such as Hawaii, soil fertility has been renewed by the periodic deposition of volcanic ash. In other areas, however, natural fertility has been quickly exhausted. This is true of most forest soils, particularly those in the humid tropics. Because continued cropping in such areas caused a rapid decline in fertility and therefore in crop yields, fertility could be restored only by abandoning the areas and allowing the natural forest vegetation to return. Over a period of time, the soil surface would be rejuvenated by parent materials, new circulation channels would form deep in the soil, and the deposition of forest debris would restore minerals to the topsoil. Primitive agriculture in such forests was of shifting nature: areas were cleared of trees and the woody material burned to add ash to the soil; after a few years of farming, the plots would be abandoned and new sites cleared. As long as populations were sparse in relation to the area of forestland, such agricultural methods did little harm. They could not, however, support dense populations or produce large quantities of surplus foods. Starting with the most easily depleted soils, which were also the easiest to farm, the practice of using various fertilizers was developed. The earliest fertilizers were organic manures, but later, larger yields were obtained by adding balanced combinations of those nutrients (e.g. potassium, nitrogen, phosphorus and calcium) that crop plants require in greatest quantity. Because high yields are essential, most modern agriculture depends upon the continued addition of chemical fertilizers to the soil. Usually these substances are added in mineral form, but nitrogen is often added as urea, an organic compound. Early in agricultural history, it was found that the practice of growing the same crop year after year in a particular plot of ground not only caused undesirable changes in the physical structure of the soil, but also drained the soil of its nutrients. The practice of crop rotation was discovered to be a useful way to maintain the condition of the soil, and also to prevent the build-up of those insects and other plant pests that are attracted to a particular kind of crop. In rotation systems, a grain crop is often grown the first year, followed by a leafy vegetable crop in the second year, and pasture crop in the third. The last usually contains legumes (e.g. clover, alfalfa), because such plants can restore nitrogen to the soil through the action of bacteria that live in nodules on their roots. In irrigation agriculture, in which water is brought in to supply the needs of crops in an area with insufficient rainfall, a particular soil-management problem that develops is the salinization (concentration of salts) of the surface soil. This most commonly results from inadequate drainage of the irrigated land; because the water cannot flow freely, it evaporates, and the salts dissolved in the water are left on the surface of the soil. Even though the water does not contain a large concentration of dissolved salts, the accumulation over the years can be significant enough to make the soil unsuitable for crop production. Effective drainage solves the problem; in many cases, drainage canals must be constructed, and drainage tiles must be laid beneath the surface of the soil. Drainage also requires the availability of an excess of water to flush the salts from the surface soil. In certain heavy soils with poor drainage, this problem can be quite severe; for example, large areas of formerly irrigated land in the Indus basin, in the Tigris-Euphrates region, in the Nile Basin, and in the Western United States, have been seriously damaged by salinization.

 

 

 

Q. 11 The areas most prone to salinization are

A. those irrigated with well-water.

B. those in which crop rotation is not practiced.

C. sub-tropical forests.

D. flat land irrigated from reservoirs.

 

Q. 12 The most appropriate title to his passage is

A. Problems of soil erosion

B. Agriculture in Volcanic islands.

C. The importance of chemical fertilizers.

D. Causes of and remedies of soil-infertility.

 

Q. 13 Natural fertility exhausts most quickly in

A. river valley lands

B. humid tropical forests

C. volcanic areas

D. lands near urban areas

 

Q. 14 The factor that can restore fertility to the soil not mentioned in the passage is

A. alluvium brought by rivers

B. bacterial action

C. fertilizer fixation through lightning

D. organic manure

 

Q. 15 Crop rotation helps to

I. increase the farmer’s seasonal income.

II. preserve soil condition.

III. desalinate the soil.

IV. destroy pests

A. I, II, III & IV

B. I, II & IV only.

C. II & IV only

D. II, III & IV only

 

Q. 16 One of the characteristics of agricultural land in Nile basin is

A. it contains a lot of bacteria.

B. it consists of heavy soil with poor drainage properties.

C. the Nile water contains an excess of salts.

D. it contains nutritive minerals.

 

Questions: 17 – 21

Passage:-

Scientism has left humanity in our technical mastery of inanimate nature, but improvised us in our quest for an answer to the riddle of the universe and of our existence in it. Scientism has done worse than that with respect to our status as social beings, that is, to our life with our fellow human beings. The quest for the technical mastery of social life, comparable to our mastery over nature, did not find scientism at a loss for an answer: reason suggested that physical nature and social life were fundamentally alike and therefore proposed identical methods for their domination. Since reason in the form of causality reveals itself most plainly in nature, nature became the model for the social world and the natural sciences the image of what the social sciences one day would be. According to scientism, there was only one truth, the truth of science, and by knowing it, humanity would know all. This was, however, a fallacious argument, its universal acceptance initiated an intellectual movement and a political technique which retarded, rather than furthered, human mastery of the social world. The analogy between the natural and social worlds is mistaken for two reasons. On the one hand human action is unable to model the social world with the same degree of technical perfection that is possible in the natural world. On the other hand, the very notion that physical nature is the embodiment of reason from which the analogy between natural and social worlds derives, is invalidated by modern scientific thought itself. Physical nature, as seen by the practitioner of science consists of a multitude of isolated facts over which human action has complete control. We know that water boils at a temperature of 212 degrees Fahrenheit and, by exposing water to this temperature, we can make it boil at will. All practical knowledge of physical nature and all control over it are essentially of the same kind. Scientism proposed that the same kind of knowledge and of control held true for the social world. The search for a single cause, in the social sciences, was but a faithful copy of the method of the physical sciences. Yet in the social sphere, the logical coherence of the natural sciences finds no adequate object and there is no single cause by the creation of which one can create a certain effect at will. Any single cause in the social sphere can entail an indefinite number of different effects, and the same effect can spring from an indefinite number of different effects, and the same effect can spring from an indefinite number of different causes.

 

 

Q. 17 The author’s attitude towards the application of scientism to the social sciences is best described as one of

A. committed scrutiny

B. dismissal

C. criticism

D. approval

 

Q. 18 According to the author, causes and effects in the social world are

A. unrelated to each other

B. difficult to identify or predict.

C. subject to manipulation at will.

D. reducible to a single cause for each effect.

 

Q. 19 Which of the following statements about scientism is best supported by the passage?

A. Scientism provides the basis for mastery of the social world

B. Scientism is only superficially concerned with cause-and – effect relationships

C. Scientism is poorly suited to explain social behaviour

D. Scientism is no longer applicable to the study of the natural sciences.

 

Q. 20 As is used in the passage, the term ‘scientism’ can best be defined as

A. belief that the methods of the physical sciences can be applied to all fields of enquiry.

B. faith that human beings can master their own physical limitations.

C. desire to keep the social sciences separate from the physical sciences

D. opinion that scientists must take moral responsibility for their actions

 

Q. 21 In the passage, the author is most concerned with doing which of the following?

A. Upholding the primacy of reason over superstition

B. Attacking a particular approach to the social sciences

C. Describing a method for achieving control over human social behaviour

D. Demonstration the superiority of the social sciences over the natural sciences

 

Questions: 22 – 25

Passage:-

From a vantage point in space, an observer could see that the Earth is engaged in a variety of motions. First, there is its rotation on its own axis, causing the alternation of day and night. This rotation, however, is not altogether steady. Primarily because of the moon’s gravitational action, the Earth‘s axis wobbles like that of an ill-spun top. In this motion, called ‘precession’, the North and South Poles each traces out the base of a cone in space, completing a circle every 25,800 years, In addition, as the Sun and the Moon change their positions with respect to the Earth, their changing gravitational effects result in a slight ‘nodding’ of the earth’s axis, called ‘mutation’, which is superimposed on precession. The Earth completes one of these ‘nods’ every 18.6 years. The earth also, of course, revolves around the Sun, in a 6-million mile journey that takes 365.25 days. The shape of this orbit is an ellipse, but it is not the center of the Earth that follows the elliptical path. Earth and Moon behave like an asymmetrical dumb-bell, and it is the center of mass of this dumb-bell that traces the ellipse around the sun. The center of the Earth-Moon mass lies about 3000 miles away from the center of the Earth, and the Earth thus moves in an S-curve that crosses and uncrosses its orbital path. Then too, the Earth accompanies the sun in the sun’s movements: 

first, through its local star cloud, and second, in a great sweep around the hub of its galaxy, the Milky Way that takes 200 million years to complete. 

 

 

Q. 22 The passage is most likely directed towards an audience of

A. geologists.

B. astronauts.

C. meteorologists interested in weather prediction.

D. person with little technical knowledge of astronomy.

 

Q. 23 Which of the following best describes the main subject of the passage?

A. The various types of the Earth’s motions

B. Past changes in the Earth’s position

C. The moon gravitational effect on the earth

D. Oddities of the Earth’s rotation of its axis.

 

Q. 24 The passage indicates that a single cycle of which of the following motions is completed in the shortest period of time?

A. Mutation.

B. Precession.

C. The Earth’s rotation on its axis.

D. The movement of the dumb-bell formed by the center of mass of Earth-Moon.

 

Q. 25 Which of the following techniques does the author use in order to make the descriptions of motion clear?

I. Comparison with familiar objects.

II. Reference of geometric forms.

III. Allusions to the works of other authors.

A. I only

B. II only

C. I and II only

D. II and III only

 

Questions: 26 – 29

Passage:-

The connective tissues are heterogeneous group of tissues derived from the mesenchyme, a meshwork of stellate cells that develop in the middle layer of the early embryo. They have the general function of maintaining the structural integrity of organs, and providing cohesion and internal support for the body as a whole. The connective tissues include several types of fibrous tissue that vary only in their density and cellularity, as well as more specialized variants ranging from adipose tissue through cartilage to bone. The cells that are responsible for the specific function of an organ are referred to as it parenchyma, while the delicate fibrous meshwork that blinds the cells together into functional units, the fibrous partitions or septa that enclose aggregations of functional units, and the dense fibrous capsule that encloses the whole organ, collectively make up its connective-tissue framework, or stroma. Blood vessels, both large and small, course through connective tissues, which is therefore closely associated with the nourishment of tissues and organs throughout the body. All nutrient materials and waste products exchanged between the organs and the blood must traverse perivascular spaces occupied by connective tissue. One of the important functions of the connective – tissue cells is to maintain conditions in the extracellular spaces that favour this exchange. Some organs are suspended from the wall of a body cavity by thin sheets of connective tissues called mesenteries; others are embedded in adipose tissue a form of a connective tissue in which the cells are specialized for the synthesis and storage of energy-rich reserves of fat, or lipid. The entire body is supported from within by a skeleton composed of bone, a type of connective tissue endowed with great resistance to stress owing to its highly ordered, laminated structure and to its hardness, which results from deposition of mineral salts in its fibres and amorphous matrix. The individual bones of the skeleton are held firmly together by ligaments, and muscles are attached to bone by tendons, both of which are examples of dense connective tissue in which many fibre bundles are associated in parallel array to provide great tensile strength. At joints, the articular surfaces of the bones are covered with cartilage, a connective tissue with an abundant intercellular substance that gives it a firm consistency well adopted to permit smooth gliding movements between the opposed surfaces. The synovial membrane, which lines the margins of the joint cavity and lubricates and nourishes the joint surfaces, is also a form of connective tissue.

 

 

Q. 26 The passage has most probably been taken from a book on

A. neurology

B. nutrition

C. physiology

D. calisthenics

 

Q. 27 Mesenteries are

A. adipose tissue in which some organs are embedded.

B. referred to as parenchyma, and are responsible for specific functions of an organ.

C. thin sheets from which some organs are suspended.

D. cells through which blood flows.

 

Q. 28 Through perivascular spaces exchange takes place between

A. blood and organs.

B. cells and embryo.

C. nutrients and waste products.

D. septa and stroma.

 

Q. 29 Some instances of connective tissues are

I. Cartilage

II. Stroma

III. Lipid

IV. Synovia

A. I, II, III & IV

B. I, III & IV only

C. I, II, & IV only

D. I and II only

 

Questions: 30 – 34

Passage:-

Emile Durkheim, the first person to be formally recognized as a sociologist and the most scientific of the pioneers, conducted a study that stands as a research model for sociologists today. His investigation of suicide was, in fact, the first sociological study to use statistics. In suicide (1964, originally published in 1897) Durkheim documented his contention that some aspects of human behaviour – even something as allegedly individualistic as suicide – can be explained without reference to individuals. Like all of Durkheim’s work, suicide must be viewed within the context of his concern for social integration. Durkheim wanted to see if suicide rates within a social entity (for example, a group, organization, or society) are related to the degree to which individuals are socially involved (integrated and regulated). Durkheim describes three types of suicide: egoistic, anomic, and altruistic. Egoistic suicide is promoted when individuals do not have sufficient social ties. Since single (never married) adults, for example, are not heavily involved with the family life, they are more likely to commit suicide than are married adults. Altruistic suicide on the other hand, is more likely to occur when social integration is too strong. The ritual suicide of Hindu widows on their husbands’ funeral pyres is one example. Military personnel, trained to lay down their lives for their country, provide another illustration. Durkheim’s third type of suicide – anomic suicide increases when the social regulation of individuals is disrupted. For example, suicide rates increase during economic depressions. People who suddenly find themselves without a job or without hope of finding one are more prone to kill themselves. Suicides may also increase during period of prosperity. People may loosen their social ties by taking new jobs, moving to new communities, or finding new mates. Using data from the government population reports of several countries (much of it from the French Government Statistical Office), Durkheim found strong support for his line reasoning. Suicide rates were higher among single than married people, among military personnel than civilians, among divorced than married people, and among people involved in nationwide economic crises. It is important to realize that Durkheim’s primary interest was not in the empirical (observations) indicators he used such as suicide rates among military personnel, married people, and so forth. Rather, Durkheim used the following indicators to support several of his contentions: (1) Social behaviour can be explained by social rather than psychological factors; (2) suicide is affected by the degree of integration and regulation within social entities; and (3) Since society can be studied scientifically, sociology is worthy of recognition in the academic world. Durkheim was successful on all three counts. 

 

 

Q. 30 In his study of suicide Durkheim’s main purpose was

A. to document that suicide can be explained without reference to the individual.

B. to provide an explanation of the variation in the rate of suicide across societies.

C. to categorize various types of suicides.

D. to document that social behavior can be explained by social rather than psychological factors.

 

Q. 31 Single adults not heavily involved with family life are more likely to commit suicide. Durkheim categorized this as

A. anomic suicide.

B. altruistic suicide.

C. egoistic suicide.

D. Both (b) and (c)

 

Q. 32 According to Durkheim, suicide rates within a social entity can be explained in terms of

A. absence of social ties.

B. disruption of social regulation.

C. nature of social integration

D. All of the above.

 

Q. 33 Basing himself on his own indicators. Durkheim was

A. right on some counts, not others.

B. vindicated on all counts.

C. wrong but did not realize that he was right.

D. substantially correct but formally wrong.

 

Q. 34 To support his contentions, Durkheim relied on the following indicators

A. social behaviour is explicable predominantly through social factors.

B. suicide is contingent upon the degree of regulation and interaction.

C. recognizing sociology is to acknowledge that society is susceptible to scientific investigation.

 

D. All of the above.

 

Questions: 35 – 38

Recently, the answers of a test held nationwide were leaked to a group of unscrupulous people. The investigative agency has arrested the mastermind and nine other people A, B, C, D, E, F, G, H and I in this matter. Interrogating them, the following facts have been obtained regarding their operation. Initially the mastermind obtains the correct answer-key. All the others create their answerkey from one or two people who already possess the same. These people are called his/her “sources”. If the person has two sources, then he/she compares the answer keys obtained from both sources. If the key to a question from both sources is identical, it is copied, otherwise it is left blank. If the person has only one source, he/she copies the source’s answers into his/her copy. Finally, each person compulsorily replaces one of the answers (not a blank one) with a wrong answer in his/her answer key. The paper contained 200 questions; so the investigative agency has ruled out the possibility of two or more of them introducing wrong answers to the same question. The investigative agency has a copy of the correct answer key and has tabulated the following data. These data represent question numbers.

 

 

Q. 35 Which one among the following must have two sources?

A. A

B. B

C. C

D. D

 

Q. 36 How many people (excluding the mastermind) needed to make answer keys before C could make his answer key?

A. 2

B. 3

C. 4

D. 5

 

Q. 37 Both G and H were sources to

A. F

B. B

C. I

D. None of the nine

 

Q. 38 Which of the following statements is true?

A. C introduced the wrong answer to question 27.

B. E introduced the wrong answer to question 46.

C. F introduced the wrong answer to question 14.

D. H introduced the wrong answer to question 46.

 

Questions: 39 – 42

The table below gives information about four different crops, their different quality categories and the regions where they are cultivated. Based on the information given in the table answer the questions given below 

 

Q. 39 How many regions produce medium qualities of Crop-1 or Crop-2 and also produce low quality of Crop-3 or Crop-4?

A. R4

B. R9

C. R8

D. R13

 

Q. 40 Which of the following statements is true?

A. All medium quality Crop-2 producing regions are also high quality Crop-3 producing regions.

B. All high quality Crop-1 producing regions are also medium and low Crop-4 producing regions. 

C. There are exactly five Crop-3 producing regions, which also produce Crop-4 but not Crop-2.

D. Some Crop-3, producing regions produce Crop-1, and high quality Crop-2.

 

Q. 41 How many low quality Crop-1 producing regions are either high quality Crop-4 producing regions or medium quality Crop-3 producing regions?

A. 1

B. 2

C. 3

D. 4

 

Q. 42 How many high quality Crop 4 producing regions are there?

A. 3

B. 4

C. 9

D. 11

 

Questions: 43 – 46

A study was conducted to ascertain the relative importance that employees in five different countries assigned to five different traits in their Chief Executive Officers. The traits were compassion (C), decisiveness (D), negotiation skills (N), public visibility (P), and vision (V). The level of dissimilarity between two countries is the maximum difference in the ranks allotted by the two countries to any of the five traits. The following table indicates the rank order of the five traits for each country.

 

Q. 43 Three of the following four pairs of countries have identical levels of dissimilarity. Which pair is the odd one out?

A. Malaysia & China

B. China & Thailand

C. Thailand & Japan

D. Japan & Malaysia

 

Q. 44 Which amongst the following countries is most dissimilar to India?

A. China

B. Japan

C. Malaysia

D. Thailand

 

Q. 45 Which of the following countries is least dissimilar to India?

A. China

B. Japan

C. Malaysia

D. Thailand

 

Q. 46 Which of the following pairs of countries are most dissimilar?

A. China & Japan

B. India & China

C. Malaysia & Japan

D. Thailand & Japan

 

Questions: 47 – 50

Purana and Naya are two brands of kitchen mixer-grinders available in the local market. Purana is an old brand that was introduced in 1990, while Naya was introduced in 1997. For both these brands, 20% of the mixer-grinders bought in a particular year are disposed off as junk exactly two years later. It is known that 10 Purana mixer-grinders were disposed off in 1997. The following figures show the number of Purana and Naya mixer-grinders in operation from 1995 to 2000, as at the end of the year.

 

Q. 47 How many Naya mixer-grinders were disposed off by the end of 2000?

A. 20

B. 124

C. 50

D. 74

 

Q. 48 How many Naya mixer-grinders were purchased in 1999?

A. 6

B. 10

C. 16

D. 17

 

Q. 49 How many Purana mixer-grinders were purchased in 1999?

A. 20

B. 24

C. 30

D. Cannot be determined

 

Q. 50 How many Purana mixer-grinders were disposed off in 2000?

A. 16

B. 20

C. 24

D. Cannot be determined

 

Questions: 51 – 54

The year was 2006. All six teams in Pool A of World Cup hockey, play each other exactly once. Each win earns a team three points, a draw earns one point and a loss earns zero points. The two teams with the highest points qualify for the semi-finals. In case of a tie, the team with the highest goal difference (Goal For – Goals Against) qualifies. In the opening match, Spain lost to Germany. After the second round (after each team played two matches), the pool table looked as shown below and no match was a draw. In the third round, Spain played Pakistan, Argentina played Germany, and New Zealand played South Africa. All the third round matches were drawn. The following are some results from the fourth and fifth round matches. (a) Spain won both the fourth and fifth round matches. (b) Both Argentina and Germany won their fifth round matches by 3 goals to 0. (c) Pakistan won both the fourth and fifth round matches by 1 goal to 0.

 

Q. 51 Which one of the following statements is true about matches played in the first two rounds?

A. Pakistan beat South Africa by 2 goals to 1.

B. Argentina beat Pakistan by 1 goal to 0.

C. Germany beat Pakistan by 2 goals to 1.

D. Germany beat Spain by 2 goals to 1.

 

Q. 52 Which one of the following statements is true about matches played in the first two rounds?

A. Germany beat New Zealand by 1 goal to 0.

B. Spain beat New Zealand by 4 goals to 0.

C. Spain beat South Africa by 2 goals to 0.

D. Germany beat South Africa by 2 goals to 1.

 

Q. 53 If Pakistan qualified as one of the two teams from Pool A, which was the other team that qualified?

A. Argentina

B. Germany

C. Spain

D. Cannot be determined

 

Q. 54 Which team finished at the top of the pool after five rounds of matches?

A. Argentina

B. Germany

C. Spain

D. Cannot be determined

 

Questions: 55 – 58 

Venkat, a stockbroker, invested a part of his money in the stock of four companies – A, B, C and D. Each of these companies belonged to different industries, viz., Cement, Information Technology (IT), Auto, and Steel, in no particular order. At the time of investment, the price of each stock was Rs.100. Venkat purchased only one stock of each of these companies. He was expecting returns of 20%, 10%, 30%, and 40% from the stock of companies A, B, C and D, respectively. Returns arc defined as the change in the value of the stock after one year, expressed as a percentage of the initial value. During the year, two of these companies announced extraordinarily good results. One of these two companies belonged to the Cement or the IT industry, while the other one belonged to either the Steel or the Auto industry. As a result, the returns on the stocks of these two companies were higher than the initially expected returns. For the company belonging to the Cement or the IT industry with extraordinarily good results, the returns were twice that of the initially expected returns. For the company belonging to the Steel or the Auto industry, the returns on announcement of extraordinarily good results were only one and a half times that of the initially expected returns. For the remaining two companies, which did not announce extraordinarily good results, the returns realized during the year were the same as initially expected.

 

 

Q. 55 What is the minimum average return Venkat would have earned during the year?

A. 30%

B. 31.25%

C. 32.5%

D. Cannot be determined

 

Q. 56 If Venkat earned a 35% return on average during the year, then which of these statements would necessarily be true?

I. Company A belonged either to Auto or to Steel Industry.

II. Company B did not announce extraordinarily good results.

III. Company A announced extraordinarily good results.

IV. Company D did not announce extraordinarily good results.

A. I and II only

B. II and III only

C. I and IV only

D. II and IV only

 

Q. 57 If Venkat earned a 38.75% return on average during the year, then which of these statement(s) would necessarily be true?

I. Company C belonged either to Auto or to Steel Industry.

II. Company D belonged either to Auto or to Steel Industry.

III. Company A announced extraordinarily good results.

IV. Company B did not announce extraordinarily good results.

A. I and II only

B. II and III only

C. I and IV only

D. II and IV only

 

Q. 58 If Company C belonged to the Cement or the IT industry and did announce extraordinarily good results, then which of these statement(s) would necessarily be true?

I. Venkat earned not more than 36.25% return on average.

II. Venkat earned not less than 33.75% return on average.

III. If Venkat earned 33.75% return on average, Company A announced extraordinarily good results.

IV. If Venkat earned 33.75% return on average, Company B

A. I and II only

B. II and IV only

C. II and III only

D. III and IV only

 

Questions: 59 – 62

The year is 2089. Beijing, London, New York, and Paris are in contention lo host the 2096 Olympics. The eventual winner is determined through several rounds of voting by members of the IOC with each member representing a different city. All the four cities in contention are also represented in IOC. In any round of voting, the city receiving the lowest number of votes in that round gets eliminated. The survivor after the last round of voting gets to host the event. A member is allowed to east votes for at most two different cities in all rounds of voting combined. (Hence, a member becomes ineligible to cast a vote in a given round if both the cities (s)he voted for in earlier rounds are out of contention in that round of voting). A member is also ineligible to cast a vote in a round if the city (s)he represents is in contention in that round of voting. As long as the member is eligible, (s)he must vote and vote for only one candidate city in any round of voting. The following incomplete table shows the information on cities that received the maximum and minimum votes in different rounds, the number of votes cast in their favour, and the total votes that were cast in those rounds. It is also known that: All those who voted for London and Paris in round 1, continued to vote for the same cities in subsequent rounds as long as these cities were in contention. 75% of those who voted for Beijing in round 1, voted for Beijing in round 2 as well. Those who voted for New York in round 1, voted either for Beijing or Paris in round 2. The difference in votes cast for the two contending cities in the last round was 1. 50% of those who voted for Beijing in round 1, voted for Paris in round 3. 

 

Q. 59 What percentage of members from among those who voted for New York in round I, voted for Beijing in round 2?

A. 33.33

B. 50

C. 66067

D. 75

 

Q. 60 What is the number of votes cast for Paris in round 1?

A. 16

B. 18

C. 22

D. 24

 

Q. 61 What percentage of members from among those who voted for Beijing in round 2 and were eligible to vote in round 3, voted for London?

A. 33.33

B. 38.10

C. 50

D. 66067

 

Q. 62 Which of the following statements must be true?

a. IOC member from New York must have voted for Paris in round 2.

b. IOC member from Beijing voted for London in round 3.

A. a

B. b

C. Both

D. None

 

Questions: 63 – 66

The table below presents the revenue (in million rupees) of four firms in three states. These firms, Honest Ltd., Aggressive Ltd., Truthful Ltd. and Profitable Ltd. are disguised in the table as A, B, C and D, in no particular order. Further, it is known that: In the state of MP, Truthful Ltd. has the highest market share. Aggressive Ltd.’s aggregate revenue differs from Honest Ltd.’s by Rs. 5 million.

 

Q. 63 What can be said regarding the following two statements?

Statement 1: Profitable Ltd. has the lowest share in MP market.

Statement 2: Honest Ltd.’s total revenue is more than Profitable Ltd.

A. If Statement 1 is true then Statement 2 is necessarily true.

B. If Statement 1 is true then Statement 2 is necessarily false.

C. Both Statement 1 and Statement 2 are true.

D. Neither Statement 1 nor Statement 2 is true.

 

Q. 64 What can be said regarding the following two statements?

Statement 1: Aggressive Ltd.’s lowest revenues are from MP.

Statement 2: Honest Ltd.’s lowest revenues are from Bihar.

A. If Statement 2 is true then Statement 1 is necessarily false.

B. If Statement 1 is false then Statement 2 is necessarily true.

C. If Statement 1 is true then Statement 2 is necessarily true.

D. None of the above.

 

Q. 65 What can be said regarding the following two statements?

Statement 1: Honest Ltd. has the highest share in the UP market.

Statement 2: Aggressive Ltd. has the highest share in the Bihar market.

A. Both statements could be true.

B. At least one of the statements must be true.

C. At most one of the statements is true.

D. None of the above

 

Q. 66 If Profitable Ltd.’s lowest revenue is from UP, then which of the following is true?

A. Truthful Ltd.’s lowest revenues are from MP.

B. Truthful Ltd.’s lowest revenues are from Bihar.

C. Truthful Ltd.’s lowest revenues are from UP.

D. No definite conclusion is possible.

 

Q. 67 Direction for questions: Answer the questions based on the following information. In a locality, there are five small cities: A, B, C, D and E. The distances of these cities from each other are as follows. AB = 2 km; AC = 2km; AD > 2 km; AE > 3 km; BC = 2 km; BD = 4 km; BE = 3 km; CD = 2 km; CE = 3 km; DE > 3 km. If a ration shop is to be set up within 3 km of each city, how many ration shops will be required?

A. 1

B. 2

C. 3

D. 4

 

Q. 68 A cube of side 12 cm is painted red on all the faces and then cut into smaller cubes, each of side 3 cm. What is the total number of smaller cubes having none of their faces painted? 

A. 4

B. 8

C. 16

D. 24

 

Q. 69 If ABCD is a square and BCE is an equilateral triangle, what is the measure of ∠DEC? 

A. 15°

B. 30°

C. 20°

D. 45°

 

Q. 70 Instead of a metre scale, a cloth merchant uses a 120 cm scale while buying, but uses an 80 cm scale while selling the same cloth. If he offers a discount of 20% on cash payment, what is his overall profit percentage?

A. 20%

B. 25%

C. 40%

D. 15%

 

Q. 71 From a circular sheet of paper with a radius 20 cm, four circles of radius 5 cm each are cut out. What is the ratio of the uncut to the cut portion?

A. 1:3

B. 4:1

C. 3:1

D. 4:3

 

Q. 72 A wooden box (open at the top) of thickness 0.5 cm, length 21 cm, width 11 cm and height 6 cm is painted on the inside. The expenses of painting are Rs. 70. What is the rate of painting per square centimetres?

A. Re. 0.7

B. Re. 0.5

C. Re. 0.1

D. Re. 0.2

 

Q. 73 A, S, M and D are functions of x and y, and they are defined as follows.

A(x, y) = x + y; S(x, y) = x – y;

M(x, y) = xy; D(x, y) = x / y , y ≠ 0;

What is the value of M(M(A(M(x, y), S(y, x)), x), A(y, x)) for x = 2, y = 3?

A. 60

B. 140

C. 25

D. 70

 

Q. 74 The cost of diamond varies directly as the square of its weight. Once, this diamond broke into four pieces with weights in the ratio 1 : 2 : 3 : 4. When the pieces were sold, the merchant got Rs. 70,000 less. Find the original price of the diamond.

A. Rs. 1.4 lakh

B. Rs. 2 lakh

C. Rs. 1 lakh

D. Rs. 2.1 lakh

 

Q. 75 If n is any odd number greater than 1, then n(n2 – 1) is

A. divisible by 96 always

B. divisible by 48 always

C. divisible by 24 always

D. None of these

 

Q. 76 The figure shows a circle of diameter AB and radius 6.5 cm.

If chord CA is 5 cm long, find the area of ΔABC.

A. 78

B. 30

C. 45

D. 75


Q. 77 A watch dealer incurs an expense of Rs. 150 for producing every watch. He also incurs an additional expenditure of Rs. 30,000, which is independent of the number of watches produced. If he is able to sell a watch during the season, he sells it for Rs. 250. If he fails to do so, he has to sell each watch for Rs. 100. If he is able to sell only 1,200 out of 1,500 watches he has made in the season, then he has made a profit of

A. Rs. 90,000

B. Rs. 75,000

C. Rs. 45,000

D. Rs. 60,000

 

Q. 78 Once I had been to the post office to buy five-rupee, two rupee and one-rupee stamps. I paid the clerk Rs. 20, and since he had no change, he gave me three more one-rupee stamps. If the number of stamps of each type that I had ordered initially was more than one, what was the total number of stamps that I bought?

A. 10

B. 20

C. 16

D. 7

 

Q. 79 In ΔABC, ∠B is a right angle, AC = 6 cm, and D is the midpoint of AC. The length of BD is 

A. 1.5

B. 2

C. 3

D. 6

 

Q. 80 A salesman enters the quantity sold and the price into the computer. Both the numbers are two-digit numbers. But, by mistake, both the numbers were entered with their digits interchanged. The total sales value remained the same, i.e. Rs. 1,148, but the inventory reduced by 54. What is the actual price per piece?

A. Rs. 82

B. Rs. 41

C. Rs. 6

D. Rs. 28

 

Q. 81 In a locality, two-thirds of the people have cable TV, one-fifth have VCR, and one-tenth have both. What is the fraction of people having either cable -TV or VCR?

A. 19/30

B. 2/3

C. 17/30

D. 23/30

 

Q. 82 Given the quadratic equation x² – (A – 3)x – (A – 2), for what value of A will the sum of the squares of the roots be zero?

A. -2

B. 3

C. 6

D. None

 

Q. 83 If a₁ = 1 and aₙ₊₁ – 3aₙ + 2 = 4n for every positive integer n, then a₁₀₀ equals

A. 3⁹⁹ – 200

B. 3⁹⁹ + 200

C. 3¹⁰⁰ – 200

D. 3¹⁰⁰ + 200

 

Q. 84 In a mile race, Akshay can be given a start of 128 m by Bhairav. If Bhairav can give Chinmay a start of 4 m in a 100 m dash, then who out of Akshay and Chinmay will win a race of one and half miles, and what will be the final lead given by the winner to the loser? (One mile is 1,600 m.)

A. Akshay, 1/12 mile

B. Chinmay, 1/32 mile

C. Akshay, 1/24 mile

D. Chinmay, 1/16 mile

 

Q. 85 Two liquids A and B are in the ratio 5 : 1 in container 1 and 1 : 3 in container 2. In what ratio should the contents of the two containers be mixed so as to obtain a mixture of A and B in the ratio 1 : 1?

A. 2:3

B. 4:3

C. 3:2

D. 3:4

 

Q. 86 A man travels three-fifths of a distance AB at a speed 3a, and the remaining at a speed 2b. If he goes from B to A and return at a speed 5c in the same time, then

A. 1/a + 1/b = 1/c

B. a + b = c

C. 1/a + 1/b = 2/c

D. None of these

 

Q. 87 There are five machines A, B C, D and E situated on a straight line at distances of 10 metres, 20 metres, 30 metres, 40 metres and 50 metres respectively from the origin of the line. A robot is stationed at the origin of the line. The robot serves the machines with raw material whenever a machine becomes idle. All the raw material is located at the origin. The robot is in an idle state at the origin at the beginning of a day. As soon as one or more machines become idle, they send messages to the robot-station and the robot starts and serves all the machines from which it received messages. If a message is received at the station while the robot is away from it, the robot takes notice of the message only when it returns to the station. While moving, it serves the machines in the sequence in which they are encountered, and then returns to the origin. If any messages are pending at the station when it returns, it repeats the process again. Otherwise, it remains idle at the origin till the next message(s) is received. Suppose on a certain day, machines A and D have sent the first two messages to the origin at the beginning of the first second, and C has sent a message at the beginning of the 5th second and B at the beginning of the 6th second, and E at the beginning of the 10th second. How much distance in metres has the robot travelled since the beginning of the day, when it notices the message of E? Assume that the speed of movement of the robot is 10 metres per second.

A. 140

B. 80

C. 340

D. 360

 

Q. 88 Out of two-thirds of the total number of basketball matches, a team has won 17 matches and lost 3 of them. What is the maximum number of matches that the team can lose and still win more than three fourths of the total number of matches, if it is true that no match can end in a tie?

A. 4

B. 2

C. 6

D. 3

 

Q. 89 What value of x satisfy x²/³ + x¹/³ – 2 ≤ 0?

A. –8 ≤ x ≤ 1

B. –1 ≤ x ≤ 8

C. 1 < x < 8

D. 1 ≤ x ≤ 8

E. –8 ≤ x ≤ 8

 

Q. 90 The points of intersection of three lines 2X + 3Y – 5 = 0, 5X – 7Y + 2 = 0 and 9X – 5Y – 4= 0 

A. form a triangle

B. are on lines perpendicular to each other

C. are on lines parallel to each other

D. are coincident

 

Q. 91 A man has 9 friends: 4 boys and 5 girls. In how many ways can he invite them, if there have to be exactly 3 girls in the invitees?

A. 150

B. 160

C. 180

D. 200

 

Q. 92 In a watch, the minute hand crosses the hour hand for the third time exactly after every 3 hr 18 min and 15 s of watch time. What is the time gained or lost by this watch in one day? 

A. 14 min 10s lost

B. 13 min 50s lost

C. 13 min 20s gained

D. 14 min 40s gained

 

Q. 93 I sold two watches for Rs. 300 each, one at the loss of 10% and the other at the profit of 10%. What is the percentage of loss(–) or profit(+) that resulted from the transaction? 

A. (+)10

B. (–)1

C. (+)1

D. (–)10

 

Questions: 94 – 95

A series S1 of five positive integers is such that the third term is half the first term`and the fifth term is 20 more than the first term. In series S2, the nth term defined as the difference between the (n+1) term and the nth term of series S1, is an arithmetic progression with a common difference of 30.

 

 

Q. 94 Second term of S2 is

A. 50

B. 60

C. 70

D. None of these

 

Q. 95 What is the average value of the terms of series S1?

A. 60

B. 70

C. 80

D. None of these

 

Q. 96 If log₁₀x – log₁₀√x = 2 logₓ10, then a possible value of x is given by

A. 10

B. 1/100

C. 1/1000

D. None of these

 

Q. 97 What is the sum of all two-digit numbers that give a remainder of 3 when they are divided by 7?

A. 666

B. 676

C. 683

D. 777

 

Q. 98 There are 12 towns grouped into four zones with three towns per zone. It is intended to connect the towns with telephone lines such that every two towns are connected with three direct lines if they belong to the same zone, and with only one direct line otherwise. How many direct telephone lines are required?

A. 72

B. 90

C. 96

D. 144

 

Q. 99 If both a and b belong to the set {1, 2, 3, 4}, then the number of equations of the form ax² + bx + 1 = 0 having real roots is

A. 5

B. 7

C. 9

D. 11

 

Q. 100 If three positive real numbers x, y, z satisfy y – x = z – y and xyz = 4, then what is the minimum possible value of y?

A. 21/3

B. 22/3

C. 21/4

D. 23/4

 

 

Answer Sheet 
Question 1 2 3 4 5 6 7 8 9 10
Answer B C A B E C E E C C
Question 11 12 13 14 15 16 17 18 19 20
Answer D D B C C B C B C A
Question 21 22 23 24 25 26 27 28 29 30
Answer B D A C C C C A C A
Question 31 32 33 34 35 36 37 38 39 40
Answer C D B D B C D C B C
Question 41 42 43 44 45 46 47 48 49 50
Answer C B C A B C B C D B
Question 51 52 53 54 55 56 57 58 59 60
Answer D B D D A B C B D D
Question 61 62 63 64 65 66 67 68 69 70
Answer D A B C C C A B A A
Question 71 72 73 74 75 76 77 78 79 80
Answer C C D C C B B A C B
Question 81 82 83 84 85 86 87 88 89 90
Answer B D C D D C A A A D
Question 91 92 93 94 95 96 97 98 99 100
Answer B B B D C B B B B B

CAT Previous Year Paper 2015

CAT 2015

Section

Questions

Marks

English

34 Questions (1 – 34)

102

DILR

32 Questions (35 – 66)

96

Quantitative Aptitude

34 Questions (67 – 100)

102

Questions: 1 – 6

Language is not a cultural artifact that we learn the way we learn to tell time or how the federal government works. Instead, it is a distinct piece of the biological makeup of our brains. Language is a complex, specialized skill, which develops in the child spontaneously, without conscious effort or formal instruction, is deployed without awareness of its underlying logic, is qualitatively the same in every individual, and is distinct from more general abilities to process information or behave intelligently. For these reasons some cognitive researchers have described language as a psychological faculty, a mental organ, a neural system, and a computational module. But I prefer the admittedly quaint term “instinct”. It conveys the idea that people know how to talk in more or less the sense that spiders know how to spin webs. Web-spinning was not invented by some unsung spider genius and does not depend on having had the right education or on having an aptitude for architecture or the construction trades. Rather, spiders spin spider webs because they have spider brains, which give them the urge to spin and the competence to succeed. Although there are differences between webs and words, I will encourage you to see language in this way, for it helps to make sense of the phenomena we will explore. Thinking of language as an instinct inverts the popular wisdom, especially as it has been passed down in the canon of the humanities and social sciences. Language is no more a cultural invention than is upright posture. It is not a manifestation of a general capacity to use symbols: a three-yearold, we shall see, is a grammatical genius, but is quite incompetent at the visual arts, religious iconography, traffic signs, and the other staples of the semiotics curriculum. Though language is a magnificent ability unique to Homo sapiens among living species, it does not call for sequestering the study of humans from the domain of biology, for a magnificent ability unique to a particular living species is far from unique in the animal kingdom. Some kinds of bats home in on flying insects using Doppler sonar. Some kinds of migratory birds navigate thousands of miles by calibrating the positions of the constellations against the time of day and year. In nature’s talent show, we are simply a species of primate with our own act, a knack for communicating information about who did what to whom by modulating the sounds we make when we exhale. Once you begin to look at language not as the ineffable essence of human uniqueness hut as a biological adaptation to communicate information, it is no longer as tempting to see language as an insidious shaper of thought, and, we shall see, it is not. Moreover, seeing language as one of nature’s engineering marvels — an organ with “that perfection of structure and co-adaptation which justly excites our admiration,” in Darwin’s words – gives us a new respect for your ordinary Joe and the much-maligned English language (or any language). The complexity of language, from the researcher’s point of view, is part of our biological birthright; it is not something that parents teach their children or something that must be elaborated in school — as Oscar Wilde said, “Education is an admirable thing, but it is well to remember from time to time that nothing that is worth knowing can be taught.” A preschooler’s tacit knowledge of grammar is more sophisticated than the thickest style manual or the most state-of-the-art computer language system, and the same applies to all healthy human beings, even the notorious syntax fracturing professional athlete and the, you know, like, inarticulate teenage skateboarder. Finally, since language is the product of a well engineered biological instinct, we shall see that it is not the nutty barrel of monkeys that entertainer columnists make it out to be.

 

Q. 1 According to the passage, all of the following stem from popular wisdom on language Except?

A. Language is a cultural artifact.

B. Language is a cultural invention.

C. Language is learnt as we grow.

D. Language is a psychological faculty.

 

Q. 2 Which of the following can be used as parallel reasoning for the “spiders know how to spin webs” analogy as used by the author?

A. A kitten learning to jump over a wall

B. Bees collecting nectar

C. A donkey carrying a load

D. A horse running a Derby

 

Q. 3 According to the passage, which of the following is unique to human beings?

A. Ability to use symbols while communicating with one another.

B. Ability to communicate with each other through voice modulation.

C. Ability to communicate information to other members of the species.

D. Ability to use sound as means of communication.

 

Q. 4 According to the passage, complexity of language cannot be taught by parents or at school to children because

A. children instinctively know language.

B. children learn the language on their own.

C. language is not amenable to teaching.

D. children know language better than their teachers or parents.

 

Q. 5 Which of the following best summarizes the passage?

A. Language is unique to Homo sapiens.

B. Language is neither learnt nor taught.

C. Language is not a cultural invention or artifact as it is made out.

D. Language is instinctive ability of human beings.

 

Q. 6 Why author has referred to ‘preschooler’s tacit knowledge of grammar’

A. To prove that Language is unique to Homo sapiens.

B. Used as an analogy for healthy human beings

C. To prove his point that language is not a cultural invention or artifact as it is made out.

D. To compare children instinctively know language.

 

Questions: 7 – 12

If American policy towards Europe in the postwar years had been a conspicuous success, and towards Asia a disappointing balance between success and failure, it could be said that the most conspicuous thing about relations with Latin America was the absence of any policy. Franklin Roosevelt, to be sure, had launched a “Good Neighbour” policy, but being a good neighbour was, it seemed, a negative rather than a positive affair, a matter of keeping hands off, of making the Monroe Doctrine, in form at least, multilateral. All through the postwar years, the states of Latin America – – Mexico and Chile were partial exceptions – – were in the throes of major economic and social crises. Population was growing faster than in any other part of the globe, without a comparable increase in wealth or productivity; the gap between the poor and the rich was widening; and as the rich and powerful turned to the military for the preservation of order and privilege, the poor turned to revolution. Deeply involved in other quarters of the globe, the United States paid little attention to the fortunes or misfortunes of her neighbours to the south, and when she did intervene, it appeared to be on the side of order and the status quo rather than on the side of reform. So frightened was the United States of “Communism” in Latin America that it preferred military dictatorship to reformers who might drift too far to the “left”, and sustained a Batista in Cuba, a Trujillo in the Dominican Republic, a Peron in Argentina, and a Jimenez in Venezuela. In his last two years, President Eisenhower had tried to mend his Latin American fences. Though rejecting a Brazilian proposal of a Marshall Plan for Latin America, he did take the initiative in setting up an Inter-American development Bank with a capital of one billion dollars, almost half of it supplied by the United States. Other government investments in Latin America ran to some four million dollars, while private investments exceeded nine billion. Yet though to most Americans, all this seemed a form of economic aid, many Latin Americans regarded it as economic imperialism. In September 1960, came a co-operative plan that could not be regarded as other than enlightened: the Act of Bogota, which authorized a grant of half a billion dollars to subsidize not only economic but social and educational progress in Latin America. “We are not saints”, said President Eisenhower when he visited Santiago de Chile, “We know we make mistakes, but our heart is in the right place”. But was it? President Kennedy was confronted by the same dilemma that had perplexed his predecessors. Clearly it was essential to provide a large-scale aid to the countries south of Rio Grande, but should this aid go to bolster up established regimes and thus help maintain status quo, or should it be used to speed up social reforms, even at the risk of revolt? As early as 1958, the then Senator Kennedy had asserted that “the objective of our aid program in Latin America should not be to purchase allies, but to consolidate a free and democratic Western Hemisphere, alleviating those conditions which might foster opportunities for communistic infiltration and uniting our peoples on the basis of constantly increasing living standards”. This conviction that raising the standards of living was the best method of checking Communism now inspired President Kennedy’s bold proposal for the creation of the alliance for progress – – a ten year plan designed to do for Latin America what Marshall Plan had done for Western Europe. It was to be “a peaceful revolution on a hemispheric scale, a vast cooperative effort, unparalleled in magnitude and nobility of purpose, to satisfy the basic needs of the American people for homes, work, land, health and schools. “To achieve this, the United States pleaded an initial grant of one billion dollars, with the promise of additional billions for the future.

 

Q. 7 Following World War II, which problem was the United States most concerned with regarding Latin America?

A. Economic stability.

B. Political ideology

C. Religious persecution.

D. Military dictatorship.

 

Q. 8 A key reason why Latin American rejected the Inter-American development Bank was that 

A. it primarily provided money for social reform subsidies.

B. the moneys provided were only for specific performance projects.

C. it constituted an extension of the Marshall Plan into Latin America

D. it was being used as a means to control the economic destiny of Latin America.

 

Q. 9 Which of the following is most closely associated with the concept of a Marshall Plan for Latin America?

A. The Good Neighbour Policy.

B. The Alliance for Progress.

C. The Act of Bogota.

D. The Monroe Doctrine.

 

Q. 10 According to the passage, the fundamental change in U.S. foreign policy directed towards Latin America

A. resulted in a deterioration of U.S. Latin American relations.

B. was responsible for Peron remaining as a dictator in Peru.

C. recognized that economic aid alone would prevent social revolutions.

D. provided for increased military and economic aid to prevent the spread of communism in Latin America.

 

Q. 11 All of the following statements are true, except?

A. Mexico and Chile did not experience the general social crises that are common to the majority of Latin American countries.

B. President Eisenhower continued in practice the theory that economic aid was the best defense against communist incursion into Latin America

C. The Good Neighbour Policy favoured a multilateral interpretation of the Monroe Doctrine.

D. The traditional U.S. approach in Latin America was to protect the status quo.

 

Q. 12 Which of the inferences can be drawn if everything said in the passage were assumed to be true?

A. Rebellions are fuelled by social reforms and avoided by supporting established authorities or continuing the present state of affairs.

B. The American policy towards Asia can be called an overall success, though small in magnitude.

C. Kennedy, in 1958, wanted America to aid South American countries to acquire more support in their fight against communism

D. Eisenhower rejected the Marshall Plan, whereas Kennedy implemented a similar one.

 

Questions: 13 – 18

Last fortnight, news of a significant development was tucked away in the inside pages of newspapers. The government finally tabled a bill in Parliament seeking to make primary education a fundamental right. A fortnight earlier, a Delhi-based newspaper had carried a report about a three month interruption in the Delhi Government’s ‘Education for All’ programme. The report made for distressing reading. It said that literacy centres across the city were closed down, volunteers beaten up and enrollment registers burnt. All because the state government had, earlier this year, made participation in the programme mandatory for teachers in government schools. The routine denials were issued and there probably was a wee bit of exaggeration in the report. But it still is a pointer to the enormity of the task at hand. That economic development will be inherently unstable unless it is built on a solid base of education, specially primary education, has been said so often that it is in danger of becoming a platitude. Nor does India’s abysmal record in the field need much reiteration. Nearly 30 million children in the six to ten age group do not go to school — reason enough to make primary education not only compulsory but a fundamental right. But is that the solution? More importantly, will it work? Or will it remain a mere token, like the laws providing for compulsory primary education? It is now widely known that 14 states and four Union Territories have this law on their statute books. Believe it or not, the list actually includes Bihar, Madhya Pradesh (MP) and Rajasthan, where literacy and education levels are miles below the national average. A number of states have not even notified the compulsory education law. This is not to belittle the decision to make education a fundamental right. As a statement of political will, a commitment by the decision-makers, its importance cannot be undervalued. Once this commitment is clear, a lot of other things like resource allocation will naturally fall into place. But the task of universalizing elementary education (UEE) is complicated by various socio-economic and cultural factors which vary from region to region and within regions. If India’s record continues to appall, it is because these intricacies have not been adequately understood by the planners and administrators. The trouble has been that education policy has been designed by grizzled mandarins ensconced in Delhi and is totally out of touch with the ground reality. The key then is to decentralise education planning and implementation. What’s also needed is greater community involvement in the whole process. Only then can school timings be adjusted for convenience, school children given a curriculum they can relate to and teachers made accountable. For proof, one has only to look at the success of the district primary education programme, which was launched in 1994. It has met with a fair degree of success in the 122 districts it covers. Here the village community is involved in all aspects of education — allocating finances to supervising teachers to fixing school timings and developing curriculum and textbooks — through district planning teams. Teachers are also involved in the planning and implementation process and are given small grants to develop teaching and learning material, vastly improving motivational levels. The consequent improvement in the quality of education generates increased demand for education. But for this demand to be generated, quality will first have to be improved. In MP, the village panchayats are responsible for not only constructing and maintaining primary schools but also managing scholarships, besides organising non-formal education. How well this works in practice remains to be seen (though the department claims the schemes are working very well) but the decision to empower panchayats with such powers is itself a significant development. Unfortunately, the Panchayat Raj Act has not been notified in many states. After all, delegating powers to the panchayats is not looked upon too kindly by vested interests. More specifically, by politicians, since decentralization of education administration takes away from them the power of transfer, which they use to grant favors and build up a support base. But if the political leadership can push through the bill to make education a fundamental right, it should also be able to persuade the states to implement the laws on Panchayat Raj. For, UEE cannot be achieved without decentralization. Of course, this will have to be accompanied by proper supervision and adequate training of those involved in the administration of education. But the devolution of powers to the local bodies has to come first. 

 

Q. 13 None of these problems are plaguing the education system in India Except

A. poverty.

B. diverse cultural and socio-economic factors.

C. male chauvinism.

D. Celebrating festivals

 

Q. 14 In the context of the passage, the term ‘grizzled mandarins’ means.

A. old hags.

B. decrepit men.

C. ineffective old men.

D. none of these

 

Q. 15 One of the reasons contributing to India’s poor performance on the education front is that 

A. its leaders do not have the conviction required to improve the education system.

B. male members of society do not want their female counterparts to be educated.

C. administrators in charge of education are out of touch with ground realities.

D. the country does not have the law for implementation of education policies in its statute books.

 

Q. 16 The only way in which the education system can be improved is by

A. decentralizing education planning and implementation.

B. introducing fresh blood in the planning body.

C. injecting funds into the exchequer solely for the purpose.

D. educating the people on the need for primary education.

 

Q. 17 Primary education

A. is a fundamental right.

B. will be made a fundamental right.

C. is only for the privileged sections of society.

D. none of these

 

Q. 18 Education policy cannot be successfully implemented by any of the following Except?

A. greater community involvement.

B. greater community development.

C. greater community awareness.

D. Greater community experience

 

Questions: 19 – 24

A game of strategy, as currently conceived in game theory, is a situation in which two or more “players” make choices among available alternatives (moves). The totality of choices determines the outcomes of the game, and it is assumed that the rank order of preferences for the outcomes is different for different players. Thus the “interests” of the players are generally in conflict. Whether these interests are diametrically opposed or only partially opposed depends on the type of game. Psychologically, most interesting situations arise when the interests of the players are partly coincident and partly opposed, because then one can postulate not only a conflict among the players but also inner conflicts within the players. Each is torn between a tendency to cooperate, so as to promote the common interests, and a tendency to compete, so as to enhance his own individual interests. Internal conflicts are always psychologically interesting. What we vaguely call “interesting” psychology is in very great measure the psychology of inner conflict. Inner conflict is also held to be an important component of serious literature as distinguished from less serious genres. The classical tragedy, as well as the serious novel reveals the inner conflict of central figures. The superficial adventure story on the other hand, depicts only external conflict; that is, the threats to the person with whom the reader (or viewer) identifies stem in these stories exclusively from external obstacles and from the adversaries who create them. On the most primitive level this sort of external conflict is psychologically empty. In the fisticuffs between the protagonists of good and evil, no psychological problems are involved or, at any rate, none are depicted in juvenile representations of conflict. The detective story, the “adult” analogue of a juvenile adventure tale, has at times been described as a glorification of intellectualized conflict. However, a great deal of the interest in the plots of these stories is sustained by withholding the unraveling of a solution to a problem. The effort of solving the problem is in itself not a conflict if the adversary (the unknown criminal) remains passive, like Nature, whose secrets the scientist supposedly unravels by deduction. If the adversary actively puts obstacles in the detective’s path toward the solution, there is genuine conflict. But the conflict is psychologically interesting only to the extent that it contains irrational components such as a tactical error on the criminal’s part or the detective’s insight into some psychological quirk of the criminal or something of this sort. Conflict conducted in a perfectly rational manner is psychologically no more interesting than a standard Western. For example, Tic-tac-toe, played perfectly by both players, is completely devoid of psychological interest. Chess may be psychologically interesting but only to the extent that it is played not quite rationally. Played completely rationally, chess would not be different from Tic-tac-toe. In short, a pure conflict of interest (what is called a zero-sum game) although it offers a wealth of interesting conceptual problems, is not interesting psychologically, except to the extent that its conduct departs from rational norms.

 

Q. 19 Find the main idea of the passage?

A. To explain game of strategy

B. To explain detective’s path

C. To explain juvenile representations of conflict

D. none of these

 

Q. 20 According to the passage, internal conflicts are psychologically more interesting than external conflicts because

A. internal conflicts, rather than external conflicts, form an important component of serious literature as distinguished from less serious genres.

B. only juveniles or very few “adults” actually experience external conflict, while internal conflict is more widely prevalent in society.

C. in situations of internal conflict, individuals experience a dilemma in resolving their own preferences for different outcomes

D. there are no threats to the reader (or viewer) in case of external conflicts.

 

Q. 21 Which, according to the author, would qualify as interesting psychology?

A. A statistician’s dilemma over choosing the best method to solve an optimization problem.

B. A chess player’s predicament over adopting a defensive strategy against an aggressive opponent.

C. A mountaineer’s choice of the best path to Mt. Everest from the base camp.

D. A finance manager’s quandary over the best way of raising money from the market.

 

Q. 22 According to the passage, which of the following options about the application of game theory to a conflict-of-interest situation is true?

A. Assuming that the rank order of preferences for options is different for different players.

B. Accepting that the interests of different players are often in conflict.

C. Not assuming that the interests are in complete disagreement.

D. All of the above

 

Q. 23 The problem solving process of a scientist is different from that of a detective because

A. scientists study inanimate objects, while detectives deal with living criminals or law offenders.

B. scientists study known objects, while detectives have to deal with unknown criminals or law offenders.

C. scientists study phenomena that are not actively altered, while detectives deal with phenomena that have been deliberately influenced to mislead.

D. scientists study psychologically interesting phenomena, while detectives deal with “adult” analogues of juvenile adventure tales.

 

Q. 24 How Tic-tac-toe as a game is different from Chess?

A. Games like Tic-tac-toe is played in a perfectly rational manner is psychologically no more interesting than chess which is played not quite rationally.

B. Both games are same when it comes to pure conflict of interest.

C. Both games are played psychology of inner conflict

D. Tic-tac-toe is a juvenile adventure tale, while chess at times been described as a glorification of intellectualized conflict.

 

Question 25

Parents usually do not insist that their children learn to walk by a certain age. Parents feel confident that the children will learn to walk within a reasonable period of time, when their bodies are ready for such an undertaking. Teachers should adopt the same attitude when teaching children in school how to read. If teachers did this, children might learn to read much more quickly and experience less anxiety while doing so.

 

Q. 25 Which of the following best describes the Essence or structure of the passage?

A. How children learn one kind of activity is described and then this method is recommended for teaching children another kind of activity.

B. Two different views of how children should be taught to read are compared and contrasted.

C. A view of how children should be taught is described and then criticized.

D. Contrasting views of parents and teachers on how quickly children actually learn are described and then analyzed.

E. The amount of time it take for children to learn one kind of activity is described and then used to predict how long it will take them to learn another kind of activity.

 

Q. 26 The sentences given, when properly sequenced, form a coherent paragraph. Each sentence is labeled with a letter. Choose the most logical order of sentences from among the given choices to construct a coherent paragraph. Sentence 1 and 6 are fixed starting and ending of the paragraph.

1. Amount of published information available varies widely by industry.

A. Unfortunately for the researcher, many industries do not meet these criteria, and there may be little published information available.

B. Generally, the problem the researcher will face in using published data for analyzing an economically meaningful industry is that they are too broad or too arranged to fit the industry.

C. However, it is always possible to gain some important information about an industry from published sources and these sources should be aggressively pursued.

D. Larger the industry, the older it is, and the slower the rate of technological change,

better is the available published information.

6. If a researcher starts a searching for data with this reality in mind, the uselessness of broad data will be better recognized and the tendency to give up will be avoided.

A. ACDB

B. BDAC

C. BCDA

D. DACB

Question 27

Trade protectionism, disguised as concern for the climate, is raising its head. Citing competitiveness concerns, powerful industrialized countries are holding out threats of a levy on imports of energy intensive products from developing countries that refuse to accept their demands. The actual source of protectionist sentiment in the OECD countries is, of course, their current lackluster economic performance, combined with the challenges posed by the rapid economic rise of China and India – in that order.

 

Q. 27 In the following question a paragraph from which the last sentence has been deleted. From the given options, choose the sentence that completes the paragraph in the most appropriate way. Type the correct option number in the given space below.

A. Climate change is evoked to bring trade protectionism through the back door.

B. OECD countries are taking refuge in climate change issues to erect trade barriers against these two countries.

C. Climate change concerns have come as a convenient stick to beat the rising trade power of China and India.

D. Defenders of the global economic status quo are posing as climate change champions.

E. Today’s climate change champions are the perpetrators of global economic inequity.

 

Question 28

Whatever their disadvantage with respect to distributing education tax dollars equally among school districts, in one respect at least, local property taxes are superior to state taxes as a means of funding public schools. Because local property taxes provide public schools with a direct source of revenue, these public schools are relatively free from competition for tax dollars with other government services. School administrators do not have to compete for a share of the state tax dollars, which are already being spent on health, criminal justice, public safety, and transportation. They are not placed in the position of having to argue that school programs must have priority over other public services financed by state taxes.

 

Q. 28 Which of the following statements best expresses essence of the passage?

A. Education is more important than other government services, such as criminal justice or public safety.

B. The disadvantage of local property taxes is that they do not equally distribute tax dollars spent on education.

C. School administrators are not accustomed to arguing that their programs must have priority over other government services.

D. Financing education with local property taxes has the advantage of eliminating competition for funding between schools and other public services.

E. School administrators must be prepared to justify the funds they spend on education programs.

 

Question 29

Most people at their first consultation take a furtive look at the surgeon’s hands in the hope of reassurance. Prospective patients look for delicacy, sensitivity, steadiness, perhaps unblemished pallor. On this basis, Henry Perowne loses a number of cases each year. Generally, he knows it’s about to happen before the patient does: the downward glance repeated, the prepared questions beginning to falter, the overemphatic thanks during the retreat to the door. 

 

Q. 29 In the following question a paragraph from which the last sentence has been deleted. From the given options, choose the sentence that completes the paragraph in the most appropriate way.

A. Other people do not communicate due to their poor observation.

B. Other patients don’t like what they see but are ignorant of their right to go elsewhere.

C. But Perowne himself is not concerned.’

D. But others will take their place, he thought.

E. These hands are steady enough, but they are large.

 

Question 30

The sentences given, when properly sequenced, form a coherent paragraph. Each sentence is labeled with a letter. Choose the most logical order of sentences from among the given choices to construct a coherent paragraph. Sentence 1 and 6 are fixed starting and ending of the paragraph. 

 

Q. 30 A. Intensity of competition in an industry is neither a matter of coincidence nor bad luck.

A. The collective strength of these forces determines the ultimate profit potential in the industry where profit potential is measured in terms of long run returns on invested capital.

B. Rather, competition in an industry is rooted in its underlying economic structure and goes well beyond the behavior of current competitors.

C. Not all industries have the same potential.

D. The state of competition in an industry depends on five basic competitive forces.

6. They differ fundamentally in their ultimate profit potential as the collective strength of the forces differs.

A. ADCB

B. BDCA

C. BDAC

D. BCAD

 

Question 31

Shakespeare wrote four types of plays: histories, comedies, tragedies, and tragicomedies. Some scholars contend that Shakespeare’s choice of three of these types of dramatic forms reflects his various psychological states. As a young man making a name for himself in London, he wrote comedies. Then, saddened by the death of this son, he turned to tragedies. Finally, seasoned by life’s joys and sorrows, he produced tragicomedies. But a look at the theater scene of his day reveals that Shakespeare was not so much writing out of his heart as into his pocketbook. When comedies were the vogue, he wrote comedies; when tragedies were the rage, he wrote tragedies; and when tragicomedies dominated the stage, he produced tragicomedies.

 

Q. 31 Which of the following statements best expresses essence of the passage?

A. Examine Shakespeare’s life in light of his dramatic works

B. Contest a theory that attempts to explain why Shakespeare wrote the kinds of plays he did

C. Explain the terms “comedy,” “tragedy,” and “tragicomedy” as they are used in discussions of Shakespeare’s plays

D. Compare Shakespeare’s plays with the works of other dramatists of his day

E. Discuss what is known about Shakespeare’s psychological states

 

Question 32

The sentences given, when properly sequenced, form a coherent paragraph. Each sentence is labeled with a letter. Choose the most logical order of sentences from among the given choices to construct a coherent paragraph.

Q. 32 A. A few months ago I went to Princeton University to see what the young people who are going to be running our country in a few decades are like.

B. I would go to sleep in my hotel room around midnight each night, and when I awoke, my mailbox would be full of replies—sent at 1:15 a.m., 2:59 a.m., 3:23 a.m.

C.One senior told me that she went to bed around two and woke up each morning at seven; she could afford that much rest because she had learned to supplement her full day of work by studying in her sleep.

D.Faculty members gave me the names of a few dozen articulate students, and I sent them e-mails, inviting them out to lunch or dinner in small groups.

E.As she was falling asleep she would recite a math problem or a paper topic to herself;

A. CEBAD

B. ADCBE

C. DACEB

D. ADBCE

 

Question 33

Mattan cherry is Indian Jewry’s most famous settlement. Its pretty streets of pastel colored houses, connected by first-floor passages and home to the last twelve saree-and-sarong-wearing, white skinned Indian Jews are visited by thousands of tourists each year. Its synagogue, built in 1568, with a floor of blueand- white Chinese tiles, a carpet given by Haile Selassie and the frosty Yaheh selling tickets at the door, stands as an image of religious tolerance. 

 

Q. 33 In the following question a paragraph from which the last sentence has been deleted. From the given options, choose the sentence that completes the paragraph in the most appropriate way.

A. Mattancherry represents, therefore, the perfect picture of peaceful co-existence.

B. India’s Jews have almost never suffered discrimination, except for European colonizers and each other.

C. Jews in India were always tolerant.

D. Religious tolerance has always been only a façade and nothing more.

E. The pretty pastel streets are, thus, very popular with the tourists.

 

Question 34

The sentences given, when properly sequenced, form a coherent paragraph. Each sentence is labeled with a letter. Choose the most logical order of sentences from among the given choices to construct a coherent paragraph.

 

Q. 34 A. Four days later, Infosist announced its own bid for Pep-Software, and invited the firm’s board to a discussion.

B. Furious that his own plans had been endangered, Pep-software’s boss, Chandan Shah, called Infosist’s offer “diabolical”, and its boss, Laxman, a “sociopath”.

C. In early June, Pep-software said that it would buy J.D.Soft, a smaller rival.

D. Moreover, said Mr. Shah, he “could imagine no price nor combination of price and other conditions to recommend accepting the offer.”

E. On June 12th, PepSoftware turned Infosist down.

A. CABED

B. CBADE

C. CABDE

D. ABEDC

 

Questions: 35 – 38

Answer the questions on the basis of the information given below.

Purana and Naya are two brands of kitchen mixer-grinders available in the local market. Purana is an old brand that was introduced in 1990, while Naya was introduced in 1997. For both these brands, 20% of the mixer-grinders bought in a particular year are disposed off as junk exactly two years later. It is known that 10 Purana mixer-grinders were disposed off in 1997. The following figures show the number of Purana and Naya mixer-grinders in operation from 1995 to 2000, as at the end of the year. 

 

Q. 35 How many Naya mixer-grinders were purchased in 1999?

A. 44

B. 50

C. 55

D. 64

 

Q. 36 How many Naya mixer-grinders were disposed off by the end of 2000?

A. 10

B. 16

C. 22

D. Cannot be determined from the data

 

Q. 37 How many Purana mixer-grinders were disposed off in 2000?

A. 0

B. 5

C. 6

D. Cannot be determined from the data

 

Q. 38 How many Purana mixer-grinders were purchased in 1999?

A. 20

B. 23

C. 50

D. Cannot be determined from the data

 

Questions: 39 – 42

Answer the questions on the basis of the information given below.

A study was conduced to ascertain the relative importance that employees in five different countries assigned to five different traits in their Chief Executive Officers. The traits were compassion (C), decisiveness (D), negotiation skills (N), public visibility (P), and vision (V). The level of dissimilarity between two countries is the maximum difference in the ranks allotted by the two countries to any of the five traits. The following table indicates the rank order of the five traits for each country.

 

Q. 39 Which of the following pairs of countries are most dissimilar?

A. China and Japan

B. India and China

C. Malaysia and Japan

D. Thailand and Japan

 

Q. 40 Which of the following countries is least dissimilar to India?

A. China

B. Japan

C. Malaysia

D. Thailand

 

Q. 41 Which amongst the following countries is most dissimilar to India?

A. China

B. Japan

C. Malaysia

D. Thailand

 

Q. 42 Three of the following four pairs of countries have identical levels of dissimilarity. Which pair is the odd one out?

A. Malaysia and China

B. China and Thailand

C. Thailand and Japan

D. Japan and Malaysia

 

Questions: 43 – 46

Answer the questions based on the table given below. The table below gives information about four different crops, their different quality, categories and the regions where they are cultivated. Based on the information given in the table answer the questions below.

 

Q. 43 How many regions produce medium qualities of Crop-1 or Crop-2 and also produce low quality of Crop-3 or Crop-4?

A. zero

B. one

C. two

D. three

 

Q. 44 Which of the following statements is true?

A. All medium quality Crop-2 producing regions are also high quality Crop-3 producing regions.

B. All high quality Crop-1 producing regions are also medium and low Crop-4 producing regions.

C. There are exactly four Crop-3 producing regions, which also produce Crop-4 but not Crop-2.

D. Some Crop-3 producing regions produce Crop-1, but not high quality Crop-2.

 

Q. 45 How many low quality Crop-1 producing regions are either high quality Crop-4 producing regions or medium quality Crop-3 producing regions?

A. one

B. two

C. three

D. zero

 

Q. 46 How many high quality Crop-1 producing regions are low quality Crop-3 producing regions? 

A. one

B. two

C. three

D. zero

 

Questions: 47 – 50 

Answer the following questions based on the information given below:

In a sports event, six teams (A, B, C, D, E and F) are competing against each other. Matches are scheduled in two stages. Each team plays three matches in stage – I and two matches in Stage – II. No team plays against the same team more than once in the event. No ties are permitted in any of the matches. The observations after the completion of Stage – I and Stage – II are as given below. 

Stage-I:

• One team won all the three matches.

• Two teams lost all the matches.

• D lost to A but won against C and F.

• E lost to B but won against C and F.

• B lost at least one match.

• F did not play against the top team of Stage-I.

Stage-II:

• The leader of Stage-I lost the next two matches.

• Of the two teams at the bottom after Stage-I, one team won both matches, while

the other lost both matches.

• One more team lost both matches in Stage-II.

 

Q. 47 The two teams that defeated the leader of Stage-I are:

A. F & D

B. E & F

C. B & D

D. E & D

E. F & D

 

Q. 48 The only team(s) that won both matches in Stage-II is (are):

A. B

B. E & F

C. A, E & F

D. B, E & F

E. B & F

 

Q. 49 The teams that won exactly two matches in the event are:

A. A, D & F

B. D & E

C. E & F

D. D, E & F

E. D & F

 

Q. 50 The team(s) with the most wins in the event is (are):

A. A

B. A & C

C. F

D. E

E. B & E

 

Questions: 51 – 54

Answer the questions on the basis of the information given below: 

Two traders, Harshit and Dhara, were involved in the buying and selling of MCS shares over five trading days. At the beginning of the first day, the MCS share was priced at Rs 100, while at the end of the fifth day it was priced at Rs 110. At the end of each day, the MCS share price either went up by Rs 10, or else, it came down by Rs 10. Both Harshit and Dhara took buying and selling decisions at the end of each trading day. The beginning price of MCS share on a given day was the same as the ending price of the previous day. Harshit and Dhara started with the same number of shares and amount of cash, and had enough of bototh. Below are some additional facts about how Harshit and Dhara traded over the five trading days. Each day if the price went up, Harshit sold 10 shares of MCS at the closing price. On the other hand, each day if the price went down, he bought 10 shares at the closing price. 2 If on any day, the closing price was above Rs 110, then Dhara sold 10 shares of MCS, while if it was below Rs 90, he bought 10 shares, all at the closing price.

 

Q. 51 If Harshit sold 10 shares of MCS on three consecutive days, while Dhara sold 10 shares only once during the five days, what was the price of MCS at the end of day 3?

A. Rs 90

B. Rs 100

C. Rs 110

D. Rs 120

E. Rs 130

 

Q. 52 If Harshit ended up with Rs 1300 more cash than Dhara at the end of day 5, what was the price of MCS share at the end of day 4?

A. Rs 90

B. Rs 100

C. Rs 110

D. Rs 120

E. Not uniquely determinable

 

Q. 53 If Dhara ended up with 20 more shares than Harshit at the end of day 5, what was the price of the share at the end of day 3?

A. Rs 90

B. Rs 100

C. Rs 110

D. Rs 120

E. Rs 130

 

Q. 54 What could have been the maximum possible increase in combined cash balance of Harshit and Dhara at the end of the fifth day?

A. Rs 3700

B. Rs 4000

C. Rs 4700

D. Rs 5000

E. Rs 6000

 

Questions: 55 – 58

Answer the questions on the basis of the information given below:

Mathematicians are assigned a number called Zohos number (named after the famous mathematician, Paul Zohos). Only Paul Zohos himself has an Zohos number of zero. Any mathematician who has written a research paper with Zohos has an Zohos number of 1. For other mathematicians, the calculation of his/ her Zohos number is illustrated below: Suppose that a mathematician X has co-authored papers with several other mathematicians. From among them, mathematician Y has the smallest Zohos number. Let the Zohos number of Y be y. Then X has an Zohos number of y+1. Hence any mathematician with no co-authorship chain connected to Zohos has an Zohos number of infinity. In a seven day long mini-conference organized in memory of Paul Zohos, a close group of eight mathematicians, call them A, B, C, D, E, F, G and H, discussed some research problems. At the beginning of the conference, A was the only participant who had an infinite Zohos number. Nobody had an Zohos number less than that of F.

1 On the third day of the conference F co-authored a paper jointly with A and C.

This reduced the average Zohos number of the group of eight mathematicians to

3. The Zohos numbers of B, D, E, G and H remained unchanged with the writing of

this paper. Further, no other co-authorship among any three members would have reduced the average Zohos number of the group of eight to as low as 3.

2 At the end of the third day, five members of this group had identical Zohos numbers while the other three had Zohos numbers distinct from each other.

3 On the fifth day, E co-authored a paper with F which reduced the group’s average Zohos number by 0.5. The Zohos numbers of the remaining six were unchanged with the writing of this paper.

4 No other paper was written during the conference.

 

Q. 55 How many participants in the conference did not change their Zohos number during the conference?

 

Q. 56 The person having the largest Zohos number at the end of the conference must have had Zohos number (at that time):

 

Q. 57 How many participants had the same Zohos number at the beginning of the conference? 

 

Q. 58 The Zohos number of C at the end of the conference was:

 

Questions: 59 – 62

Answer the questions on the basis of the information given below.

Coach John sat with the score cards of Indian players from the 3 games in a oneday cricket tournament where the same set of players played for India and all the major batsmen got out. John summarized the batting performance through a table, one column for each game. In each column, the three entries communicate the number of runs scored by the three top scores from India. The last row in the table denotes the percentage of the total score that was scored by the top three Indian scorers in that game. No two players score the same number of runs in the same game. John also calculated two batting indices for each player based on his scores in the tournaments; the R-index of a batsman is the difference between his highest and lowest scores in the 3 games while the M-index is the middle number, if his scores are arranged in a non-increasing order.

 

Q. 59 For how many Indian players is it possible to calculate the exact M-index?

A. 0

B. 1

C. 2

D. More than 2

 

Q. 60 Among the players mentioned, who can have the lowest R-index from the tournament? 

A. Only Kaif, Rahul or Yuvraj

B. Only Kaif or Rahul

C. Only Kaif or Yuvraj

D. Only Kaif

 

Q. 61 How many players among those listed definitely scored less than Yuvraj in the tournament?

A. 0

B. 1

C. 2

D. More than 2

 

Q. 62 Which of the players had the best M-index from the tournament?

A. Rahul

B. Saurav

C. Virender

D. Yuvraj

 

Questions: 63 – 66

Answer the following questions based on the information given below:

For admission to various affiliated colleges, a university conducts a written test with four different sections, each with a maximum of 50 marks. The following table gives the aggregate as well as the sectional cut-off marks fixed by six different colleges affiliated to the university. A student will get admission only if he/she gets marks greater than or equal to the cut-off marks in each of the sections and his/her aggregate marks are at least equal to the aggregate cut-off marks as specified by the college.

 

Q. 63 Ramaya got calls from all colleges. What could be the minimum aggregate marks obtained by her?

 

Q. 64 Gauri got calls from two colleges. What could be the minimum marks obtained by him in a section?

 

Q. 65 Minakshi did not get a call from even a single college. What could be the maximum aggregate marks obtained by him?

 

Q. 66 What is the maximum score required by a Cetking student in Section D so that student clear all colleges cut-off?

 

Q. 67 A person who has a certain amount with him goes to market. He can buy 50 oranges or 40 mangoes. He retains 10% of the amount for taxi fares and buys 20 mangoes and of the balance he purchases oranges. Number of oranges he can purchase is 

A. 36

B. 40

C. 15

D. 20

 

Q. 68 2/5 of the voters promise to vote for P and the rest promised to vote for Q. Of these, on the last day 15% of the voters went back of their promise to vote for P and 25% of voters went back of their promise to vote for Q, and P lost by 2 votes. Then the total number of voters is 

 

Q. 69 A stockist wants to make some profit by selling sugar. He contemplates about various methods . Which of the following would maximize his profit?

I. Sell sugar at 10% profit.

II. Use 900 g of weight instead of 1 kg.

III. Mix 10% impurities in sugar and selling sugar at cost price.

IV. Increase the price by 5% and reduce weights by 5%.

A. I or III

B. II

C. II, III and IV

D. profits are same

 

Q. 70 For the product n(n + 1)(2n + 1), n ∈ N, which one of the following is not necessarily true? 

A. It is even

B. Divisible by 3

C. Divisible by the sum of the square of first n natural numbers

D. Never divisible by 237

 

Q. 71 ABCD is a square of area 4 with diagonals AC and BD, dividing square into 4 congruent triangles. Figure looks like four non-over lapping triangles. Then the sum of the perimeters of the triangles is

A. 8(2 + √2)

B. 8(1 + √2)

C. 4(1 + √2)

D. 4(2 + √2)

 

Q. 72 PQRS is a square. SR is a tangent (at point S) to the circle with centre O and TR = OS. Then the ratio of area of the circle to the area of the square is

A. π/3

B. 11/7

C. 3/π

D. 7/11

 

Q. 73 From a circular sheet of paper with a radius 20 cm, four circles of radius 5 cm each are cut out. What is the ratio of the uncut to the cut portion?

A. 1 : 3

B. 4 : 1

C. 3 : 1

D. 4 : 3

 

Q. 74 A wooden box (open at the top) of thickness 0.5 cm, length 21 cm, width 11 cm and height 6 cm is painted on the inside. The expenses of painting are Rs. 70. What is the rate of painting per square centimetres?

A. Re 0.7

B. Re 0.5

C. Re 0.1

D. Re 0.2

 

Q. 75 The cost of diamond varies directly as the square of its weight. Once, this diamond broke into four pieces with weights in the ratio 1 : 2 : 3 : 4. When the pieces were sold, the merchant got Rs. 70,000 less. Find the original price of the diamond.

 

Q. 76 The figure shows the rectangle ABCD with a semicircle and a circle inscribed inside in it as shown. What is the ratio of the area of the circle to that of the semicircle?

A. (√2 – 1)² : 1

B. 2(√2 – 1)² : 1

C. (√2 – 1)² : 2

D. none of these

 

Q. 77 In a mile race, Akshay can be given a start of 128 m by Bhairav. If Bhairav can give Chinmay a start of 4 m in a 100 m dash, then who out of Akshay and Chinmay will win a race of one and half miles, and what will be the final lead given by the winner to the loser? (One mile is 1,600 m.)

A. Akshay, 1/12 mile

B. Chinmay, 1/32 mile

C. Akshay, 1/24 mile

D. Chinmay, 1/16 mile

 

Q. 78 Two liquids A and B are in the ratio 5 : 1 in container 1 and 1 : 3 in container 2. In what ratio should the contents of the two containers be mixed so as to obtain a mixture of A and B in the ratio 1 : 1?

A. 2 : 3

B. 4 : 3

C. 3 : 2

D. 3 : 4

 

Q. 79 if x² + y² = 0.1 and |x – y| = 0.2, then |x| + |y| is equal to

A. 0.3

B. 0.4

C. 0.2

D. 0.6

 

Q. 80 ABCD is a rhombus with the diagonals AC and BD intersecting at the origin on the x-y plane. The equation of the straight line AD is x + y = 1. What is the equation of BC? 

A. x + y = –1

B. x – y = –1

C. x + y = 1

D. none of these

 

Q. 81 The set of all positive integers is the union of two disjoint subsets: {f(1), f(2), …, f(n), …} and {g(1), g(2), …, g(n), …}, where f(1) < f(2) < … < f(n) …, and g(1) < g(2) < … < g(n) …, and g(n) = f(f(n)) + 1 for all n ≥ 1. What is the value of g(1)?

A. 0

B. 2

C. 1

D. Cannot be determined

 

Q. 82 For all non-negative integers x and y, f(x, y) is defined as below.

f(0, y) = y + 1 f(x + 1, 0) = f(x, 1) f(x + 1, y + 1) = f(x, f(x + 1, y)).

Then what is the value of f(1, 2)?

 

Q. 83 In the figure above, AB = BC = CD = DE = EF = FG = GA. Then ∠DAE is approximately.

A. 15°

B. 20°

C. 30°

D. 25°

 

Q. 84 A water tank has three taps A, B, and C. A fills four buckets in 24 mins, B fills 8 buckets in 1 hour and C fills 2 buckets in 20 minutes. If all the taps are opened together a full tank is emptied in 2 hours. If a bucket can hold 5 litres of water, what is the capacity of the tank in litres?

 

Q. 85 Shyam went from Delhi to Shimla via Chandigarh by car. The distance from Delhi to Chandigarh is ¾ times the distance from Chandigarh to Shimla. The average speed from Delhi to Chandigarh was half as much again as that from Chandigarh to Shimla. If the average speed for the entire journey was 49 kmph. What was the average speed from Chandigarh to Shimla?

A. 39.2 kmph

B. 63 kmph

C. 42 kmph

D. none of these

 

Q. 86 Fourth term of an arithmetic progression is 8. What is the sum of the first 7 terms of the arithmetic progression?

A. 7

B. 64

C. 56

D. Cannot be determined

 

Q. 87 Two towns A and B are 100 km apart. A school is to be built for 100 students of town B and 30 students of Town A. Expenditure on transport is Rs. 1.20 per km per student. If the total expenditure on transport by all 130 students is to be as small as possible, then the school should be built at

A. 33 km from Town A.

B. 33 km from Town B

C. Town A

D. Town B

 

Q. 88 One man can do as much work in one day as a woman can do in 2 days. A child does one third the work in a day as a woman. If an estate-owner hires 39 pairs of hands, men, women and children in the ratio 6 : 5 : 2 and pays them in all Rs. 1113 at the end of the days work. What must the daily wages of a child be, if the wages are proportional to the amount of work done?

A. Rs.14

B. Rs.5

C. Rs.20

D. Rs.7

 

Q. 89 Then u₁₀ nearest to

 

Q. 90 Let x < 0.50, 0 < y < 1, z > 1. Given a set of numbers, the middle number, when they are arranged in ascending order, is called the median. So the median of the numbers x, y, and z would be

A. less than one

B. between 0 and 1

C. greater than 1

D. cannot say

 

Q. 91 The maximum possible value of y = min (1/2 – 3x² /4, 5x² /4) for the range 0 < x < 1 is 

A. 1/3

B. 1/2

C. 5/27

D. 5/16

 

Q. 92 Let x < 0, 0 < y < 1, z > 1. Which of the following may be false?

A. (x² – z² ) has to be positive.

B. yz can be less than one.

C. xy can never be zero

D. (y² – z² ) is always negative

 

Q. 93 A young girl counted in the following way on the fingers of her left hand. She started calling the thumb 1, the index finger 2, middle finger 3, ring finger 4, little finger 5, then reversed direction, calling the ring finger 6, middle finger 7, index finger 8, thumb 9, then back to the index finger for 10, middle finger for 11, and so on. She counted up to 1994. She ended on her.

A. thumb

B. index finger

C. middle finger

D. ring finger

 

Q. 94 Consider the set S = {1, 2, 3, . . ., 1000}. How many arithmetic progressions can be formed from the elements of S that start with 1 and end with 1000 and have at least 3 elements? 

A. 3

B. 4

C. 6

D. 7

 

Q. 95 The number of solutions of the equation 2x + y = 40 where both x and y are positive integers and x ≤ y is:

 

Q. 96 If logᵧx = (a . logzᵧ) = (b . logₓz) = ab, then which of the following pairs of values for (a, b) is not possible?

A. (-2, 1/2)

B. (1, 1)

C. (π, 1/π)

D. (2, 2)

 

Q. 97 When you reverse the digits of the number 13, the number increases by 18. How many other two digit numbers increase by 18 when their digits are reversed?

 

Q. 98 Survey was conducted of 100 people to find out whether they had read recent issues of Golmal, a monthly magazine. The summarized information regarding readership in 3 months is given below: Only September: 18; September but not August: 23; September and July: 8; September: 28; July: 48; July and August: 10; None of the three months: 24. What is the number of surveyed people who have read exactly two consecutive issues (out of the three)?

A. 7

B. 9

C. 12

D. 14

 

Q. 99 Amol was asked to calculate the arithmetic mean of 10 positive integers, each of which had 2 digits. By mistake, he interchanged the 2 digits, say a and b, in one of these 10 integers. As a result, his answer for the arithmetic mean was 1.8 more than what it should have been. Then b – a equals 1

 

Q. 100 The angle of elevation of the top of a tower 30 m high, from two points on the level ground on its opposite sides are 45 degrees and 60 degrees. What is the distance between the two points?

A. 30

B. 51.96

C. 47.32

D. 81.96

 

 

Answer Sheet 
Question 1 2 3 4 5 6 7 8 9 10
Answer D B B A D B B D A C
Question 11 12 13 14 15 16 17 18 19 20
Answer B A B D C A D A D C
Question 21 22 23 24 25 26 27 28 29 30
Answer B B C A A B D D B C
Question 31 32 33 34 35 36 37 38 39 40
Answer B D B C B C D A D A
Question 41 42 43 44 45 46 47 48 49 50
Answer B D B D C B B D E E
Question 51 52 53 54 55 56 57 58 59 60
Answer C B A D 5 7 3 2 C A
Question 61 62 63 64 65 66 67 68 69 70
Answer B B 181 25 50 46 D 100 B D
Question 71 72 73 74 75 76 77 78 79 80
Answer B A C C 1,00,000 D D D A C
Question 81 82 83 84 85 86 87 88 89 90
Answer B 4 D 240 C C D D 1023 B
Question 91 92 93 94 95 96 97 98 99 100
Answer D A B D 13 D 6 B 2 C

CAT Previous Year Paper 2014

CAT 2014

Section

Questions

Marks

Quantitative Aptitude

40 Questions

120

DILR

28 Questions

84

English

32 Questions

96

 

Q. 1 If ABCD is a square and BCE is an equilateral triangle, what is the measure of ∠DEC?

A. 15°

B. 30°

C. 20°

D. 45°

 

Q. 2 Instead of a meter scale, a cloth merchant uses a 120 cm scale while buying, but uses an 80 cm scale while selling the same cloth. What is his overall profit percentage?

A. 50%

B. 25%

C. 40%

D. 15%

 

Q. 3 From a circular sheet of paper with a radius 20 cm, four circles of radius 5 cm each are cut out. What is the ratio of the uncut to the cut portion?

A. 1 : 3

B. 4 : 1

C. 3 : 1

D. 4 : 3

 

Q. 4 The cost of diamond varies directly as the square of its weight. Once, this diamond broke into four pieces with weights in the ratio 1 : 2 : 3 : 4. When the pieces were sold, the merchant got Rs. 70,000 less. Find the original price of the diamond.

A. Rs. 1.4 lakh

B. Rs. 2 lakh

C. Rs. 1 lakh

D. Rs. 2.1 lakh

 

Q. 5 The question is followed by two statements, I and II.

If x, y and z are real numbers, is z – x even or odd?

I. xyz is odd.

II. xy + yz + zx is even.

Mark the answer as.

A. if the question can be answered with the help of statement I alone.

B. if the question can be answered with the help of statement II, alone.

C. if both statement I and statement II are needed to answer the question.

D. if the question cannot be answered even with the help of both the statements.

 

Questions: 6 – 9

Answer the questions based on following data.

The pages of a book are numbered 0, 1, 2 … upto M, M>0. There are four categories of instructions that direct a person in positioning the book at a page. The instruction types and their meanings are :

1. NEW : Position the book at page No. 1

2. END : Position the book at page No. 0

3. ONWARD, n :From the current page move forward by n pages; if, in this process, page number M is reached, stop at M.

4. REGRESS, n : From the current page, move backward by n pages; if in this process, page number 0 is reached, stop at page number 0.

In each of the following questions, you will find a sequence of instructions formed from the above categories. In each case, let n1 be the page number before the instructions are executed and n2 be the page number at which the book is positioned after the instructions are executed.

 

Q. 6 ONWARD, 25 ; REGRESS, 10. which of the following statements is true?

A. n1=n2 if M=10 and n1=0

B. M=20 provided n1>0

C. n1>30 provided M=900

D. n1=37 provided M=25

 

Q. 7 REGRESS, 5; ONWARD, 5. Which of the following statements is true about the above set of instructions?

A. n1=n2 provided n1 ≥ 5

B. n1=n2 provided n1>0

C. n2=5 provided M>0

D. n1>n2 provided M>0

 

Q. 8 ONWARD, 10; ONWARD, 10. Which of the following statements about the above instructions is true?

A. n2-n1=20 only if n1=0

B. n2-n1=20 if M>20 and n1=1

C. n2-n1=10 if M=21 and n1=0

D. n2>n1 if M>0

 

Q. 9 ONWARD, 5; REGRESS, 4. Which of the following statements about the above instructions is true?

A. n2=n1+4 Provided 1<n1<7

B. n2=n1 provided M<6

C. n2=n1+1 provided M-n1>5

D. n2-n1< 0 provided M>0

 

Q. 10 A circle is inscribed in a given square and another circle is circumscribed about the square. What is the ratio of the area of the inscribed circle to that of the circumscribed circle?

A. 2 : 3

B. 3 : 4

C. 1 : 4

D. 1 : 2

 

Q. 11 If y = f(x) and f(x) = (1-x) / (1 + x), which of the following is true?

A. f(2x) = f(x) – 1

B. x = f(2y)-1

C. f(1/x) = f(x)

D. x = f(y)

 

Questions: 12 – 15

Directions for questions: Answer the questions on the basis of the information given below.

The HR Manager of the IT company recently scanned employees training results of various exams into the central computer system. When their character reading software cannot read something, it leaves the space blank. The scanner output reads as follows:

In the grading system, A, B, C, D, and F grades fetch 6, 4, 3, 2, and 0 grade points respectively. The Grade Point Average (GPA) is the arithmetic mean of the grade points obtained in the five subjects. For example Navdeep’s GPA is (6 + 2 + 4 + 6 + 0) / 5 = 3.6. Some additional facts are also known about the students’ grades. These are Vipul obtained the same grade in C Language as Amanpreet obtained in Java and

Analysis. Fazal obtained the same grade in Analysis as Utkarsh did in C Language.

Tara received the same grade in exactly three courses.

 

Q. 12 What grade did Preeti obtain in Testing?

A. A

B. B

C. C

D. D

 

Q. 13 In Project Management, Tara could have received the same grade as

A. Ismet

B. Hari

C. Jagdeep

D. Manab

 

Q. 14 In Analysis, Gowri’s grade point was higher than that obtained by

A. Fazal

B. Hari

C. Navdeep

D. Rahul

 

Q. 15 What grade did Utkarsh obtain in Java?

A. B

B. C

C. D

D. F

 

Questions: 16 – 18

Answer the questions based on following data.

A dealer deals only in colour TVs and VCRs. He wants to spend up to Rs.12 lakhs to buy 100 pieces. He can purchase a colour TV at Rs.10,000 and a VCR at Rs.15,000. He can sell a colour TV at Rs.12,000 and a VCR at Rs.17,500. His objective is to maximize profits. Assume that he can sell all the items that he stocks.

 

Q. 16 For the maximum profit, the number of colour TVs and VCRs that he should respectively stock are

A. 80, 20

B. 20, 80

C. 60, 40

D. None of these

 

Q. 17 If the dealer would have managed to get an additional space to stock 20 more items, then for maximizing profit, the ratio of number of VCRs and number of TVs that he should stock is

A. 7 : 3

B. 0

C. 1 : 2

D. None of these

 

Q. 18 The maximum profit, in rupees lakh, the dealer can earn from his original stock if he can sell a colour TV at Rs. 12200 and VCR at Rs.18300 is

A. 2.64

B. 2.49

C. 2.72

D. 2.87

 

Questions: 19 – 22

Directions for questions: Answer the questions on the basis of the information given below.

The data points in the figure below represent monthly income and expenditure data of individual members following families. For these questions, savings is defined as

Savings = Income – Expenditure

 

Q. 19 Which family has the lowest average income?

A. Abhijeet

B. Baljeet

C. Camaljeet

D. Damanjeet

 

Q. 20 Which family has the highest average expenditure?

A. Abhijeet

B. Baljeet

C. Camaljeet

D. Damanjeet

 

Q. 21 Which family has the lowest average savings?

A. Abhijeet

B. Baljeet

C. Camaljeet

D. Damanjeet

 

Q. 22 The highest amount of savings accrues to a member of which family?

A. Abhijeet

B. Baljeet

C. Camaljeet

D. Damanjeet

 

Q. 23 In a Tennis Open tournament 71 persons have signed up for elimination rounds. All players are to be paired up for the first round, but because 71 is an odd number one player gets a bye, which promotes him to the second round, without actually playing in the first round. The pairing continues on the next round, with a bye to any player left over. If the schedule is planned so that a minimum number of matches are required to determine the champion, the number of matches which must be played is

A. 71

B. 70

C. 69

D. 36

 

Q. 24 There are ten 50 paise coins placed on a table. Six of these show tails four show heads. A coin is chosen at random and flipped over (not tossed). This operation is performed seven times. One of the coins is then covered. Of the remaining nine coins, five show tails and four show heads. The covered coin shows

A. a head

B. a tail

C. more likely a head

D. more likely a tail

 

Q. 25 From each of the two given numbers, half the smaller number is subtracted. Of the resulting numbers the larger one is three times as large as the smaller. What is the ratio of the two numbers?

A. 2 : 1

B. 3 : 1

C. 3 : 2

D. None

 

Q. 26 Three identical cones with base radius r are placed on their bases so that each is touching the other two. The radius of the circle drawn through their vertices is

A. smaller than r.

B. equal to r

C. larger than r

D. depends on the height of the cones.

 

Q. 27 The line AB is 6 metres in length and is tangent to the inner one of the

two concentric circles at point C. It is known that the radii of the two circles are integers.

The radius of the outer circle is

A. 5 meters

B. 4 meters

C. 6 meters

D. 3 meters

 

Questions: 28 – 31

Answer the following questions based on the information given below:

The following table shows the break-up of actual costs incurred by a company in last five years (year 2012 to year 2016) to produce a particular product:

The production capacity of the company is 2000 units. The selling price for the year 2016 was Rs. 125 per unit. Some costs change almost in direct proportion to the change in volume of production, while others do not follow any obvious pattern of change with respect to the volume of production and hence are considered fixed. Using the information provided for the year 2016 as the basis for projecting the figures for the year 2017, answer the following questions.

 

Q. 28 What is the approximate cost per unit in rupees, if the company produces and sells 1400 units in the year 2017?

A. 104

B. 107

C. 110

D. 114

 

Q. 29 What is the minimum number of units that the company needs to produce and sell to avoid any loss?

A. 313

B. 350

C. 384

D. 747

 

Q. 30 If the company reduces the price by 5%, it can produce and sell as many units as it desires. How many units the company should produce to maximize its profit?

A. 1400

B. 1600

C. 1800

D. 2000 2000

 

Q. 31 Given that the company cannot sell more than 1700 units, and it will have to reduce the price by Rs.5 for all units, if it wants to sell more than 1400 units, what is the maximum profit, in rupees, that the company can earn?

A. 25400

B. 24400

C. 31400

D. 32900

 

Questions: 32 – 33

Use the following information for next two questions: A function f(x) is said to be even if f(-x)=f(x), and odd if f(-x) = -f(x). Thus, for example, the function given by f(x) = x2 is even, while the function given by f(x) = x3 is odd. Using this definition,

answer the following questions.

 

Q. 32 The function given by f(x) = |x|³

A. even

B. odd

C. neither

D. both

 

Q. 33 The sum of two odd functions

A. is always an even function

B. is always an odd function

C. is sometimes odd and sometimes even

D. may be neither odd nor even

 

Q. 34 A five digit number is formed using digits 1, 3, 5, 7 and 9 without repeating any one of them. What is the sum of all such possible numbers?

A. 6666600

B. 6666660

C. 6666666

D. None

 

Q. 35 A box contains 6 red balls, 7 green balls and 5 blue balls. Each ball is of a different size. The probability that the red ball selected is the smallest red ball, is

A. 1/18

B. 1/3

C. 1/6

D. 2/3

 

Q. 36 ABC forms an equilateral triangle in which B is 2 km from A. A person starts walking from B in a direction parallel to AC and stops when he reaches a point D directly east of C. He, then, reverses direction and walks till he reaches a point E directly south of C. Then D is

A. 3 km east and 1 km north of A

B. 3 km east and 3 km north of A

C. 3 km east and 1 km south of A

D. 3 km west and 3 km north of A

 

Q. 37 A lead cuboid of 8 inches in length, 11 inches in breadth, and 2 inches thick was melted and resolidified into the form of a rod of 8 inches diameter. The length of such a rod, in inches, is nearest to

A. 3

B. 3.5

C. 4

D. 4.5

 

Questions: 38 – 41

Directions for Questions: Answer the following questions based on the information from an airline about their passengers (pax) in particular sectors is given below:

The proportion of males and the proportion of vegetarian pax are given below. The airline has a total of 800 passengers, 80% of whom are in the Mumbai – Delhi sector and rest are equally divided between Mumbai – Hyderabad and Mumbai – Bangalore.

 

Q. 38 What is the percentage of male pax in the Mumbai – Delhi sector?

A. 40

B. 45

C. 50

D. 60

 

Q. 39 In Mumbai – Bangalore, twenty five per cent of the vegetarians are male. What is the difference between the number of female vegetarians and male non-vegetarians?

A. less than 8

B. 10

C. 12

D. 16

 

Q. 40 What is the percentage of vegetarian pax in Mumbai – Bangalore?

A. 40

B. 45

C. 50

D. 60

 

Q. 41 What is the percentage of vegetarian pax in Mumbai – Bangalore? 40 45 50 60 41. In the Mumbai – Delhi sector, 50% of the pax are vegetarian males. Which of the following statements is correct?

A. Except vegetarian males, all other groups have same number of pax.

B. Except non-vegetarian males, all other groups have same number of pax.

C. Except vegetarian females, all other groups have same number of pax.

D. None of these.

 

Questions: 42 – 43

Use the following information: Eighty five people went to a lottery shop where they could bet on the DhanLaksmi, Rajshri, and Gauri lotteries. It was known that 20 of them took all three ets, and 55 of them took at least two of the three bets. Each bet cost Re. 1, and the total receipt of the lottery shop was Rs. 145.

 

Q. 42 How many people did not try any of the bets?

A. 5

B. 10

C. 15

D. 20

 

Q. 43 How many people took exactly one bet?

A. 5

B. 10

C. 15

D. 20

 

Q. 44 John bought five toffees and ten chocolates together for forty rupees. Subsequently, he returned one toffee and got two chocolates in exchange. The price of an chocolate would be

A. 1

B. 2

C. 3

D. 4

 

Q. 45 Let aᵥ₊₁= 2 aᵥ + 1 (ᵥ = 0,1,2,….) and a₀ = 0 . Then v₁₀ nearest to

A. 1023

B. 2047

C. 4095

D. 8195

 

Q. 46 Suppose you have a currency, named Rubble, in three denominations: 1 Rubble, 10 Rubbles and 50 Rubbles. In how many ways can you pay a bill of 95 Rubbles?

A. 15

B. 16

C. 18

D. 19

 

Q. 47 Consider four-digit numbers for which the first two digits are equal and the last two digits are also equal. How many such numbers are perfect squares?

A. 3

B. 2

C. 4

D. 1

 

Q. 48 The price of Coffee (in rupees per kilogram) is 100 + 0.10n, on the nth day of 2007 (n = 1, 2, …, 100), and then remains constant. On the other hand, the price of Ooty tea (in rupees per kilogram) is 89 + 0.15n, on the nth day of 2007 (n = 1, 2, …, 365). On which date in 2007 will the prices of coffee and tea be equal?

A. May 21

B. April 11

C. May 20

D. April 10

 

Q. 49 Two circles with centres P and Q cut each other at two distinct points A and B. The circles have the same radii and neither P nor Q falls within the intersection of the circles. What is the smallest range that includes all possible values of the angle AQP in degrees?

A. Between 0 and 90

B. Between 0 and 30

C. Between 0 and 60

D. Between 0 and 75

 

Q. 50 A quadratic function ƒ(x) attains a maximum of 3 at x = 1. The value of the function at x = 0 is 1. What is the value ƒ(x) at x = 10?

A. –119

B. –159

C. –110

D. –180

 

Questions: 51 – 54

Answer the questions below on the basis of the following passage.

More selective than most chemical pesticides in that they ordinarily destroy only unwanted species, biocontrol. agents (such as insects, fungi, and viruses) eat, infect, or parasitize targeted plant or animal pests. However, biocontrol agents can negatively affect nontarget species by, for example, competing with them for resources: a biocontrol agent might reduce the benefits conferred by a desirable animal species by consuming a plant on which the animal prefers to lay its eggs. Another example of indirect negative consequcnces occurred in England when a virus introduced to control rabbits reduced the amount of open ground (because large rabbit populations reduce the ground cover), in turn reducing underground ant nests and triggering the extinction of a blue butterfly that had depended on the nests to shelter its offspring. The paucity of known extinctions or disruptions resulting from indirect interactions may reflect not the infrequency of such mishaps but rather the failure to look for or to detect them: most organisms likely to be adversely affected by indirect interactions are of little or no known commercial value and the events linking a biocontrol agent with an adverse effect are often unclear. Moreover, determining the potential risks of biocontrol agents before they are used is difficult, especially when a nonnative agent is introduced, because, unlike a chemical pesticide, a biocontrol agent may adapt in unpredictable ways. so that it can feed on or otherwise harm new hosts.

 

Q. 51The passage is primarily concerned with

A. explaining why until recently scientists failed to recognize the risks presented by biocontrol agents.

B. emphasizing that biocontrol agents and chemical pesticides have more similarties than differences.

C. suggesting that only certain biocontrol agents should be used to control plant or animal pasts.

D. arguing that biocontrol agents involve risks, some of which may not be readily discerned.

 

Q. 52 The passage suggests that the author would be most likely to agree with which of the following statements about the use of biocontrol agents?

A. Biocontrol agent should be used only in cases where chemical pesticides have proven ineffective or overly dangerous.

B. Extinctions and disruptions resulting from the use of biocontrol agents are likely to have increasingly severe commercial consequences.

C. The use of biocontrol agents does not require regulation as stringent as that required by the use of chemical pesticides.

D. The risks of using native biocontrol agents may be easier to predict than the risks of using nonnative biocontrol agents.

 

Q. 53 Which of the following is mentioned in the passage as an indirect effect of using a biocontrol agent?

A. Reduction of the commercial value of a desirable animal species

B. An unintended proliferation of a nontarget animal species

C. An unforeseen mutation in a target species

D. Diminution of the positive effects conferred by a nontarget animal species.

 

Q. 54 The example presented by the author in highlight text most clearly serves to illustrate

A. a situation in which a species is less vulnerable to biocontrol agents than it would have been to chemical pesticides.

B. a way in which the introduction of a biocontrol agent can affect a nontarget species.

C. a nonnative agent’s adapting in an unpredictable way that results in damage to a new host.

D. The contention that biocontrol agents can harm nontarget species by competing with them for resources

 

Q. 55 Arrange sentences A, B, C and D between sentences 1 and 6 to form a logical sequence of six sentences.

1. Buddhism is a way to salvation.

A. But Buddhism is more severely analytical.

B. In the Christian tradition there is also a concern for the fate of human society conceived as a whole, rather than merely as a sum or network of individuals.

C. Salvation is a property, or achievement of individuals.

D. Not only does it dissolve society into individuals, the individual in turn is dissolved into component parts and instants, a stream of events.

6. In modern terminology, Buddhist doctrine is reductionist.

A. ABDC

B. CBAD

C. BDAC

D. ABCD

 

Q. 56 Arrange sentences A, B, C and D between sentences 1 and 6 to form a logical sequence of six sentences.

1. The problem of improving Indian agriculture is both a sociological and an administrative one.

A. It also appears that there is a direct relationship between the size of a state and development.

B. The issues of Indian development, and the problem of India’s agricultural sector, will remain with us long into the next century.

C. Without improving Indian agriculture, no liberalisation and delicensing will be able to help India.

D. At the end of the day, there has to be a ferment and movement of life and action in the vast segment of rural India.

6. When it starts marching, India will fly.

A. DABC

B. CDBA

C. ACDB

D. ABCD

 

Questions: 57 – 58

Hidden Island is an obscure island which is inhabited by two types of people: the ‘Yes‘ type and the ‘No‘ type. Native of type ‘Yes‘ ask only questions the right answer to which is ‘Yes‘ while those of type ‘No‘ ask only questions the right answer to which is ‘No’. For ex., The ‘Yes‘ type will ask questions like “Is 2 plus 2 equal to 4?” while the ‘No‘ type will ask questions like “Is 2 plus 2 equal to five?” The following questions are based on your visit to the Hidden Island.

 

Q. 57 If an islander asks, “Do I belong to the ‘No‘ type, which of the following is correct?

A. He is a ‘No‘.

B. He is a ‘Yes‘.

C. It is impossible for him to have asked such a question.

D. His type cannot be identified.

 

Q. 58 Aman and Mohan are brothers from the Island. Mohan asks you, “Is at least one of us brothers of type ‘No’”? You can conclude that

A. Aman is ‘NO‘, Mohan is ‘Yes‘

B. both are ‘Yes‘.

C. Aman is ‘Yes‘, Mohan is ‘No‘.

D. both are ‘No‘.

 

Q. 59 Arrange sentences A, B, C and D between sentences 1 and 6 to form a logical sequence of six sentences.

1. Good literary magazines have always been good because of their editors.

A. Furthermore, to edit by committee, as it were, would prevent any magazine from finding its own identity.

B. The more quirky and idiosyncratic they have been, the better the magazine is, at least as a general rule.

C. But the number of editors one can have for a magazine should also be determined by the number of contributions to it.

D. To have four editors for an issue that contains only seven contributions, it is a bit silly to start with.

6. However, in spite of this anomaly, the magazine does acquire merit in its attempt to give a comprehensive view of the Indian literary scene as it is today.

A. ABCD

B. BCDA

C. ABDC

D. CBAD

 

Questions: 60 – 63

Directions for Questions: Answer the following questions based on the statements given below: Following questions are about 6 project reports of different heights from 6 different students and of 6 different colors displayed on a cupboard.

(i) There are three Project Reports on each side of the aisle.

(ii) These six Project Reports are labeled as Piyush, Qadar, Richa, Sandesh, Tanvi and Urvashi.

(iii) The Project Reports are of different colours, namely, Red, Blue, Green, Orange, Yellow and White.

(iv) The Project Reports are of different heights.

(v) Tanvi, the tallest Project Report, is exactly opposite to the Red coloured Project Report.

(vi) The shortest Project Report is exactly opposite to the Green coloured Project Report.

(vii) Urvsahi, the Orange coloured Project Report, is located between Piyush and Sandesh.

(viii) Richa, the Yellow coloured Project Report, is exactly opposite to Piyush.

(ix) Qadar, the Green coloured Project Report, is exactly opposite to Urvashi.

(x) Piyush, the White coloured Project Report, is taller than Richa, but shorter than Sandesh and Qadar.

 

Q. 60 What is the colour of the Project Report diagonally opposite to the Yellow coloured Project Report?

A. White

B. Blue

C. Green

D. Red

E. none of these

 

Q. 61 Which is the second tallest Project Report?

A. Piyush

B. Sandesh

C. Qadar

D. Richa

E. Can’t determine

 

Q. 62 What is the colour of the tallest Project Report?

A. Red

B. Blue

C. Green

D. none of these

 

Q. 63 What is the color of the report named Richa?

A. Red

B. Blue

C. Green

D. Yellow

 

Q. 64 In the following question, a part of a sentence is left blank. Choose from among the four options given below the question, the one which would best fill the blanks.

When we call others dogmatic, what we really object to is ___.

A. their giving the dog a bad name

B. their holding beliefs that are different from our own

C. the extremism that goes along with it

D. the subversion of whatever they actually believe in concomitantly

 

Q. 65 Science, because people engage in it, is a socially embedded activity. It progresses by hunch, vision, and intuition. Much of its change through time does not record a closer approach to absolute truth, but the alternation of the cultural contexts that influence it so strongly. Facts are not pure and unsullied bits of information— culture influences what we see and how we see it. Theories, moreover, are not inexorable inductions from facts. The most creative theories are often imaginative visions imposed upon facts; the source of imagination is also strongly cultural. The author implies that those who rely on scientific results should

A. realize that science relies on imagination to approach absolute truth

B. insist on pure and unsullied facts rather than on theories

C. understand that theories are frequently strict inductions from facts

D. consider the cultural biases of scientists

 

Q. 66 Baking for winter holidays is tradition that may have a sound medical basis. In midwinter, when days are short, many people suffer from a specific type of seasonal depression caused by lack of sunlight. Carbohydrates, both sugars and starches, boost the brain‘s levels of serotonin, a neurotransmitter that improve the mood. In this respect, carbohydrates act on the brain in the same way as some antidepressants. Thus, eating holiday cookies may provide an effective form of self-prescribed medication. Which one of the following can be properly inferred from the passage?

A. Seasonal depression is one of the most easily treated forms of depression.

B. Lack of sunlight lowers the level of serotonin in the brain.

C. People are more likely to be depressed in midwinter than at other times of the year.

D. Some antidepressants act by changing the brain‘s level of serotonin.

 

Questions: 67 – 70

Answer the questions below on the basis of the following passage.

When views can freely flourish in the marketplace of ideas, individuals are afforded the advantage of deciding what notions and concepts to question, support or reject. On June 8, 1789, James Madison introduced in the House of Representatives an amendment to the Constitution: ―The people shall not be deprived or abridged of their right to speak, to write, or to publish their sentiments; and the freedom of the press, as one of the great bulwarks of liberty, shall be inviolable.ǁ This commitment to a free press is a principle Americans hold firmly, because they view it as a necessary ingredient for a properly functioning political process and a critical component of a free society. Yet, since the time of America‘s founding, the politicized nature of the press has not fundamentally changed.

While conservatives and liberals alike claim that today‘s mainstream media is biased, opinionated, and devoid of objectivity and balanced analysis, American newspapers at the time of this nation‘s birth were all partisan, believing that their responsibility was not to report news, but to convey, without apology, a particular political position. Perhaps the high point of partisan newspapers was in New York during the 1920‘s, when the city had over a dozen daily papers, each geared toward a particular ethnic and political niche; people selected the paper that made the most sense of the world to them. Despite the naysayers who warn that the lack of objectivity and fair-mindedness is corrosive to society, partisan journalism can be good journalism. It produces plenty of excellent reporting and analysis and is the norm in many nations. Two centuries ago, newspapers subsidized by Andrew Jackson’s Democrats and Henry Clay’s Whigs were dependable supporters of their parties. Today‘s newspapers claim that they too are only giving their readership what it wants. Legally, the Supreme Court has tried since 1919 to clarify how free the press is. Over time, older laws that allowed publications to be punished for libel, obscenity, sedition, and publishing inflammatory material have given way to more expansive rights to publish. The First Amendment protections offered to journalists have evolved to a broader interpretation of freedom of the press. During the 1960‘s and 1970‘s, journalists exposed the government‘s mismanagement of the Vietnam War, and their investigative reporting eventually brought about the resignation of President Nixon. By the end of the twentieth century, the Constitution‘s protections were broadly held to cover the content of all papers, from the highly regarded New York Times to tabloids such as The National Enquirer.

 

Q. 67 According to the author, which of the following is true about partisan journalism throughout American history?

A. It has had a limited impact on the political process.

B. Its lack of objectivity is detrimental.

C. It has played an important role in reliably informing individuals from diverse cultural backgrounds.

D. It has essentially been the status quo since America‘s founding.

 

Q. 68 Which of the following statements about American newspapers is supported by information contained in the passage?

A. America‘s newspapers in 1789 resembled those of today in form and content.

B. The character of the press has matured since the time of America‘s founding.

C. In recent years, the press has become biased in regard to its political reporting.

D. Early American journalists did not necessarily provide a balanced analysis of events.

 

Q. 69 The author of the passage would disagree with which of the following statements?

A. The legal understanding of press freedoms has shifted over time.

B. Over time, the First Amendment protections offered to the press have become absolute.

C. America‘s legal evolution has given way to a more liberal understanding of press freedom.

D. First Amendment press rights today protect a broad section of the newspaper industry.

 

Q. 70 All of the following are examples of limitations the courts have placed on freedom of the press,

EXCEPT

A. articles deemed maliciously defamatory of individuals

B. articles viewed as offensive to society‘s views of decency

C. articles that comment negatively on a political affiliation

D. articles clearly dangerous to national security

 

Q. 71 Street crime can be averted through regulations mandating the lighting of streetlights during daytime. As daytime visibility is worse in nations farther from the equator, so obviously such regulations would be more successful in averting crime there. Actually, the only nations that have adopted such regulations are farther from the equator than the continental United States. Which of the following conclusions could be most properly drawn from the information given above?

A. Bystanders in the continental United States who were near lit streetlights during the day would be just as likely to become victims of a crime as would bystanders who were not near lit streetlights.

B. Inadequate daytime visibility is the single most important factor in street crime in numerous nations that are located farther from the equator than is the continental United States.

C. In nations that have daytime streetlight regulations, the percentage of street crime that happens in the daytime is greater than in the continental United States.

D. Daytime streetlight regulations would probably do less to avert street crime in the continental United States than they do in the nations that have the regulations.

 

Q. 72 During the construction of the Quebec Bridge in 1907, the bridge’s designer, Theodore Cooper, received word that the suspended span being built out from the bridge’s cantilever was deflecting downward by a fraction of an inch (2.54 centimeters). Before he could telegraph to freeze the project, the whole cantilever arm broke off and plunged, along with seven dozen workers, into the St. Lawrence River. It was the worst bridge construction disaster in history. As a direct result of the inquiry that followed, the engineering “rules of thumb” by which thousands of bridges had been built around the world went down with the Quebec Bridge. Twentieth-century bridge engineers would thereafter depend on far more rigorous applications of mathematical analysis. Which one of the following statements can be properly inferred from the passage?

A. Prior to 1907 the mathematical analysis incorporated in engineering rules of thumb was insufficient to completely assure the safety of bridges under construction.

B. Cooper’s absence from the Quebec Bridge construction site resulted in the breaking off of the cantilever.

C. Nineteenth-century bridge engineers relied on their rules of thumb because analytical methods were inadequate to solve their design problems.

D. Only a more rigorous application of mathematical analysis to the design of the Quebec Bridge could have prevented its collapse.

 

Q. 73 Choose the option that best captures the essence of the passage.

There are many and good reasons why women have left little in the way of literary monuments, especially poetry. The main reason is education, or more specifically the lack of it in most women‘s lives until well into the twentieth century. In the light of this, what is surprising is not that so few women wrote poetry, but that any women wrote poetry at all. When a woman who had never been to grammar school, never learnt Latin and did not know the rules of syntax, let alone of prosody, set herself to writing lines that rhymed, she was imitating an art that, admire it though she might, she did not understand — a male art, a male tradition. This meant she generally produced poetry that was at best imperfect, at worst, frankly bad.

 

A. The reason why women haven‘t created literary monuments is because until recently they were not educated. In this case, the only option for an uneducated woman was to plagiarize in order to show herself as superior.

B. The lack of education is the biggest reason for women‘s lack of achievements in the literary sphere. In the absence of the knowledge of basics, she would see the male creations as ‘ideal‘ and try to copy them.

C. The lack of education is the biggest reason for women‘s lack of achievements in the literary sphere. Feeling inferior to educated men, they would imitate them in every area including literature.

D. The lack of education is the biggest reason for women‘s lack of achievements in the literary sphere. It suited men well since in such a scenario the women had no choice but to imitate the men.

 

Questions: 74 – 77

Recently, Rajesh visited the local casino where he came across a new card game. Two players, using a normal deck of 52 playing cards, play this game. One player is called the ‘dealer‘ and the other is called the ‘player‘. First, the player picks a card at random from the deck. This is called the base card. The amount in rupees equal to the face value of the base card is called the base amount. The face values of ace, king, queen and jack are ten. For other cards the face value is the number on the card. Once the ‘player‘ picks a card from the deck, the ‘dealer‘ pays him the base amount. Then the ‘dealer‘ picks a card from the deck and this card is called the top card. If the top card is of the same suit as the base card, the ‘player‘ pays twice the base amount to the ‘dealer‘. If the top card is of the same colour as the base card (but not the same suit), then the ‘player‘ pays the base amount to the ‘dealer‘. If the top card happens to be of a different colour than the base card, the ‘dealer‘ pays the base amount to the ‘player‘. Rajesh played the game four times. First time he picked eight of clubs and the ‘dealer‘ picked queen of clubs. Second time, he picked ten of hearts and the ‘dealer‘ picked two of spades. Next time, Rajesh picked six of diamonds and the ‘dealer‘ picked ace of hearts. Lastly, he picked eight of spades and the ‘dealer‘ picked jack of spades. Answer the following questions based on these four games.

 

Q. 74 If Rajesh stopped playing the game when his gain would be maximized, the gain in Rs. Would have been

A. 12

B. 20

C. 16

D. 4

 

Q. 75 The initial money Rajesh had (before the beginning of the game sessions) was Rs. X. At no point did he have to borrow any money. What is the minimum possible value of X?

A. 16

B. 8

C. 100

D. 24

 

Q. 76 If the final amount of money that Rajesh had with him was Rs. 100, what was the initial amount he had with him?

A. 120

B. 8

C. 4

D. 96

 

Q. 77 If Rajesh stopped playing the game when his loss would be maximumm, the loss in Rs. Would have been

A. -12

B. -20

C. -8

D. -4

 

Questions: 78 – 80

Answer the questions below on the basis of the following passage.

The communities of ants are sometimes very large, numbering even up to 500, individuals: and it is a lesson to us that no one has ever yet seen quarrel between any two ants belonging to the same community. On the other hand, it must be admitted that they are in hostility not only with most other insects, including ants of different species, but even with those of the same species if belonging to different communities. I have over and over again introduced ants from one of my nests into another nest of the same species; and they were invariably attacked, seized by a leg or an antenna, and dragged out. It is evident, therefore, that the ants of each community all recognize one another, which is very remarkable. But more than this, I several times divided a nest into two halves and found that even after separation of a year and nine months they recognize one another and were perfectly friendly, while they at once attacked ants from a different nest, although of the same species.

It has been suggested that the ant of each nest have some sign or password by which they recognize one another. To test this I made some of them insensible, first I tried chloroform; but this was fatal to them, and I did not consider the test satisfactory. I decided therefore to intoxicate them. This was less easy than I had expected. None of my ants would voluntarily degrade themselves by getting drunk. However, I got over the difficulty by putting them into whisky for a few moments. I took fifty specimens – – twenty five percent from one nest and twenty five percent from another made them dead drunk, market each with a spot of paint, and put them on a table close to where other ants from one the nests were feeding.

The table was surrounded as usual with a moat of water to prevent them from straying. The ants, which were feeding, soon noticed those, which I had made drunk. They seemed quite astonished to find their comrades in such a disgraceful condition, and as much at a loss to know what to do with their drunkards as we were. After a while, however, they carried them all away; the strangers they took to the edge of the moat and dropped into the water, while they bore their friends home into the nest, where by degrees they slept off the effects of the spirits. Thus it is evident that they know their friends even when incapable of giving any sign or password.

 

Q. 78 Attitudes of ants towards strangers of the same species may be categorized as

A. indifferent

B. curious

C. hostile

D. passive

 

Q. 79 The author’s anecdotes of the inebriated ants would support all the following inductions except the statement that

(I) ants take unwillingly to intoxicants

(II) ants aid comrades in distress

(III) ants have invariable recognition of their community members

(IV) ants recognize their comrades by a mysterious password.

A. I and II

B. I and III

C. Only III and IV

D. only IV

 

Q. 80 According to the passage, chloroform was less successful than alcohol for inhibiting communication because of

(I) its expense

(II) its unpredictable side effects

(III) its unavailability

(IV) its fatality

A. I and II

B. I and III

C. Only III and IV

D. only IV

 

Q. 81  All of the following sentences A, B, C and D are taken from a same passage and jumbled up. but one of them is incoherent. Find the incoherent statement.

A. Reliance Industries (RIL), India‘s largest company, saw its earnings before interest and taxes jump 15 per cent from the first quarter to the second, though net profits grew at a far more modest 2.6 per cent.

B. Part of the reason for this was a sharp, 8 per cent-plus fall in its refining margins, down from $8.40 per barrel in Q1 to $7.70 per barrel in Q2.

C. For RIL, refining and selling oil and making petrochemicals are not new. Investors understand these businesses and value them realistically.

D. This fall happened because the Asian demand — and, therefore, pricing — of lighter fuels like diesel and petrol was under pressure, possibly because most major emerging economies are growing slower than they were before.

Incoherent statement is:

A. A.

B. B.

C. C.

D. D.

 

Q. 82 All of the following sentences A, B, C and D are taken from a same passage and jumbled up. but one of them is incoherent. Find the incoherent statement.

Sachin did not ask for this honour.

A. There are consolation prizes which console, and some which char the soul.

B. I am not suggesting we write a condolence letter, but sympathy is certainly due to Sachin.

C. Membership of Parliament is a handsome freebie for Sachin Tendulkar

D. Nomination to the Rajya Sabha is a pretty desultory substitute for someone who has been promised the Bharat Ratna.

Delhi‘s politicians, ever eager to climb a bandwagon, led the clamour for Sachin‘s elevation to jewel of India after he got his 99th international hundred.

Incoherent statement is:

A. A.

B. B.

C. C.

D. D.

 

Q. 83 Education Secretary: Too many adults lack sufficient skills for job advancement because of the cost of higher education. Businesses should partner with educators to create curricula that are tailored to the needs of these people, thus increasing the likelihood that the cost of enrollment will be a safe investment.

Union Leader: What good is altering the curricula if students simply do not have the funds to enroll or the time to attend? What we need are more generous educational grants targeted to working adults, and more flexible work hours for working students. Both the education secretary and the union leader make the point that … ?

A. the cost of higher education for many adults is prohibitively high

B. many adult students do not have the time to attend courses in higher education

C. the cost of higher education is unfairly high

D. many adult students will not attend classes in higher education unless they are convinced that doing so will be a good investment.

 

Q. 84 In a political system with only two major parties, the entrance of a third-party candidate into an election race damages the chances of only one of the two major candidates. The third-party candidate always attracts some of the voters who might otherwise have voted for one of the two major candidates, but not voters who support the other candidate. Since a third-party candidacy affects the two major candidates unequally, for reasons neither of them has any control over, the practice is unfair and should not be allowed. If the factual information in the passage above is true, which of the following can be most reliably inferred from it?

A. If the political platform of the third party is a compromise position between that of the two major parties, the third party will draw its voters equally from the two major parties.

B. If, before the emergence of a third party, voters were divided equally between the two major parties, neither of the major parties is likely to capture much more than onehalf of the vote.

C. A third-party candidate will not capture the votes of new voters who have never voted for candidates of either of the two major parties.

D. The political stance of a third party will be more radical than that of either of the two major parties.

 

Q. 85 It is the powerful compound capsaicin that makes a chili pepper hot; “a single drop that has no taste and odor is capable of detection” by humans at one part per million.

A. a single drop that has no taste and odor is capable of detection

B. a single drop is detectable, though without taste and odor

C. a single tasteless and odorless drop can be detected

D. single tasteless and odorless drops are capable of detection

 

Q. 86 Old, longstanding firms concentrate on protecting what they have already amassed. Consequently, they rarely innovate and often underestimate what consequences innovation by other companies will have. The best example of one such defensive strategy is the fact that ___.

Which of the following best completes the passage?

A. electronics and mass-produced gears eliminated the traditional market for pocket watches, clearing the way for marketing them as elegant, old-fashioned luxury items.

B. an extremely popular prefabricated house was introduced by a company that, several years before, had failed miserably with its product line of glass houses.

C. a once-leading maker of buggy whips responds to the new availability of stick shifts by attempting to make better buggy whips.

D. smoking pipes, originally designed for use by typically older, more traditional smokers of tobacco, are now bought mostly by young smokers of scented or flavored herbal blends.

 

Q. 87 Arrange the sentences A, B, C and D in a proper sequence so as to make a coherent paragraph.

A. After several routine elections there comes a ‘critical’ election which redefines the basic pattern of political loyalties, redraws political geography and opens up political space.

B. In psephological jargon, they call it realignment.

C. Rather, since 1989, there have been a series of semi-critical elections.

D. On a strict definition, none of the recent Indian elections qualifies as a critical election.

A. ABCD

B. ABDC

C. DBAC

D. DCBA

 

Q. 88  Arrange the sentences A, B, C and D in a proper sequence so as to make a coherent paragraph.

A. Trivial pursuits marketed by the Congress, is a game imported from Italy.

B. The idea is to create an imaginary saviour in times of crisis so that the party doesn’t fall flat on its collective face.

C. Closest contenders are Mani Shankar Aiyar, who still hears His Master’s Voice and V. George, who is frustrated by the fact that his political future remains Sonia and yet so far.

D. The current champion is Arjun for whom all roads lead to Rome, or in this case, 10 Janpath.

A. ABDC

B. ABCD

C. DCBA

D. CDBA

 

Questions: 89 – 92

Directions for questions: Answer the questions based on the following

information. A and B are two sets (e.g. A = Mothers, B = Women). C = A . B =>The elements that could belong to both the sets (e.g. women who are mothers) is given by the set C = A . B. D = A ∪ B =>The elements which could belong to either A or B, or both, is indicated by the set D = A ∪ B.

∅ => A set that does not contain any elements is known as a null set represented by ∅ (e.g. if none of the women in the set B is a mother, then C = A .B is a null set, or C = ∅ ).

Let ‘V‘ signify the set of all vertebrates,

‘M‘ the set of all mammals,

‘D‘ dogs,

‘F‘ fish

‘A‘ alsatian and

‘P‘, a dog named Pluto.

 

Q. 89 Given that X = M.D is such that X = D. Which of the following is true?

A. All dogs are mammals

B. Some dogs are mammals

C. X = ∅

D. All mammals are dogs

 

Q. 90 If Y = F . (D . V) is not a null set, it implies that

A. all fish are vertebrates

B. all dogs are vertebrates

C. some fish are dogs

D. None of these

 

Q. 91 If Z = (P . D) ∪M, then

A. the elements of Z consist of Pluto, the dog, or any other mammal

B. Z implies any dog or mammal

C. Z implies Pluto or any dog that is a mammal

D. Z is a null set

 

Q. 92 If P . A = ∅ and P ∪ A = D, then which of the following is true?

A. Pluto and alsatians are dogs

B. Pluto is an alsatian

C. Pluto is not an alsatian

D. D is a null set

 

Questions: 93 – 95

Read each of the following passages carefully and answer the questions that follow.

Atmospheric jet streams were discovered towards the end of World War II by U.S. bomber pilots over Japan and by German reconnaissance aircraft over the Mediterranean. The World Meteorological Organization defines a jet stream as a strong, narrow air current that is concentrated along nearly horizontal axis in the upper troposphere or stratosphere (10 to 50km altitude), characterized by wind motions that produce strong vertical lateral shearing action and featuring one of more velocity maximum. Normally a jet stream is thousands of kilometers long, hundreds of kilometers wide and several kilometers deep. The vertical wind shear is of the order of 5 to 10 m/sec per kilometer, and the lateral shear is of the order of 5 m/sec per 100 km. An arbitrary lower limit of 30m/sec is assigned to the speed of the wind along the axis of a jet stream. With abundant radio-sonic data now available over the Northern Hemisphere it is possible to map the jet streams in the upper troposphere (near 10 to 12 km) in their daily occurrence and variation and to forecast them reasonably well with numerical prediction techniques. Upper-air information from the Southern Hemisphere is still sparse. Constant-level balloons (the so-called GHOST balloons) and satellite information on temperature structure and characteristic cloud formations in the atmosphere are serving to close the data on the global jet stream distribution. The strongest winds known in jet streams have been encountered over Japan, where speeds up to 500 km/hr (close to 300 knots) occur. A persistent band of strong winds occurs during the winter season over this region, flowing from the southwest and leading tropical air northern India into juxtaposition with polar and arctic air from Siberia. A similar region of confluence of air masses with vastly different temperatures exists over the central and eastern United States, leading to a maximum frequency of occurrence of jet streams during winter and spring.

 

Q. 93 An atmospheric jet stream is

A. a rare phenomenon.

B. three dimensional.

C. concentrated in the northern hemisphere.

D. more common in summer.

 

Q. 94 Detailed studies of atmospheric streams have been made over

A. South Africa

B. Europe

C. Australia

D. Antarctica

 

Q. 95 The atmospheric jet stream consists of

A. cumulous clouds bearing saturated moisture.

B. debris caused by meteorites.

C. air currents.

D. effluents from speeding aircraft.

 

Q. 96 Some decisions will be fairly obvious — ‘no-brainers‘. Your bank account is low, but you have a two week vacation coming up and you want to get away to some place warm to relax with your family. Will you accept your in-laws‘ offer of free use of their Florida beachfront condo? Sure. You like your employer and feel ready to move forward in your career. Will you step in for your boss for three weeks while she attends a professional development course? Of course.

Choose the option that best captures the essence of the text.

A. Some decisions are obvious under certain circumstances. You may, for example, readily accept a relative‘s offer of free holiday accommodation. Or step in for your boss when she is away.

B. Some decisions are no-brainers. You need not think when making them. Examples are condo offers from in-law and job offers from bosses when your bank account is low or boss is away.

C. Easy decisions are called ‘no-brainers‘ because they do not require any cerebral activity. Examples such as accepting free holiday accommodation abound in our lives.

D. Accepting an offer from in-laws when you are short on funds and want a holiday is a no-brainer. Another no-brainer is taking the boss‘s job when she is away.

 

Questions: 97 – 100

Direction for questions: Answer the questions based on the following information.

A series S₁ of five positive integers is such that the third term is half the first term and the fifth term is 20 more than the first term. In series S₂, the nth term defined as the difference between the (n+1) term and the nth term of series S₁, is an arithmetic progression with a common difference of 30.

 

Q. 97 First term of S₁ is

A. 80

B. 90

C. 100

D. 120

 

Q. 98 What is the difference between second and fourth terms of S₁?

A. 10

B. 20

C. 30

D. 60

 

Q. 99 What is the average value of the terms of series S₁?

A. 60

B. 70

C. 80

D. Average is not an integer

 

Q. 100 What is the sum of series S₂?

A. 10

B. 20

C. 30

D. 40

 

 

Answer Sheet
Question 1 2 3 4 5 6 7 8 9 10
Answer A A C C A A A B C D
Question 11 12 13 14 15 16 17 18 19 20
Answer D A D B C C B A C D
Question 21 22 23 24 25 26 27 28 29 30
Answer D A A A A C A B C D
Question 31 32 33 34 35 36 37 38 39 40
Answer A A B A C B B B D A
Question 41 42 43 44 45 46 47 48 49 50
Answer D C C B A A D C C B
Question 51 52 53 54 55 56 57 58 59 60
Answer D D D B B D C A B D
Question 61 62 63 64 65 66 67 68 69 70
Answer E B D B D D D D B C
Question 71 72 73 74 75 76 77 78 79 80
Answer D A B A B D C C D D
Question 81 82 83 84 85 86 87 88 89 90
Answer C C A C C C A B A C
Question 91 92 93 94 95 96 97 98 99 100
Answer A C B B C A C A C B

CAT Previous Year Paper 2013

CAT 2013

Section

Questions

Marks

Quantitative Aptitude

22 Questions

66

DILR

19 Questions

57

English

19 Questions

57

Questions: 1 – 2

Functions g and h are defined on n constants, a₀,a₁,a₂,a₃, … aₙ₋₁, as follows:

g(aₚ, aq) = a|ₚ₋q| , if |p-q| ≤ (n-4)

= aₙ₋|ₚ₋q, if |p-q|>(n-4)

h (aₚ,aq) = aₖ, where k is the remainder when p+q is divided by n.

 

Q. 1 If n=10, find the value of g(g(a₂,a₈),(a₁,a₇)),

A. a₉

B. a₇

C. a₂

D. a₀

 

Q. 2 If h(aₖ,aₘ)=aₘ for all m, where 1≤m

A. 0

B. 1

C. n-1

D. n-2

 

Q. 3 In a bag there are total of 150 coins in three denominators -₹1,₹2 and ₹5-with at least one coin of each denomination being present in the bag. The total value of the Re.1 coins is at least 50% of the total value of the coins in the bag. If there are 23 ₹5 coins in the bag and the total value of the ₹2 coins is at least 3% of the total value of the coins in the bag, find the number of ₹2 coins in the bag.

A. 2

B. 3

C. 4

D. 1

 

Q. 4 Let P,Q,S,R,T,U and V represent the seven distinct digits from 0 to 6, not necessarily in that order. If PQ and RS are both two-digit numbers adding up to the three-digit number TUV, find the value of V.

A. 3

B. 6

C. 5

D. Cannot be determined

 

Q. 5 There are five cards lying on a table in one row. Five numbers from among 1 to 100 have to be written on them, one number per card, such that the difference between the numbers on any two adjacent cards is not divisible by 4 is written down on another card, i.e., a sixth card, in that order. How many sequences can be written down on the sixth card?

A. 2²3³

B. 4(3)⁴

C. 4²3³

D. 4²3⁴

 

Questions: 6 – 9

Summary of the estimate of memory space occupied by the information worldwide, stored in various storage media, in the year 2000

 

Q. 6 If the information contained in each X-Rat occupies 30MB of memory space on an average, and by using a new technology each X-Ray is now stored in magnetic media that saves 60% of the memory space required, what is the total amount of memory space of magnetic media that is required to to store all the X-Rays available in the year 2000. (1MB=10⁶ Bytes of memory space)

A. 10,170 TB

B. 6,780

C. 1,703,170 TB

D. None of these

 

Q. 7 The information stored in Newspapers, Books and Periodicals forms what percentage of the total information stored on paper media? Assume that the information stored per unit memory space occupied is the same for all media mentioned.

A. 37.5%

B. 45%

C. 57%

D. 54%

 

Q. 8 When compared to the total memory space occupied by the information stored in any single category of storage media, what is the highest percentage share of memory space occupied by the information stored in any single media within that

category(approximately)?

A. 68.75%

B. 62.5%

C. 96%

D. 98.3%

 

Q. 9 Due to advances in technology, the information stored per unit memory space occupied worldwide increases by 20% every year, while the total memory space available worldwide increases at the rate of 10% every year. If in the year 2000, 80% of the memory space available worldwide is occupied by the information available worldwide, and the information available worldwide were to increase by 45% every year, then which of the following years is the earliest by which there will be a shortage of memory space?

A. 2002

B. 2003

C. 2004

D. 2005

 

Q. 10 a,b and are the lengths of the triangle ABC and d,e and f are the lengths of the sides of the triangle DEF. If the following equations hold true:

a(a+b+c)=d²

b(a+b+c)=e²

c(a+b+c)=f²

then which of the following is always true of triangle DEF?

A. It is an acute-angled triangle

B. It is an right-angled triangle

C. It is an obtuse-angled triangle

D. None of the above

 

Q. 11 Sujith looked at the six-digit number on his CAT admit card and said “If I multiply the first two digits with three, I get all ones. If I multiply the last two digits 9, I get all threes.” What is the sum of the digits of the number on Sujith’s admit card?

A. 30

B. 33

C. 60

D. 45

 

Q. 12 Two cars P and Q start from two points A and B towards each other simultaneously. They meet for the first time 40km from B. After meeting they exchange their speeds as well as directions and proceed to their respective starting points. On reaching their starting points, they turn back with the same speeds and meet at a point 20 km from A. Find the distance between A and B.

A. 130 km

B. 100 km

C. 120 km

D. 110 km

 

Q. 13 Consider the following two curves in the XY plane:

y=2x³+3x²+4 and

y=3x²-2x+8

Which of the following statements is true for -3≤x≤2?

A. The two curves intersect thrice

B. The two curves intersect twice

C. The two curves intersect once

D. The two curves do not intersect

 

Q. 14 A cuboidal aquarium, of base dimensions 100cm*80cm and height 60cm, is filled with water to its brim. The aquarium is now tilted along one of the 80cm edges and the water begin to spill. The tilting is continued till the water surface touches a line on the base which is at a distance of one-third of the length from(and parallel to) the edge on which the aquarium is being tilted. Now the box is returned to its original position. By how many centimeters has the height of water reduced?

A. 50

B. 40

C. 20

D. 10

 

Q. 15 Some persons are standing at distinct points on a circle, all facing towards the center. Each possible pair of persons who are not adjacent sing a three-minute sing, one pair after another. If the total time taken by all the pairs to finish singing is 1 hour, find the number of persons standing on the circle

A. 5

B. 7

C. 9

D. 8

 

Q. 16 In a triangle PQR, PQ=12 cm and PR=9 cm and ∠Q+∠R=120⁰

Find the length of QR

A. 15/√2 cm

B. 3√13 cm

C. 5√5 cm

D. 5√17 cm

 

Q. 17 In a triangle PQR, PQ=12 cm and PR=9 cm and ∠Q+∠R=120⁰

If the angle bisector of ∠P meets QR at M, find the length of PM

A. 28√5/9 cm

B. 42√5/11 cm

C. 36√3/7 cm

D. 4√3 cm

 

Questions: 18 – 21

The following is the table of points drawn at the end of all matches in a six-nation Hockey tournament, in which each country played with every other country exactly once. The table gives the positions of the countries in terms of their respective total points scored(i.e., in the decreasing order of their total points). Each win was worth three points, each draw one point, and there were no points for a loss. Some information in the table has been intentionally left out. The results of none of the individual matches are known, except that Pakistan beat India and no two teams finished with the same number of points.

 

Q. 18 Which of the following matches was a draw?

A. India vs South Korea

B. Spain vs Netherlands

C. Netherlands vs South Korea

D. Spain vs South Korea

 

Q. 19 The total number of points won by India is

A. 5

B. 6

C. 7

D. Cannot be determined

 

Q. 20 The total number of goals scored in the match between Netherlands and Pakistan is

A. 0

B. 1

C. 2

D. Cannot be determined

 

Q. 21 The number of goals scored by Australia against India is at most

A. 5

B. 4

C. 3

D. 2

 

Q. 22 Outside a sweet shop, its name “Madhu Sweet House” is displayed using blinking lights. Each word flashes at a regular interval and remains lit for 1 second. After remaining lit for 1 second, “Madhu” remains unlit for 3¹/₂seconds, “Sweet” remains unlit for 5³/₄ seconds and “House” remains until 9¹/₈ seconds. If all the words flash together at 8:00 p.m. sharp, find the time interval between the next time the first two words flash together and the next time the last two words flash together

A. 45 seconds

B. 22.5 seconds

C. 112 seconds

D. 6.75 seconds

 

Q. 23 If g(x)=p|x|-qx², where p and q are constants, then at x=0, g(x) will be

A. maximum when p>0,q>0

B. minimum when p<0,q<0

C. minimum when p>0,q<0

D. maximum when p>0,q<0

 

Q. 24 A television company manufactures two models of televisions-A and B. Each unit of model A requires four hours to manufacture and each unit of model B requires two hours to manufacture. The total time available in a month to manufacture these two models is 1600 hours. The profits generated on selling each unit of model A and each unit of model B are ₹1200 and ₹1000 respectively. Find the number of units of each of model A and model B television to be manufactured to maximize the profit.

A. 200 model as As and 600 model Bs

B. 800 model as As

C. 800 model Bs

D. None of the above

 

Q. 25 The age of a son, who is more than two years old, is equal to the units digit of the age of his father. After ten years, the age of the father will be thrice the age of the son. What is the sum of the present ages of the son and the father?

A. 30 years

B. 36 years

C. 40 years

D. Cannot be determined

 

Q. 26 Given that -3

A. max[(x+y)(x-y)]-min[(x+y)(x-y)]=57¹/₂

B. max[(x-y)²]=169/4

C. min[(x-y)²]=1

D. All of the above

 

Q. 27 Each side of a polygon is either parallel to the x-axis or parallel to the y-axis. A corner of the polygon is known as convex if the corresponding internal angle is 90⁰ and as concave if the corresponding internal angle is 270⁰. If the polygon has 26 convex corners, the number of its concave corners is

A. 18

B. 22

C. 26

D. 24

 

Q. 28 The density of a liquid us defined as the weight per unit volume of the liquid. The densities of two liquids A and B are in the ratio 2:1. The liquid B evaporates at a rate (in kg/hr) which is twice as fast compared to that of liquid A, which evaporates at a rate of 1 kg/hour. If 70 kg of liquid A is mixed with 30 kg of liquid of B to form a mixture, find the number of hours the mixture needs to be evaporated so that the density of the resultant mixture is 1.04 times that of the original mixture. Assume that there is no chemical reaction between the liquids

A. 2.5

B. 3

C. 3.5

D. 4

 

Q. 29 Let f(x)=1/(1+x²) and g(x)=e⁻ˣ/(1+[x]), where [x] is the greatest integer less than or equal to x. Then which of the following domain is truw?

I. domain of (f+g)=R-(-2,-1]

II. domain of (f+g)=R-[-1,0)

III. [range of f]∩[range of g]=[-2,1/2]

iV. [range of f]∩[range of g]=[-1/2,1/2]-{0}

A. Both II and IV

B. Both I and III

C. Both I and IV

D. Both II and III

 

Q. 30 The line L passing through the points (1,1) and (2,0) meets the y-axis at A. The line through the point (1/2,0) and perpendicular to L meets the y-axis at B and L at C. Find area of the triangle ABC

A. 25/16

B. 16/9

C. 32/19

D. 40/23

 

Q. 31  The following question presents four statements, of which three, when places in appropriate order, would form a contextually complete paragraph. Pick the statements that is not part of the context

(A) But as access to other texts is enjoyed more widely, some of the dominance textbooks now enjoy will wane.

(B) As indeed will the power of teachers-whose prejudices may often be just as ingrained as those found in textbooks, and rather harder to pin down

(C) It won’t be long before children, will be able to access, by way of smartphones, the textbooks prescribed for their courses

(D) As long as textbooks in one form or another are used and as long as they are issued or approved by the state, they will remain a political issue

A. (A)

B. (B)

C. (C)

D. (D)

 

Questions: 32 – 35

Psychotherapeutic processes deal with psychological problems, ranging From mild ones like a depressed mood, to more subtle ones like interpretation of dreams to more  controversial problems like dissociative identity disorder. Denied emotions trot admitting or voicing one’s emotions to the terapist) is a root cause of many psychological problems as honest communication is the %metro uno factor for the psychotherapeutic process to work. Emotional honesty can be a difficult task for the client or patient, Psychotherapists make analysis of dreams a significant part of their work. Il is tempting to wish petulantly that the unconscious would speak to us more clearly as significance of many dreams eludes us. But dreams that can be interpreted provide helpful information like warnings of personal pitfalls; solution guides to problems; sources of necessary information and judgement; as direction-finders when we feel lost; as pointers to the way we need to go when we are floundering and the message always seems to be one designed to nurture spiritual growth The unconscious may communicate to us when we are awake with as much elegance and beneficence as when we are asleep. although in a slightly different Form of ‘idle thoughts’ or oven Fragments of thought. As with dreams. we pay these idle thoughts no attention and cast them aside as insignificant. Hence patients in psychoanalysis are instructed to say everything, however insignificant, that comes in their minds. Idle thoughts provide us with insight into ourselves and others. The seemingly alien and unwanted quality is characteristic of unconscious material and its manner of presentation to the conscious mind. This and the associated resistance of the conscious mind led Freud to perceive the unconscious as a repository of the primitive, the antisocial and the evil within us_ He tended to assume that mantel illness somehow resided in the unconscious as a demon in the subterranean depths of our mind. To Carl Jung fell the responsibility of correcting this which he did through his work The Wisdom of the Unconscious.” As he concluded, mental illness i5 not a product of the unconscious but a phenomenon or consciousness or a disordered relationship between conscious and unconscious. Consider the matter of repression_ Freud discovered in his patents sexual desires and hostile feelings of which they were unaware but which were making them ill, Because these desires and Feelings resided in the unconscious, the notion arose that it was the Unconscious that caused mental illness. But why were these desires and feelings in the unconscious in the firs’ place? Why were they repressed? The answer is that the conscious mind did not want them. And it is in this not wanting, this disowning. that the problem lies.

 

Q. 32 A major difference between the points of view expressed by Freud and Jung as discussed in the passage is

A. One considered ailments of the mind to be the resu:t. of conflict between the conscious and the unconscious, while the other considered them to be inherent in the unconscious

B. One considered that ailments of the mind are grounded in the conscious, while the other considered them to be triggered by the unconscious

C. One oonsidere. that the evil qualities of human beings reside in their unconscious mind, while the other considered lhat the unconscious mind repressed desires and feeling

D. One considered that ailments of the mind are grounded in the unconscious, while the other considered them to be triggered by the conscious.

 

Q. 33 Which of the following statements are logically consistent with the content of the paragraph?

(I) Idle thoughts can sometimes illuminate the situation of the person as valuable messages from the unconscious can be received

(III Emotionally dishonest clients are very poor communicators and suffer from psychologcal problems.

(III) Dissociative identity disorder is primarily due to denied emotions_

(IV) Dreams can help effect. in us. better understanding, and development, of file spirit within.

(V) Honest communication and open in can positively influence a psychotherapeutic decision

A. I,III,IV,V

B. I,IV,V

C. I,II,IV

D. II,IV,V

 

Q. 34 If a paragraph were lo be inserted between the first and the second paragraphs, it would most likely deal with which of the following?

A. Dreams can be instrumental in gauging a person’s emotional slate.

B. Emotions are, very often, the substance of a person’s idle thoughts.

C. It is very difficult to gauge emotional honesty

D. Dreams often, are indicative of emotions that re main unexpressed.

 

Q. 35 In saying it is tempting to wish petulantly that the unconscious would speak to us more clearly”…. (in pare 2) the author wishes to indicate that

A. the inability lo understand the unconscious can irritate psyohotherapists when pursuing their objectives.

B. the inability to understand the unconscious can guile psychotherapists in pursuit of they objectives.

C. the inability lo understand the unconscious can frustrate psychotherapists when pursuing their objectives.

D. the inability to understand the unconscious can divert psychotherapists from their objectives.

 

Questions: 36 – 38

Each of nine persons. P, 0, R, S. T. U, V, Ar and X. rives in a different flat in an aparimont building, which has six floors (excluding the ground floor, which is used only for parking) anc three flats on each floor, The three flats on each floor are in a row and no two adjacent flats on a floor are occupied. Al least one person lives on each floor.

Further the fcllow.ng information is known:

(i)  P and 0 live on the same floor.

(ii) R and S live on different floors.

(iii) T lives in the middle flat on the fourth Floor

(iv) U lives on the sixth floor and V lives on the first /1001.

(v) W Fives on the floor which is immediately above the floor on which X lives.

 

Q. 36 If W and U do not live on the same floor, then which of the following cannot be true?

A. W lives on the third floor

B. Q lives on the third floor

C. R lives on the second floor

D. P lives on the second floor.

 

Q. 37 If S and R are living on the first floor and the sixth floor respectively, then which of the following must be true’

A. T is living on the same floor as X

B. O is living on the second floor.

C. P is living on the third floor.

D. W is living alone on his floor.

 

Q. 38 If Q lives on the third floor, then how many combinations of persons could live on the second floor?

A. 8

B. 6

C. 5

D. 7

 

Q. 39 there are five sentences or parts of sentences that form a paragraph. Identity the sentence(s or part(s) of sentence(s) that is/are correct in terms of grammar and usage.

Then. choose the most appropriate option

(a) Leonardo da Vinci wed a self-taught man and began Leeching himself Latin at the early age.

(b) He became a great engineer and was the first to discover that blood circuleled through the body

(c) He believed that coarse people of bad habits and shallow judgments did not deserve so beautiful an instrument and such a complex anatomical equipment than the human body.

(d) They should merely have a sack for taking in food and letting it out again, for they are nothing but the alimirtary r.nnal

(e) Very fond of animals, he was himself a vegetarian and had the habit of buying caged birds from the market end setting thorn free immediately.

A. a and c

B. b and d

C. only a

D. only e

 

Q. 40 From among the lour choices given below the question, choose the most logical order of sentences that constructs a coherent paragraph.

(a) Although thoughts are primary. thoughts in themselves have no method of transmission and are therefore dependent on speech.

(b) If we were to summarize the logo-centric approach to meaning, we should state that what emerges is that speech rs the original signifier of meaning

(c) Language, the cornerstone of humanity, emerges as a process to allow our thoughts to travel across space and !between people

(d) Language can then be viewed as a system of verbal signs that signify individual thought.

(e) Language produces speech to transmit thoughts and writing to transmit speech.

A. cabed

B. baced

C. beard

D. caledb

 

Q. 41 The word in capitals is used in four different ways. Choose the option in which the usage of the word is INCORRECT or INAPPROPRIATE.

MELT

A. The crowd melted away after the prayer meeting.

B. Even the sternest mother’s heart melts at the sight of her baby crying.

C. His anxiety melted away when he received an SMS from his daughter confirming that she had reached her destination safely.

D. The cries of opposition suddenly melted to cheers when the principal agreed to the demands of the students.

 

Q. 42 The sentences given in the following question, when properly sequenced, form a coherent paragraph. Each sentence is labelled wilh a letter_ From among the four choices given below the question, choose the most logical order of sentences that constructs a coherent paragraph.

(a) Generally speaking, in pre-capitalist societies people produced things directly for other people, not fir sale on a market – in Marx’s Language they produced for use, not exchange.

(b) However, producing things for sale (or exchange) creates a new dynamic, different from societies That produce directly for use_

(C] Capitalism is very different from past modes of production.

(d) Under capitalism, nearly all of the products of human labor are commodities, that is, they are produced for sale,

(e) Every system of production has to regulate now much of people’s labor is spent producing one thing versus another. 5,0 that society does not expend labor on things that are useless.

(f) Marx called this -generalized commodity production’ – people obtain their needs and wants by purchasing them on a market, and people produce what other people need and want by selling thing’s. on a market.

A. dfbeac

B. abcdef

C. eacdfb

D. cdfabe

 

Questions: 43 – 45

Marcel Pious! ( 1871-1922) was immensely well read. ‘In search of Lost Time encapsulates within itself the main traditions in French literature: both in Fiction (from Madame de Lafayette through Stendhal, Balzac. Flaubert and Zola) and in the bellelettristic-philosophical line {from Montaigne through Pascal. La Rochefoucauld and Chamfort). Proust formed a strong long taste for generalization through these latter writers, I own a small book of his maxims, drawn from the novel and his discursive writings. and an unusually high quotient of them are dazzling. Let one example suffice: “It has been said that the greatest praise of God lies in the negation of the atheist, who considers creation sufficiently perfect to dispense with a creator.’ As an asthmatic child, Proust read more than most children. By the age of 15, he was already immersed in contemporary literature. having read the essays and novels of Anatole France and Pierre Lob. the poetry of Mallarme and Leconle de Lisle, and a number of the novels or Dostoyevsky, Tolsloy. Dickens and George Eliot. Linl4ce Henry James, who referred to their works as `baggy monsters,” Proust ‘Fully appreciated the great Russian novelists. He thought Tolstoy -a serene god”, valuing especially his ability to generalize in the form of setting down laws about human nature. For Proust, Dostoyevsky surpassed all other writers, and he found “The Idiot” the most beautiful novel he had ever read. He admired Dostoyesky’s skill with sudden twists in plot, providing the plausible surprises that propelled his novels. In his 1905 essay ‘On Reading,’ a key document in Proust’s freeing himself to write his great novel, he quoted Descartes: The reading of all Rudd books is tike a conversation with the most cultivated of men of past centuries who have been their authors.” Proust’s examinalion of ‘the original psychological act calted “reading.” that “noblest of distractions.” He stated that reading is superior to conversalion, which ‘dissipates immediately.” A book. he fell. is “a friendship… and the fact that it is directed to one who is absent, gives it something disinterested. almost moving.” Books are actually better than friends, Proust thought. because you turn to there only when you truly desire their company and can ignore them when you wish. neither of which is true of a friend. One also frequently loves people in books, -to whom one had given more of ones attention and tenderness thrill to people in real life.” In his own novel, Proust wrote-Real life, life at last laid bare and illuminated—the on’y life in consequence which can be said to be really lived—is literature.”

 

Q. 43 In the passage the author is primarily concerned with?

A. critically examining Proust’s in Search of Lost Time.”

B. providing a synopsis of Proust’s reading tastes

C. evaluating Proust’s position in the great literary tradition

D. discussing the intellectual influence Proust’s contemporaries had on his works.

 

Q. 44 The author quotes art example of Proust’s maxims to highlight his (Proust’s)

A. grasp at the metaphysical

B. penchant for the philosophical.

C. belief in a Supreme Being

D. exceptional choice of thought and word

 

Q. 45 The passage implies that Proust subscribes to which of the following views?

(a)Reading a good book is like having a conversation with a classical writer.

(b)Reading is a virtuous pastime and it leaves an indelible impression on one’s mind.

(c) Literature Imitates life.

(d) A reader can invest in the feelings for characters in a book.

(e) Dosloyetisky’s `The Iclibl” was appreciated by him for the unanticipated turns in the plot.

(f)) Elosloyevsky’s ‘The Idiot” was known for its aesthetics. its gapping pace and its unlikely element of surprise

A. a,c,d,e

B. a,e,f

C. a,b,c,e

D. a,b,c,d,e

 

Q. 46 Eight men have their first names. as Ratan. Rama. Ramesh. Rarnu, Ramesh, Rajah, Rishabh and Rohit and their surnames are Kulkarni, Anora, Jain, Dutta. Singh. Sharma. Sen and Murthy, not necessarily in the same order. These eight persons are sitting around a circular table as per the following instructions;

(i) Ramu is sitting opposite Kulkarni and to the left of Sing h.

(ii)Rajan is sitting opposite Sharma and next to Murthy, who is sitting to the left of Rama.

(iii) Arora is sitting opposite Ratan and Rohit sits adjacent to Dutta.

(iv)Rama, who is next to Rishabh and Ramesh, sits opposite Sen.

If Rishabh Arora sits between Kulkarni and Sharma. and opposite Singh, then who sits opposite of Rakesh Dutta?

A. Ramesh Murthy

B. Rama Murthy

C. Rajah Jain

D. Rishabh Arora

 

Questions: 47 – 49

“SOVEREIGN in tastes, steely-eyed and point-on in perception of risk, and relentless in maximization of happiness.’ This was Daniel McFadden’s memorable summation, in 20013, of the idea of Every man held by economists. That this description is unlike any real person was Mr.McFadden’s point. The Nobel prizewinning economist at the University of California, Berkeley, wryly termed homo economicus is rare species”. In his latest paper he outlines a new science of pleasure”, in which he argues that economics should draw much more heavily on Fields such as psychology, neuroscience and anthropology. He wants economists to accept that evidence from other discilplines does not just explain those bits of behavior that do not fit the standard models Rather. What economists consider anomalous is the norm, Homo economics, not his fallible counterpart, is the oddity.

To take one example, the “people” in economic models have fixed preferences which are taken as given. Vet a large body of research from cognitive psychology shows that preferences are in fad rather fluid People value mundane things much more highly when they think of them as somehow ‘Their own”: they insist on a much higher price for a coffee cup they think of as theirs, for instance. Than for an identical one that isn’t. This -endowment effect’ means that people hold on to shares well past the point where it makes sense to sell tern. Cognitive scientists have also found that people dislike losing something much more than they like gaining the same amount. Such ‘loss aversion’ can explain why people often pick insurance policies with lower deductible charges even that they are more expensive. At the moment of an accident a deductible feels like a loss, whereas all those premium payments are part of the status quo. Such tools have Implications for policy. Plenty of poor people in America are wary of programmes like the Earned Income Tax Credit (EITC) because the idea of getting a handout Irom the government reinforces a sense of helpnessness. Dignity is not something mainstream economics has much truck with. 80 creating a sense of dignity turns out to be a powerful way of affecting decisions. One study by Crystal Hall, Waying Zhao and Elder Sher, a trio of psychologists, found that getting poor people in a soup kitchen to recall a time when they felt ‘successful and proud” made them almost twice as likely to accept leaflets that told them how to get an EITC refund than members of another group who were

merely asked about the Fast meal they had eaten Taking the path Mr.McFarlden urges might also lead economists to reassess some articles of faith. Economists tend to think that more choice is good Yet people with many options sometimes fail to make any choice at all: think of workers who prefer their employers to put them by “default’ into pension plans at preset contribution rates. Explicitly modelling the process of making a choice might prompt economists to take a more ambiguous view of an abundance of choices. This is undoubtedly messier than standard economics. So is real life.

 

Q. 47 Which of the Following most accurately represents the author’s criticism of ‘Homo economicus’?

A. It gives an inaccurate picture of consumer behavior in real economic transactions

B. The ideal person who makes choices in conservative economic models is, in fact, the opposite of ‘homo economicus

C. It is nowhere close to the unpredictable consumer in real economic situations.

D. It is the economists’ mythical EveryMan.

 

Q. 48 The author of the passage mentions the observations of Hall, Zhao and Sharif in order to

A. demonstrate that people take pride in their achievements even in hard times

B. demonstrate how empathy can play a significant role in persuasion.

C. provide support for the assertion that dignity is a powerful factor in decision-making.

D. illustrate that authorities would be able to implement policies more effectively if they understand their citizens.

 

Q. 49 The view mentioned in the last paragraph (-Taking the path… real life’) refers to which of the following?

A. People are loath to make any choice when faced with a plethora of options

B. Consumers prefer to seek expert guidance while making a choice.

C. Employers coax workers lo accept pension plans with fixed contribution rates.

D. The view that more choice is good for consumers should be regarded with skepticism.

 

Q. 50 The word at the top is used in four different ways, numbered 1 to 4. Choose the option in which the usage of the word is INCORRECT or INAPPROPRIATE.

PULL

A. Pull aside the curtains and let in some fresh air

B. I decided to pull away from the venture due to differences of opinion with my partners.

C. Being a charismatic leader that he is, he can certainly pull the crowds.

D. The municipal corporation has decided to pull down all illegal constructions in the city.

 

Q. 51 The scientst and the artist are both concerned to change the world — the one the external world of man’s objective relations with nature, the other the internal world of his subjective relations with his fellow men. The scientist discovers a contradiction in his consciousness of the external world and resolves it in a scientific hypothesis: the artist discovers a contradiction in his consciousness of the internal world and resolves it in a work of art. Both are creative acts. The scientist extends our knowledge and hence also Our control of nature_

A. The artist takes complex explanations and renders them simple.

B. In doing so, he proves that there is nothing we cannot do – everything is brought within our reach.

C. The artist heightens our sense of ourselves as social beings and so advances the class struggle.

D. The artist teaches us to think for ourselves

 

Q. 52 In the following question, a part of a sentence is left blank. Choose from among the four options given below the question, the one which would best fill the blanks.

When we call others dogmatic, what we really object to is ___.

A. their giving the dog a bad name

B. their holding beliefs that are different from our own

C. the extremism that goes along with it

D. the subversion of whatever they actually believe in concomitantly

 

Questions: 53 – 55

Four friends John. Mike. Lewis and Peter went on a picnic and they participated in four adventure sports – Paragliding, Skiing, Bungee Jumping and Rock climbing Further_the following information is known about them:

The number of persons who participated in Skiing is one more than that of those who participated in Bungee Jumping, which, in turn, is same as that of those who participated in Paragliding. which. in turn, is twice that of those who participated in Rock Climbing

(ii) Every person participated in at least one event and each sport was taken up by at least one person.

(iii) John participated in Skiing but not in Rock Climbing while Lewis participated in Bungee Jumping but not in Paragliding

(iv) None of them participated in both Bungee Jumping and Rock Climbing

(v) Peter participated in three sports.

(vi) Between Skiing and Paraglidling, Mike participated in exactly one sport

 

Q. 53 If Lewis participated in two sports, which of the following is definitely false?

A. Mike did not participate in Skiing

B. John participated in Paragliding

C. Lewis participated in Skiing.

D. Mike participated in Paragliding

 

Q. 54 If John participated in Paragliding, which of the following statements is definitely true?

A. Mike participated in Rock Climbing

B. John Participated in Bungee Jumping.

C. Lewis did not participate in Skiing

D. Peter did not participate in Rock Climbing.

 

Q. 55 Which of the following is not a possible combination of number of sports taken up by John, Mike and Lewis?

A. John -.1, Mike- 2, Lewis -2

B. John – 3, Mike- 1, Lewis – 1

C. John – 1, Mike – 3, Lewis

D. John – 2, Mike – 2, Lewis – 1

 

Questions: 56 – 59

There are ten boxes. numbered 1 to 10, each containing g gold coins. Each 01 the coins in nine of these ten boxes weighs 10 gm, whereas each of the coins in the tenth box weighs 20 gm_ A digital weighing machine is provided. Now, a logician, Mr. Kapil, is invited The task assigned to him is that he has to find out the box containing the coins weighing 20 gm each.

 

Q. 56 If g = 9, then what is the minimum possible number of times for which the weighing machine is to be used?

A. 1

B. 2

C. 3

D. 4

 

Q. 57 If g = T. what is the minimum possible number of times For which the weighing machine is to be used’?

A. 1

B. 2

C. 3

D. 4

 

Q. 58 If g = 3. what is the minimum possible number of times For which the weighing machine is to be used?

A. 3

B. 2

C. 5

D. 4

 

Q. 59 If g = 2. what is the minimum possible number of times for which the weighing machine is to be used?

A. 2

B. 3

C. 5

D. 4

 

Q. 60 The following question presents four statements, of which three, when places in appropriate order, would form a contextually complete paragraph. Pick the statement that is not part of the context

(A) The trade in iron ore make it the second-largest commodity market by value after crude oil

(B) The metal provides the backbone of skyscrapers, bridges and motorways, and the carapace and internal organs of cars, fridges and washing machines

(C) Given steel’s ubiquity-it makes up 95% of global metal production-iron ore, the raw material from which it is made, attracts strangely little attention

(D) The development of a process to turn raw earth into steel merits a high spot on a list of mankind’s most ingenious achievements

A. (A)

B. (B)

C. (C)

D. (D)

 

Answer Sheet
Question 1 2 3 4 5 6 7 8 9 10
Answer D A C C B B A C B A
Question 11 12 13 14 15 16 17 18 19 20
Answer A B C A D B C D B A
Question 21 22 23 24 25 26 27 28 29 30
Answer C D C C C B B D A A
Question 31 32 33 34 35 36 37 38 39 40
Answer C A B D C D D C B B
Question 41 42 43 44 45 46 47 48 49 50
Answer D C B C C A C C D B
Question 51 52 53 54 55 56 57 58 59 60
Answer C B B D D A B B B A

CAT Previous Year Paper 2012

CAT 2012

Section

Questions

Marks

Quantitative Aptitude

27 Questions

66

DILR

14 Questions

57

English

19 Questions

57

Q. 1 Consider a sequence S whose nth Tern is defined as 1+3/n, where n = 1, 2,… Find the product of all the consecutive terms of S starting from the 4th term to the 60th term

A. 1980.55

B. 1985.55

C. 1990.55

D. 1975.55

 

Q. 2 Let P = {2, 3, 4,….100} and q = {101, 102, 103,…200}. How many elements of Q are there such that they do not have any element of P as a factor?

A. 20

B. 24

C. 23

D. 21

 

Q. 3 What is the sum of all the 2-digit numbers which leave a remainder of 6 when divided by 8?

A. 612

B. 594

C. 324

D. 872

 

Q. 4 Which of the terms 2¹/³, 3¹/⁴, 4¹/⁶, 6¹/⁸ and 10¹/¹² is the largest?

A. 2¹/³

B. 3¹/⁴

C. 4¹/⁶

D. 10¹/¹²

 

Q. 5 If the roots of the equation (a²+b²)x²+2(b²+c²)x+(b²+c²)=0 are real, which of the following must hold true?

A. c² ≥ a²

B. c⁴ ≥ a² (b²+c²)

C. b² ≥ a²

D. a⁴ ≤ b² (a²+c²)

 

Q. 6 Find the remainder of 2¹⁰⁴⁰ divided by 131.

A. 1

B. 3

C. 5

D. 7

 

Q. 7 In the figure below, ∠MON =∠MPO = ∠NQO = 90° and OQ is the bisector of ∠MON and QN = 10, OR=40/7. Find OP.

A. 4.8

B. 4.5

C. 4

D. 5

 

Q. 8 If (a²+b²),(b²+c²) and (a²+c²) are in geometric progression, which of the following holds true?

A. b²-c²=a⁴-c⁴/b²+a²

B. b²-a²=a⁴-c⁴/b²+c²

C. b²-c²=b⁴-a⁴/b²+a²

D. b²-a²=b⁴-c⁴/b²+a²

 

Q. 9 p is a prime and m is a positive integer. How many solutions exist for the equation p⁶-p=(m² +m+6)(p-1)?

A. 0

B. 1

C. 2

D. Infinite

 

Q. 10 A certain number written in a certain base is 144. Which of the following is always true?

I. Square root of the number written in the same base is 12.

II. If base is increased by 2, the number becomes 100.

A. Only I

B. Only II

C. Neither I nor II

D. Both I and II

 

Q. 11 A rectangle is drawn such that none of its sides has length greater than ‘a’. All lengths less than ‘a’ are equally likely. The chance that the rectangle has its diagonal greater than ‘a’ is (in terms of %)

A. 29.3%

B. 21.5%

C. 66.66%

D. 33.33%

 

Q. 12 If x is a real number, [x] is greatest integer less than or equal to x, then 3[x]+2-[x]=0/ Will the above equation have any real root?

A. Yes

B. No

C. Will have real roots for x<0

D. Will have real roots for x>0

 

Q. 13 If a=x/y+z, b=y/z+y, c=z/x+y, then which of the following statements is/are true?

I. b+c-1/yz + a+c-1/xzz + a+b-1/yx = 1

II. x²/a(1-bc) = y²/b(1-ca) = z²/c(1-ab)

III. (a+b)c+(b+c)a+(a+c)b = {2(x+y+z)(xy+xz+yz)-6xyz}/(x+y)(y+z)(z+x)

A. I and II

B. I and III

C. II and III

D. None of these

 

Q. 14 If α and β are the roots of the quadratic equation x²-10x+15=0, then find the quadratic equation whose roots are (α+α/β) and (β+β/α)

A. 15x²+71x+210=0

B. 5x²-22x+56=0

C. 3x²-44x+78=0

D. Cannot be determined

 

Questions: 15 – 17

Directions (Q. Nos. 15-17)

Read the information carefully and answer the questions that follow.

A cricket tournament had three teams – India, Australia and Sri Lanka taking part in it. The format of the tournament was such that in the preliminary stage each of these teams, would play the other teams four times. Four points are awarded for a win and in case a team beats another team by a huge margin, it is given a bonus point in addition to the four points. At the end of the preliminary stage, the top two teams, in terms of points scored, reaches the finals. No match in the tournament ends in a tie and if two teams end up with the same number of points at the end of the preliminary stage, the team with the better net run rate is placed higher.

 

Q. 15 If India reached the finals, then what is the minimum numbers of points it would have scored in the preliminary stage?

A. 8

B. 10

C. 12

D. 16

 

Q. 16 If Sri Lanka was eliminated in the preliminary stage, then what is the maximum number of points it could have scored?

A. 12

B. 14

C. 16

D. 20

 

Q. 17 If Australia had the highest number of points at the end of the preliminary stage, then at least how many points did it have?

A. 16

B. 17

C. 18

D. 20

 

Q. 18 A vessel has a milk station in which milk and water are in the ratio 4:1. By addition of water to it, milk solution with milk and water in the ratio 4:3 was formed. On replacing 14 L of this solution with pure milk the ratio of milk and water changed to 5:3. What is the volume of the water added?

A. 12 L

B. 60 L

C. 32 L

D. 24 L

 

Q. 19 A car 􀀁 starts from a point P towards another point Q. Another car 􀀂 starts (also from P) 1 h after the first car and overtakes it after covering 30% of the distance PQ. After that, the cars continue. On reaching Q, car 􀀂 reverses and meets car 􀀁, after covering 23+1/3 of the distance QP. Find the time taken by car 􀀂 to cover the distance PQ (in hours).

A. 3

B. 4

C. 5

D. 3+1/3

 

Q. 20 A, B and C can independently do a work in 15 days, 20 days and 30 days, respectively. They work together for some time after which C leaves. A total of ₹18000 is paid for the work and B gets ₹6000 more than C. Find how many days did A work?

A. 2

B. 4

C. 6

D. 8

 

Q. 21 In the figure given, OABC is a parallelogram. The area of the parallelogram is 21 sq units and the point C lies on the line x=3. Find the coordinates of B.

A. (3, 10)

B. (10, 3)

C. (10, 10)

D. (8, 3)

 

Q. 22 Find the complete set of values that satisfy the relations ||x|-3|<2 and ||x|-2|<3.

A. (-5, 5)

B. (-5, -1) ∪ (1, 5)

C. (1, 5)

D. (-1, 1)

 

Q. 23 If ax²+bx+c=0 and 2a, b and 2c are in arithmetic progression, which of the following are roots of the equation?

A. a, c

B. -a, -c

C. -a/2, -c/2

D. -c/a, -1

 

Q. 24 A solid sphere of radius 12 inches is melted and cast into a right circular cone whose base diameter is √2 times its slant height. If the radius of the sphere and the cone are the same, how many such cone are the same, how many such cones can be made and how much material is left out?

A. 4 and 1 cubic inch

B. 3 and 12 cubic inches

C. 4 and 0 cubic inch

D. 3 and 6 cubic inches

 

Q. 25 If logₓ(a-b)-logₓ(a+b)=logₓ(b/a), find a²/b² + b²/a²

A. 4

B. 2

C. 3

D. 6

 

Q. 26 Letters of the word “ATTRACT” are written on cards and are kept on a table. Manish is asked to lift three cards at a time, write all possible combinations of the three letters on a piece of paper and then replace the three cards. The exercise ends when all possible combinations of letters are exhausted. Then, he is asked to strike out all words in his list, which look the same when seen in a mirror. How many words is he left with?

A. 40

B. 20

C. 30

D. None of these

 

Q. 27 S is a set given by S={1, 2, 3,….4n}, where n is a natural number. S is partitioned into n disjoint subsets A₁, A₂, A₃,…An each containing four elements. It is given that in everyone of these subsets there is one element, which is the arithmetic mean of the other three elements of the subsets. Which of the following statements is then true?

A. n≠1 and n≠2

B. n≠1 but can be equal to 2

C. n≠2 but can be equal to 1

D. it is possible to satisfy the requirement for n =1 as well as for n = 2

 

Q. 28 When asked for his taxi number, the driver replied, “If you divide the number of my taxi by 2, 3, 4, 5, 6 each time you will find a remainder of one. But if, you divide it by 11, the remainder is zero. You will also not find any other driver with a taxi having a lower number who can say the same.” What is the taxi number?

A. 121

B. 1001

C. 1881

D. 781

 

Q. 29 A student is asked to form numbers between 3000 and 9000 with digits 2, 3, 5, 7 and 9. If no digit is to be repeated, in how many ways can the student do so?

A. 24

B. 120

C. 60

D. 72

 

Q. 30 The side of an equilateral triangle is 10 cm long. By drawing parallels to all its sides, the distance between any two parallel lines being the same. The triangle is divided into smaller equilateral triangle, each of which has sides of length 1 cm. How many such small triangles are formed?

A. 60

B. 90

C. 120

D. None of these

 

Questions: 31 – 32

Directions (Q. Nos. 31-32)

Each of the following questions has a paragraph from which a sentence has been deleted. From the given options, choose the one that completes the paragraph in the most appropriate way.

 

Q. 31 RD Laing developed a broad range of thought on interpersonal psychology. This deals with interactions between people, which he considered important, for an ethical action always occurs between one person and another. In books, such as The Politics of Experience, he deal with issues concerning how we should relate to persons labelled by the psychiatric establishment as “schizophrenic”.

A. He came to be seen as a champion for the rights of those considered mentally ill

B. He spoke out against (and wrote about) practices of psychiatrists which he considered inhumane or barbaric, such as electric shock treatment

C. Laing also did work in establishing true asylums as places of refuge for those who feel disturbed and want a safe place to go through whatever it is they want to explore in themselves and with others

D. He suggested that the effects of psychiatric drugs (some of which are very deleterious, such as tardive diskensia) be called just that: “effects” and not be referred to by the preferred euphesisms of the drug companies, who prefer to call them “side effects”

 

Q. 32 Jurisprudence is the theory and philosophy of law. Students of jurisprudence aim to understand fundamental nature of law and to analyse its purpose, structure and application. Jurisprudential scholars (sometimes confusingly referred to as “jurists”) hope to obtain a deeper understanding of the law, the kind of power that it exercises and its role in human societies.

They seek a deeper understanding behind law’s seemingly unpredictable and certain nature.

A. At a practical level, some jurists hope to improve society by studying what the Law is, what it ought to be, and how it actually operates

B. A common starting point in understanding jurisprudence is the objective of law to achieve justice

C. Hence, the arguable scientific nature of jurisprudence

D. Jurisprudence seeks to draw on unrestricted elements of life and the world to aid the critical study of law

 

Questions: 33 – 35

Directions (Q. Nos. 33-35)

Read the following passage carefully and answer the questions based on that.

Human reason, in one sphere of its cognition, is called upon to consider questions, which it cannot decline, as they are presented by its own nature but which it cannot answer, as they transcend every faculty of the mind. It falls into this difficulty without any fault of its own. It begins with principles which cannot be dispensed within the field of experience and the truth and sufficiency of which are, at the same time insured by experience, With these principles it rises, in obedience to the laws of its own nature, to even higher and more remote conditions. But it quickly discovers that in this way, its labours must remain ever incomplete because new questions never cease to present themselves and thus it finds itself compelled to have recourse to principled which transcend the region of experience while they are regarded by common sense without distrust. It thus falls into confusion and contradictions from which it conjectures the presence of latent errors, which however, it is unable to discover because the principles it employs transcending the limits of experience cannot be tested by that criterion. The arena of these endless contests is called Metaphysic. Time, when she was the queen of all the sciences and if we take the will for the deed, she certainly deserves, so far as regards the high importance of her objectmatter, this title of honour.

 

Q. 33 According to the author, ‘Metaphysic’ is best defined when human reason

A. Becomes conscious of the presence of latent errors

B. Solves pending old questions, tackles new ones that arise

C. Employs principles that transcend the limits of experience

D. Rises to higher and more remote conditions

 

Q. 34 If there were a paragraph succeeding the last, it would probably be about

A. The rise of Metaphysic into the realm of popular acclaim

B. Metaphysic as the final solution to human misery

C. The modern day contempt for metaphysical reasoning

D. The subjugation of science by a transcendental human consciousness

 

Q. 35 The passage provides an answer to which of the following questions?

A. How does experience limit the human mind’s recourse to principles in combating new questions that present themselves?

B. Why does human reason restrain its forays to within its known limitations?

C. How does the human mind attempt to resolve problems beyond its scope?

D. None of the above

 

Q. 36 Find the incorrect usage of the word in the following questions.

The word is SLAM –

A. I heard the door slam behind him.

B. She slammed down the phone angrily.

C. She slammed his face hard.

D. The ear skidded and slammed into a tree.

 

Q. 37 Find the incorrect usage of the word in the following questions.

The word is STOP –

A. The car stopped at the traffic light.

B. Shantaram immediately stopped what he was doing.

C. We need more laws to stop pollution.

D. He is stopped by law from holding a license.

 

Q. 38 Find the incorrect usage of the word in the following questions.

The word is TIME –

A. I can remember very few times when we had to cancel due to ill health.

B. This is the first time that I have been to London.

C. The train arrived right on time.

D. The changing seasons mark the passing of time.

 

Q. 39 Find the incorrect usage of the word in the following questions.

The word is SOMBRE –

A. He was dressed in sombre shades of grey and black.

B. Paul was in a sombre mood.

C. The year ended on a sombre note.

D. He is in the sombre position of not having to worry about money.

 

Q. 40 Find the incorrect usage of the word in the following questions.

The word is CALL –

A. She payed him a call from the pay phone near her home.

B. I will call on you tomorrow evening at 7 pm.

C. Vikram decided to call a meeting to discuss the trade fair.

D. She felt the call of religion early in her life.

 

Questions: 41 – 42

Read the following passage carefully and answer the questions that follow:

It is essential to rid ourselves of the false impressions of time, which our human limitations seem to impose upon us. Above all, we must rid ourselves of the belief that the future is in some way less determined than the past, if the borderline between past and future is illusory, then so must be the distinction between two regions of time which it is supposed to separate. The only reason we believe the future to be still undecided while the past is immutable is that we can remember the one and not the other. To avoid these prejudices we must picture the history of the universe not as a three-dimensional stage on which things change but as a static four-dimensional space time structure of which we are a part. We believe that events are not real until they “happen”, whereas in reality past, present and future are all frozen in the four dimensions of space time. Unfortunately even if all this is accepted, we have to continue using the language of a “moving” time, for we have no other but we must try to interpret this language always as a description of the unchanging space time structure of the universe. Contemplating the history of the universe in this way, it is attractive to believe that the periods of expansion and contraction could be related to each other by symmetry. Both points of view merit serious consideration and that we cannot say with any certainty that the contracting universe will or will not, differ fundamentally from the expanding phase that we observe today.

 

Q. 41 According to the author of the passage,

A. The time value called ‘now’ is most essential to the understanding of the universe

B. The impression of a moving time is not a false imposition of human limitation

C. There is noting with respect to which time could move

D. The future is better determined than the past

 

Q. 42 Which of the following best exemplifies the author’s attitude to time?

A. The impression of a moving time is false

B. ‘Now’ is a purely subjective phenomenon existing only within the human mind

C. The future is not in anyway less determined than the past

D. All of the above

 

Questions: 43 – 47

Read the passage and answer the questions that follow:

Amidst the increasing clamour for a discourse on educational improvement, on budgetary allocations and retention rates, there is one crucial question which is insufficiently discussed. And the question is this what is the purpose of education today? At various times, over the past 100 yr, that question has been answered differently – in colonial India, the official answer would have been, “to create a cadre of clerks and officials to run the colonial state”, while in a newly decolonized India, the official answer could be, “to create a nationalist sensibility and the national citizen.” Today, I suspect the official answer to the question about the purpose of education would be, “to give people jobs”. Increasingly, the emphasis in education is towards vocationalisation and skills development. In a recent private conversation, the Education Minister of a North Indian state said, “we have a lot of jobs. We just don’t have the people skilled enough to do them. We need bio technologists, fitters, crane operators, nurses and lab assistants. But our education does not prepare young people for what we need. We need to change that.” Similarly, we find that the Confederation of Indian Industry is showing increasing interest in school education. The CII recently commissioned a study to look at the challenges and opportunities which face the Indian industry and this is its thesis that in the year 2025, there will be about 40 million jobs worldwide, which need to be filled. India will be one of the few countries in the world to have a labour surplus of the right age group. It, therefore believes that we need to think about the kinds of education system necessary to develop skills whereby our children will be best equipped to function in this scenario. Public consensus on the way to improve educational access is increasingly moving towards a public-private partnership. But we must be concerned about the terrible narrowness of the vision for educational improvement which characterizes our discourse. Education, in this picture, is about the implanting of useful skills – the assumption being that it will ultimately lead to both personal and national enrichment but as Martha Nussbaum writes, education is not simply a producer of wealth; it is a producer of citizens. Citizens in a democracy need, above all, freedom of mind – to learn to ask searching questions; to reject shoddy historical argument; to imagine alternative possibilities from a globalizing, service and market-driven economy, to think what it might be like to be in others’ shoes. Recently, the Israeli novelist, Amos Oz, spoke about the importance of reading novels as what he calls an antidote to hate. He said, “I believe in literature as a bridge between peoples. I believe curiosity can be a moral quality. I believe imagining the other can be an antidote to fanaticism. Imagining the other will make you not only a better businessperson or a better lover but even a better person. Part of the tragedy between Jew and Arab is the inability of so many of us, Jews and Arabs, to imagine each other – really imagine each other; the loves, the terrible fears, the anger, the passion. There is too much hostility between us, too little curiosity.” The skills and thought processes while engender the curiosity, the imagining are associated with the humanities, the arts and literature and despite the splendid interventions in the NCERT’s new textbooks for History and Political Science, these areas are terribly neglected. Our dominant conception of worthwhile education is increasingly technical and mechanistic. The thinking processes engendered by the social sciences are today seen as quaint, vaguely leftyintellectual, a kind of quixotic idealism – which has very little to do with the real business of life. It is a strange irony that in the educational world of Gandhi, Tagore and Aurobindo, there are tragically few voices which assert a more holistic vision.

 

Q. 43 The true purpose of education in India as inferred from the passage ?

A. Is to create a nationalist sensibility in every citizen

B. Has been a topic of debate since independence

C. Is a concept that has been changing from time to time

D. Is to teach an individual the necessary skills to earn his livelihood

 

Q. 44 In the author’s perception, our vision for educational improvement is narrow because our system

A. Gives importance only to vocationalisation and skills development

B. Believes in making people earn more so that they can stand up to the challenges of a globalizing economy

C. Does not acknowledge the importance of humanist concepts

D. Does not support a public-private partnership in improving educational access to everyone

 

Q. 45 Amos Oz believes that the world will become a peaceful place, if people

A. Become less hostile

B. Become less narrow minded

C. Become less fanatic

D. Empathise with each other

 

Q. 46 The Indian concept of worthwhile education is that which

I. Gives technical training

II. Makes people think

III. Has a measurable outcome

IV. Kindles our curiosity and imagination

V. Helps people become wealthy

A. I and V

B. II and III

C. I, II and IV

D. I, III and IV

 

Q. 47 Which of the following is not an attribute of a good citizen in a democracy?

A. Learning to ask searching questions

B. Not accepting inadequate reasons from history

C. Thinking out of the box

D. Learning to negotiate with people

 

Questions: 48 – 51

Directions (Q. Nos. 48-51)

Read the following information carefully and answer the questions based on that.

Two teams of five each must be selected from a group of ten persons – A through J – of which A, E and G are doctors; D, H and J are lawyers; B and I are engineers; C and F are managers. It is also known that

i) Every team must contain persons of each of the four professions

ii) C and H cannot be selected together

iii) I cannot be selected into a team with two lawyers

iv) J cannot be in a team with two doctors

v) A and D cannot be selected together

 

Q. 48 If C and G are in different teams, then who are the other team members of A?

A. C, D, E and I

B. B, F, I, and J

C. B, C, H and J

D. F, H, I and G

 

Q. 49 Who among the following cannot be in the same team as I?

A. H

B. J

C. C

D. F

 

Q. 50 Who among the following must always be in the same team as A?

A. D

B. B

C. H

D. J

 

Q. 51 If F and G are in the same team, which among the following statements is true?

A. B and H will be in the other team

B. E and I must be in the same team

C. H must be in the same team but B must be in the other team

D. C must be in the other team but D must be in the same team

 

Questions: 52 – 55

Directions (Q. Nos. 52-55)

Read the following information carefully and answer the questions based on that.

Two families are planning to go on a canoe trip together. The families consists of the following people : Robert and Mary Henderson and their three sons Tommy, Don and William, Jerome and Ellen Penick and their two daughters Kate and Susan. There will be three canoes with three people in each canoe. At least one of the four parents must be in each canoe. At least one person from each family must be in each canoe.

 

Q. 52 If the two mothers ride together in the same canoe and the three brothers each ride in a different canoe, which of the following must be true?

A. Each canoe has both males and females in it

B. One of the canoes has only females in it

C. One of the canoes has only males in it

D. The sisters ride in the same canoe

 

Q. 53 If Ellen and Susan are together in one of the canoes, which of the following could be a list of the people together in another canoe?

A. Dan, Jerome, Kate

B. Dan, Jerome, William

C. Dan, Kate, Tommy

D. Jerome, Kate, Mary

 

Q. 54 If Jerome and Mary are together in one of the canoes, each of the following could be a list of the people together in another canoe except

A. Dan, Ellen, Susan

B. Ellen, Robert, Tommy

C. Ellen, Susan, William

D. Ellen, Tommy, William

 

Q. 55 If each of the Henderson children rides in a different canoe, which of the following must be true?

I. The Penick children do not ride together

II. The Penick parents do not ride together

III. The Henderson parents do not ride together

A. Only I

B. Only II

C. I and II

D. I and III

 

Questions: 56 – 57

Directions (Q. Nos. 56-57)

Each question has a sentence with a part of the sentence quoted that may contain an error. Four alternative substitutes are given for the quoted portion. Identify the choice that replaces the quoted part to form a logically and grammatically correct statement and mark it as your correct answer.

 

Q. 56 Feminism is not simply a movement to ensure that women will have equal rights with men “but that a commitment for eradiating the ideology of domination” that permeates Indian culture at various levels

A. That is a commitment to eradicate the ideology to dominate that

B. It is a commitment to eradicating the ideology of domination that

C. Whose commitment to eradicating the ideology of domination which

D. But that a commitment for eradicating the ideology of domination that

 

Q. 57 It is unfortunate that the lure of visiting foreign countries still draws a very large number if our people, “who do not seem to be realizing what their own country is and how much can be seen and learnt from it?”

A. Who do not seem to realize their country and see and learn from it

B. Who are not realizing what their own country is and how much there is in it to see and learn from

C. Who do not seem to realize what their own country is and how much there is in it to see and learn from it

D. Who do not seem to realize what their own country is and how much there is in it to see and learn from

 

Questions: 58 – 60

Directions (Q. Nos. 58-60)

Read the following information carefully and answer the questions based on that.

Each of five people – A, B, C, D and E owns a different car among Maruti, Mercedes, Sierra, Fiat and Audi and the colours of these cars are Black, Green, Blue, White and Red, not necessarily in that order. No two cars are of the same colour. It is also known that

i) A’s car is not Black and it is not a Mercedes

ii) B’s car is Green and it is not a Sierra

iii) E’s car is not White and it is not an Audi

iv) C’s car is a Mercedes and it is not Blue

v) D’s car is not Red and it is a Fiat

 

Q. 58 If A owns a Blue Sierra, then E’s car can be a

A. Red Maruti

B. White Maruti

C. Black Audi

D. Red Audi

 

Q. 59 If A owns a White Audi, then E’s car can be a

A. Red Maruti

B. Blue Maruti

C. Green Audi

D. Black Sierra

 

Q. 60 If A’s car is a Red Maruti and D’s car is White, then E owns a

A. Black Audi

B. Blue Sierra

C. Black Sierra

D. Blue Audi

 

Answer Sheet
Question 1 2 3 4 5 6 7 8 9 10
Answer B D B D A A C B B D
Question 11 12 13 14 15 16 17 18 19 20
Answer B B A C A D B C D D
Question 21 22 23 24 25 26 27 28 29 30
Answer B B D C D A B A D D
Question 31 32 33 34 35 36 37 38 39 40
Answer A A B C C C D A D A
Question 41 42 43 44 45 46 47 48 49 50
Answer C D C A D A D D B C
Question 51 52 53 54 55 56 57 58 59 60
Answer C A B B D B D A D B

CAT Previous Year Paper 2010

CAT 2010

Section

Questions

Marks

DILR

20 Questions (1 – 20)

60

Quantitative Aptitude

20 Questions (21 – 40)

60

English

20 Questions (41 – 60)

60

Q. 1 – 3 Read the graph carefully and answer the questions that follow.

One of the graph below represent the market share of five different brands of bikes sold in a particular city. The total number of bikes sold was 25000. The other one shows the unit sales price and the tax the brand is subjected to. Tax is calculated on the sales price.

 

Q. 1 What is the difference in number of units sold by the most popular brand and the second most popular brand?

A. 800

B. 1000

C. 1200

D. 1400

 

Q. 2 Which brand earned the maximum revenue before tax?

A. A

B. B

C. C

D. D

 

Q. 3 Which brand earned the maximum revenue after tax?

A. A

B. B

C. C

D. D

 

Q. 4 Whenever Amit sings, Bharat gets a headache and Chandu complains. If Chandu is not complaining, which of the following statements must be true?

A. Amit is singing and bharat has a headache

B. Bharat has a headache but Amit is not necessarily singing.

C. Amit is singing, but Bharat does not necessarily have a headache.

D. Amit is not singing.

 

Q. 5 Atul will eat the apple if Bhanu does not cook. Based only on the information above, which of the following must be true?

A. Atul will not eat the apple if Bhanu cooks

B. If Atul did not eat the apple, Bhanu did cook

C. If Atul eats the apple, then Bhanu did not cook

D. If Bhanu does not cook, Atul will not eat the apple

 

Questions: 6 – 8

Answer the questions on the basis of the information given below.

Table A below provides data about ages of children in a school. For the age given in the first column, the second column gives the number of children not exceeding that age. For example, first entry indicates that there are 9 children aged 4 years or less. Tables B and C provide data on the heights and weights respectively of the same group of children in a similar format. Assuming that an older child is always taller and weighs more than a younger child.

 

Q. 6 What is the number of children of age 9 years or less whose height does not exceed 135 cm?

A. 48

B. 45

C. 3

D. Cannot be determined

 

Q. 7 How many children of age more than 10 years are taller than 150 cm and do not weigh more than 48 kg?

A. 16

B. 40

C. 9

D. Cannot be determined

 

Q. 8 Among the children older than 6 years but not exceeding 12 yeas, how many weigh more than 38 kg?

A. 34

B. 52

C. 44

D. Cannot be determined

 

Questions: 9 – 12

Answer the questions on the basis of the information given below.

A study was conducted to ascertain the relative importance that employees in five different countries assigned to five different traits in their chief executive officers. The traits were compassion (C), decisiveness (D), negotiation skills (N), public visibility (P) and vision (V). The level of dissimilarity between two countries to any of the five traits. The following table indicates the rank order of the five traits. The following table indicates the rank order of the five traits for each country.

 

Q. 9 Which of the following countries is least dissimilar to India?

A. China

B. Japan

C. Malaysia

D. Thailand

 

Q. 10 Which among the following countries is most dissimilar to India?

A. China

B. Japan

C. Malaysia

D. Thailand

 

Q. 11 Which of the following pairs of countries are most dissimilar?

A. China and Japan

B. India and China

C. Malaysia and Japan

D. Thailand and Japan

 

Q. 12 Three of the following four pairs of countries have identical levels of dissimilarity. Which pair is the odd one out?

A. Malaysia and China

B. China and Thailand

C. Thailand and Japan

D. Japan and Malaysia

 

Questions: 13 – 15

Answer the questions on the basis of the information given below.

Spam that enters our electronic mailboxes can be classified under several spam heads. The following table shows the distribution of such spam worldwide over time. The total number of spam e-mails received during December 2002 was larger than the number received in June 2003. The total number of spam e-mails received during September 2002 was larger than the number received in March 2003. The figures in the table represent the percentage of all spam e-mails received during that period, failing into those respective categories.

 

Q. 13 In which category was the percentage of spam e-mails increasing but at a decreasing rate?

A. Financial

B. Scams

C. Products

D. None of these

 

Q. 14 In the health category, the number of spam e-mails received in December 2002 as compared to June 2003 was

A. Larger

B. Smaller

C. Equal

D. cannot be determined

 

Q. 15 In the financial category, the number of spam e-mails received in September 2002 as compared to March 2003 was

A. Larger

B. Smaller

C. equal

D. cannot be determined

 

Questions: 16 – 20

Refer to the following graph and answer the questions.

 

Q. 16 Which month has the highest profit per employee?

A. September

B. July

C. January

D. March

 

Q. 17 In which month is die percentage increase in sales over the sales two months before, the highest?

A. March

B. September

C. July

D. May

 

Q. 18 In which month is the total increase in the cost highest as compared to the cost two months ago?

A. March

B. September

C. July

D. May

 

Q. 19 Assuming that no employee left the job, how many more people did the company take on in the given period?

A. 4600

B. 5100

C. 5800

D. 6400

 

Q. 20 In which month is the total increase in the cost highest as compared to the cost two months ago?

A. March

B. May

C. September

D. July

 

Q. 21 The largest number amongst the following that will perfectly divide 101¹⁰⁰ – 1 is

A. 100

B. 10000

C. 100¹⁰⁰

D. 100000

 

Question:- 22

Rohan and Sohan take a vacation at their grandparents house. During the vacation, they do any activity together. They either played Tennis in the evening or practiced Yoga in the morning, ensuring that they do not undertake both the activities on any single day. There were some days when they did nothing. Out of the days that they stayed at their grandparents’ house, they involved in one of the two activities on 22 days.However, their grandmother while sending an end of vacation report to their parents stated that they did not do anything on 24 mornings and they did nothing on 12 evenings.

 

Q. 22 How long was their vacation?

A. 36 days

B. 14 days

C. 29 days

D. cannot be determined

 

Q. 23 The equation, 2x² + 2(p + 1)x + p = 0, where p is real, always has roots that are

A. Equal

B. Equal in magnitude but opposite in sign

C. Irrational

D. real

 

Q. 24 Ram Kumar buys every year Bank’s cash certificates of value exceeding the last year’s purchase by 300. After 20 years, he finds that the total value of the certificates purchased by him is 83000. Find the value of the certificates purchased by him in the 13th year.

A. Rs. 4900

B. Rs. 6900

C. Rs. 1300

D. none of these

 

Q. 25 Ram starts working on a job and works on it for 12 days and completes 40% of the work_ To help him complete the work, he employs Ravi and together they work for another 12 days and the work gets completed. How much more efficient is Ram than Ravi?

A. 50%

B. 200%

C. 60%

D. 100%

 

Q. 26 A pump can be used either to fill or to empty a tank. The capacity of the tank is 3600 m³. The emptying capacity of the pump is 10 m³/min higher than its filling capacity. What is the emptying capacity of the pump, if pump needs 12 more minutes to fill the tank than to empty it?

A. 50 m³/min

B. 60 m³/min

C. 45 m³/min

D. 90 m³/min

 

Q. 27 How many integers, greater than 999 but not greater than 4000, can be formed with the digits 0, 1, 2, 3 and 4, if repetition of digits is allowed?

A. 499

B. 500

C. 375

D. 376

 

Q. 28 How many five digit positive integers that are divisible by 3 can be formed using the digits 0, 1, 2, 3, 4 and 5, without any of the digits getting repeating?

A. 15

B. 96

C. 216

D. 120

 

Q. 29 There are four boxes. Each box contains two balls: one red and one blue. You draw one ball from each of the four boxes. What is the probability of drawing at least one red ball?

A. 1/2

B. 1/4

C. 1/16

D. 15/16

 

Q. 30 Which of the following statements is not correct?

A. log₁₀10 = 1

B. log (2 + 3) = log (2 x 3)

C. log₁₀1 = 0

D. log (1 + 2 + 3) = log 1 + log 2 + log 3

 

Q. 31 The sum of the number of factors of the number N and N² is 34. How many such distinct numbers N < 150 exist?

A. 6

B. 2

C. 4

D. 3

 

Q. 32 The sum of 3rd and 15th elements of an arithmetic progression is equal to the sum of 6th, 11th and 13th elements of the same progression. Then which element of the series should necessarily be equal to zero?

A. 1st

B. 9th

C. 12th

D. none of these

 

Q. 33 The function f(x) – lX – 2 l+l 2.5 – x l + l3.6 – xl , where x is a real number, attains a minimum at

A. x = 2.3

B. x = 2.5

C. x = 2.7

D. none of these

 

Q. 34 The remainder, when (1523 + 2323 ) is divided by 19, is

A. 4

B. 15

C. 0

D. 18

 

Q. 35 In a race of 200 m, A beats S by 20 m and N by 40 m. If S and N are running a race of 100 m with exactly the same speed as before, then by how many meters will S heat N?

A. 11.11 m

B. 10 m

C. 12 m

D. 25 m

 

Questions: 36 – 37

These questions are based on the given information

There are three different cable channels namely Ahead, Luck and Bang. In a survey, it was found that 85% of viewers respond to Bang, 20% to Luck, and 30% to Ahead. 20% of viewers respond to exactly two channels and 5% to none.

 

Q. 36 What percentage of the viewers responded to all three?

A. 10

B. 12

C. 14

D. none of these

 

Q. 37 Assuming 20% respond to Ahead and Bang, and 16% respond to Bang and Luck, what is the percentage of viewers who watch only Luck?

A. 20

B. 10

C. 16

D. none of these

 

Q. 38 A milkman mixes 20 L of water with 80 L of milk. After selling one-fourth of this mixture, he adds water to replenish the quantity that he has sold. What is the current proportion of water to milk?

A. 2:3

B. 1:2

C. 1:3

D. 3:4

 

Q. 39 Let u = (log₂ x) 2 – 6 log₂ , x + 12 where, x is a real number. Then the equation x^u = 256, has

A. no solution for x

B. exactly one solution for x

C. exactly two distinct solutions for x

D. exactly three distinct solutions for x

 

Q. 40 A positive whole number M less than 100 is represented in base 2 notation, base 3 notation, and base 5 notation. It is found that in all three cases the last digit is 1, while in exactly two out of the three cases the leading digit is 1. Then M equals

A. 31

B. 63

C. 75

D. 91

 

Questions: 41 – 43

The passage given below is followed by a set of three questions. Choose the most appropriate answer to each question.

Bruce Robbins’s excellent article points up the paradox of cosmopolitanism – that it seems ‘perpetually torn between an empirical dimension and a normative dimension’. For Robbins, the paradox of cosmopolitanism is rooted in the limited empirical sense of political community. For genuine democracy people need to belong to the same community of fate’, and there is at present little evidence of such a sense of cosmopolitan consciousness. Although leading (Western) governments make claims in support of cosmopolitan human rights established by virtue of membership of a common humanity, their practice is often limited by the ‘communitarian’ reality. The lack of ‘shared fate’ leads to inequalities in practice as governments are often reluctant to sacrifice either treasury resources or military lives in the cause of others, and citizens appear unwilling to shoulder the tax burdens involved in any potential cosmopolitan redistribution of wealth and opportunities. Robbins suggests that it would be wrong to use the empirical limits to cosmopolitan practices as an argument against normative cosmopolitan claims. He asserts that there is ‘no possibility of simply choosing the actual over the normative’ and instead suggests that we should accept that the ‘contradiction’ exists.

A solution to the problem lies in political change which seeks ‘to bring abstraction and actuality together’. A ‘Left cosmopolitanism’ is one that denies ‘the past authority over the present’ – the empirical reality that ‘there is as yet little evidence of transnational solidarity’ should be the justification for engagement and struggle on the side of the progressive cosmopolitan cause. This campaigning perspective is advocated by several cosmopolitan theorists who, in different ways, seek to develop ideas and mechanisms whereby global civil society can encourage and further cosmopolitan practices against the communicant inclinations of national governments and their electorates. This article suggests that the ‘cosmopolitan paradox’ – the gap between universal aspiration and hierarchical practice – is not merely one of cosmopolitan ‘consciousness’ lagging behind an immanent cosmopolitan ‘reality’. Rather, the paradox is rooted in the essence of the cosmopolitan thesis itself. The limitations of abstract normative cosmopolitan conceptions of ‘rights’ and ‘responsibilities’, in a world structured by economic and social inequalities, raise major questions over the progressive claims made by cosmopolitan theorists. In fact, rather than challenging existing international structures of power, there is a real danger that the cosmopolitan impulse will legitimize a much more hierarchical set of international relationships.

Whether the cosmopolitan aspiration takes the form of Robbins’s call for a transnational welfare safety net or claims for the protection and promotion of a more extensive range of human rights, all cosmopolitan perspectives reflect the increasing prominence of individual rights claims in the international sphere. Leading cosmopolitan theorists seek to challenge the restrictions of the UN Charter framework, imposed by the major powers in the aftermath of the Second World War, which formally prioritized the ‘state-based’ principles of sovereignty and non-intervention_ They argue that these principles need to be replaced by a new set of cosmopolitan principles, which make the universal individual rights of members of ‘global society’ the primary focus.

 

Q. 41 Which of the following is a part of the empirical dimension as per the passage?

A. Shared fate

B. Universal aspiration

C. Inequalities in practices

D. The cosmopolitan paradox

 

Q. 42 Which of the following can he inferred from the passage?

A. Cosmopolitan theorists seek to legitimize a much more hierarchical set of international relationships

B. Cosmopolitan theorists feel that the principles of sovereignty and non-intervention need to be implemented at a global level

C. the theories and conceptions of cosmopolitanism are responsible for the cosmopolitan paradox

D. Cosmopolitan consciousness does not really lag behind an immanent cosmopolitan reality

 

Q. 43 The author is primarily concerned with

A. exploring the cosmopolitan paradox and solutions and reasons for the Mee

B. exploring the tussle between the advocates of nationalism and cosmopolitanism

C. arguing that the cosmopolitan paradox will continue to exist

D. enthusing that ’empirical reality’ is not an excuse to do away with cosmopolitan aspirations.

 

Question:-  44

For the question word below, a contextual usage is provided. Pick the word from the alternatives given that is closest, in meaning, in the given context.

 

Q. 44 Amortise It was a rude shock to witness the sober, usually docile child in a DIOK, fighting ferociously while the parents amortised his insatiable demands.

A. Servile

B. Delibrate

C. Abort

D. Decant

 

Question 45

Five sentences are given below, labelled A, B, C, D and E. They need to be arranged in a logical order to form a coherent paragraph/passage. From the given options, choose the most appropriate option.

 

Q. 45 (A) The revolution began as an attack on despotism.

(B) Already by 1762 Rousseau was implying in his ‘Social Contract’ that there was no meaningful difference between the authority of a despot and that of a monarch.

(C) As usual, regular usage soon diluted the original rigor of the expression’s meaning.

(D) Montesquieu has defined its spirit as “The rule of one, according to no law”.

(E) Obeying no law, authority was arbitrary and its animating spirit was fear.

A. ABDCE

B. AEDCB

C. ADECB

D. None of these

 

Q. 46 The word given below has been used in sentences in four different ways. Choose the option corresponding to the sentence in which the usage of the word is incorrect or inappropriate.

Make

A. May I make so bold as to suggest that you comply to the rules of the game?

B. Thiruvan decided to make over her studio to the house where he grew up.

C. It’s so strange that Aparna could not make up the meaning of the riddle.

D. The two friends fought vicariously, but their neighbors forced them to make up.

 

Question 47

On the other hand, some writers have expressed_______ that a few publishing houses refuse to publish women writers whose works are, as they are described, “not bold enough”, that is they are not sensational enough to ______ the market.

 

Q. 47 There are two gaps in the sentence/paragraph given . From the pairs of words given, choose the one that fills the gaps most appropriately.

A. concern, stimulate

B. disquiet, titillate

C. anxiety, manoeuvre

D. apprehension, excite

 

Q. 48 Given below are sets of four sentences that form part of a paragraph. Arrange the four sentences so that the given sentences constitute a coherent paragraph.

(A) What came out was very large garland made out of currency notes?

(B) The unsuspecting governor opened the box in full view of the gathering

(C) When the RBI governor came to inaugurate the new printing press, the local unit of the BJP handed him a gift wrapped box

(D) There was a twist – the notes were all as tattered as notes could get

A. DACB

B. CABD

C. CBAD

D. DCAB

 

Q. 49 In the question, there are four sentences or parts of sentences that form a paragraph.

Identify the sentence(s) or part(s) of sentence(s) that is/are incorrect in terms of grammar and usage. Then, choose the most appropriate option.

(A) The finding suggests some cases of the disease could be caused by the immune system running amok and attacking healthy tissues or failing to fight infection that leave people susceptible to the condition.

(B) The study is the first to use evidence from the human genome to confirm the long-held suspicion that the immune system plays role in the disease.

(C) “People have speculated about a link between the immune system and Parkinson’s disease for sometime and this study suggests that a link is real”, said Cyrus Zabetian.

(D) Parkinson’s disease is caused by steady dying-off of brain cells that produce a chemical called dopamine

A. A and D

B. B and C

C. Only D

D. Only C

 

Q. 50 In the question, there are four sentences that form a paragraph. Identify the sentence(s) or part(s) of sentence(s) that is/are correct in terms of grammar and usage. Then, choose the most appropriate option.

(A) But the ‘economic miracle’ came to a juddering halt at the beginning of the 1990s when the property bubble busted.

(B) What followed was a decade in the doldrums and the country has never fully recovered.

(C) Today, it faces deflation, an ageing and shrinking population and only minimal growth.

(D) Economists also cited the figures as evidence that the global recovery was still facing strong headwind.

A. C and D

B. B and C

C. B, C and D

D. A and C

 

Questions: 51 – 54

Read the passage given below and answer questions that follow based on the information given in the passage.

First AOL and Time Warner announced their intention to combine. Then came Time Warner/EMI and Tribune/ Times Mirror. Even more significant, however, has been the speculation that these mergers have caused: If these transactions are consummated, a large number of additional media mergers are expected. There is even the possibility of a nightmare scenario-a wave of media mergers so large that within a decade most of our information will be supplied by perhaps six of these huge conglomerates and a fringe of much smaller firms. It’s time to ask two critical questions. Is this kind of media oligopoly what we, as a loom. want? And if not, can the anti-trust laws effectively prevent the threatened merger wave? The answer to the first IllaelltiCKI is clear. We do not want a media oligopoly. The answer to the second question, however, is far less certain_ We should distrust a media oligopoly because it would give undue control to a small number of individuals. This need not manifest itself in a price rise for the daily newspaper or AOL’s monthly fee. Rather, it could consist of a change in editorial viewpoints, a shift in the relative prominence of links to certain websites or a decision not to cover certain topics, became they ate not ‘newsworthy’. These problems could exist without any improper intent on the part of the media barons.Even if they try to be fair and objective, they will necessarily bring their own worldview to the job, And in time some of these conglomerates may be controlled by people who are not fair or objective. At first, it might appear that the anti-trust laws can be of little help in grappling ’emit rite roans presented by large media mergers. The anti-merger laws are commonly understood as protecting price competition and a relatively small number of firms-to greatly oversimplify, let’s say at most half a dozen-are normally thought to be enough to keep a market price-competitive. In industry after industry firms merge until there is only a handful left ad the antitrust enforcers are normally unable to do anything to prevent this. (In former years, mergers were governed by an Incipiency’ standard that prevented mergers and merger waves well before they would have led to very large at likely anti – competitive problems). Even if a handful of firms are enough to insure effective competition in most industries, would six conglomerate media firms be sufficient for the diversity of viewpoints necessary to democracy? Would have be reassured if they could somehow guarantee that they would sell their magazines and

Internet advertisements at competitive prices? I am hopeful that the anti-trust laws, if correctly and vigorously interpreted, are adaptable aweigh to meet this challenge. This is because anti-trust is not exclusively about price. It is essentially about choice hem giving consumers a competitive range of options in the marketplace so that they can make their own, effective seined= boas the market’s offerings. Consumers should be able to make their choices along any dimension important to them including price, variety and editorial viewpoint. Communications media compete in part by offering independent editorial viewpoints and an independent gatekeeper function. Six media firms cannot effectively respond to the demand for choice ar tibialsay 0-Ir.:petition by extending their product lines, because new media products will inevitably bear, to some degree the perspective of their corporate parent. For these reasons, competition in terms of editorial viewpoint or gate-keeping can be guaranteed only by insuring that a media market contains a significantly larger number of firms than is required for price competition in other, more conventional markets. It is unclear, however, whether this interpretation of the anti-trust laws will be applied by the enforcement agencies and the courts. What is needed, therefore, is a much more careful look at the challenges that will be raised by future media mergers. This could best be accomplished if Congress created a Temporary Committee to study Media Mergers and Media Convergence. This committee could include members of Congress; the heads of the federal Trade Commission, the Federal Communications Commission and the Justice Department’s anti-trust divisions, CEOs of media companies; and representatives of consumer groups. The committee would identify problems that are used by large media mergers and by media convergence. If the committee concludes that existing anti-trust laws are inadequate, it should recommend to Congress that new anti-merger legislation he enacted. This may be the only way to prevent the nightmare scenario of a media oligopoly.

 

Q. 51 A wave of media mergers could

A. be a threat to democracy

B. result in limiting editorial viewpoints

C. result in misuse of certain laws

D. Both (a) and (b)

 

Q. 52 According to the passage, what could be the most significant outcome of media oligopoly?

A. An increase in the cost of newspapers

B. The fact that in the long run, there will be a shift of power to people who might not be balanced and fair in the way they deal with the media

C. Certain websites may get more prominence than others

D. There will be no competition among the newspapers

 

Q. 53 Which of the following statements, according to the author, are true?

(A) Half a dozen firms are enough to keep the market price-competitive

(B) Half a dozen companies are not enough to provide a democratic media

(C) Enforcement agencies may not interpret the anti-trust laws correctly

(D) Half a dozen companies will be inadequate to meet the consumer demand for product diversity

A. A, B

B. A, B,C

C. A, B,C,D

D. B,C,D

 

Q. 54 To get a clear picture of the challenges posed by media mergers, the author recommends

A. creation of strict laws

B. strengthening the enforcement agencies

C. creation of a study committee by the Congress

D. none of the above

 

Questions: 55 – 58

Read the passage given below and answer questions that follow based on the information given in the passage.

One major obstacle in the struggle to lower carbon dioxide emissions, which are believed to play a role in climate change, is the destruction of tropical rainforests. Trees naturally store more carbon dioxide as they age, and the trees of the tropical rain forests in the Amazon, for example, store an average of 500 tonnes of carbon dioxide per hectare (10,000 square miles). When such trees are harvested, they release their carbon dioxide into the atmosphere. This release of carbon dioxide through the destruction of tropical forests, which experts estimate accounts for 20% of global carbon dioxide emissions annually, traps heat in the earth’s atmosphere, which leads to global warming. The Kyoto treaty set forth a possible measure to curtail the rate of deforestation. In the treaty, companies that exceed their carbon dioxide emission limits are permitted to buy the right to pollute by funding reforestation projects in tropical rainforests. Since forests absorb carbon dioxide through photosynthesis, planting such forests helps reduce the level of atmospheric carbon dioxide, thus balancing out the companies’ surplus of carbon dioxide emissions. However, attempts at reforestation have so far been unable to keep up with the alarming rate of deforestation, and it has become increasingly clear that further steps must be taken to curtail deforestation and its possible deleterious effects on the global environment. One possible solution is to offer incentives for governments to protect their forests. While this solution could lead to a drastic reduction in the levels of carbon dioxide, such incentives would need to be tied to some form of verification, which is extremely difficult, since most of the world’s tropical forests are in remote areas, like Brazil’s Amazon basin or the island of New Guinea, which makes on-site verification logistically difficult. Furthermore, heavy cloud cover and frequent heavy rain make conventional satellite monitoring difficult.

Recently, scientists at the Japan Aerospace Exploration Agency have suggested that the rates of deforestation could be monitored using new technology to analyse radar waves emitted from a surveillance satellite. By analysing multiple radar microwaves sent by a satellite, scientists are able to prepare a detailed, high resolution map of remote tropical forests. Unlike photographic satellite images, radar images can be measured at night and during days of heavy cloud cover and bad weather. Nevertheless, critics of government incentives argue that radar monitoring has been employed in the past with little success, citing the Global Rainforest Mapping Project which was instituted in the mid 1990s amid concern over rapid deforestation in the Amazon. However, the limited data of the Mapping Project was due only to the small amount of data that could be sent from the satellite. Modern satellites can send and receive 10 times more data than their predecessors of the mid 1990s, obviating past problems with radar monitoring. Furthermore, ream recent technology advances in satellite radar that allow for more accurate measurements to be made, even in remote meek make such technology a promising step in monitoring and controlling global climate change.

 

Q. 55 Which one of the following most accurately expresses the main point of the passage?

A. Although scientists continue to search for a solution, there is. as vet. no good solution for the problem of rain forest deforestation.

B. One major obstacle to lessening the contribution of atmospheric carbon dioxide caused by deforestation may be removed through satellite radar monitoring.

C. Recent increases in the rate of deforestation of tropical rainforests have caused serious concern and spurred efforts to curb such deforestation.

D. Although an excellent first step, the solutions set forth by the Kyoto treaty will not significantly curb the rate of deforestation unless companies begin to lessen their carbon dioxide emissions.

 

Q. 56 It can be inferred from the passage that photographic satellite images

A. are impervious to bad weather

B. cannot be used efficiently at night

C. are less expensive than radar monitoring

D. can send only a small amount of data from a satellite to a base

 

Q. 57 Which one of the following most accurately describes the author’s attitude toward radar monitoring as expressed in the passage?

A. Wary skepticism

B. Cautious ambivalence

C. Grudging respect

D. Reasoned optimism

 

Q. 58 The information presented in the passage implies which one of the following about the Mapping Project?

A. The project was unsuccessful because it used only satellite radar monitoring

B. If the satellite had been able to send more data, the project may have been successful

C. It was established by the Kyoto treaty in response to widespread concern over deforestation

D. The project used only conventional satellite monitoring and on-site verification visits

 

Questions: 59 – 60

Given are sets of four sentences that form part of a paragraph. Arrange the four sentences so that the given sentences constitute a coherent paragraph.

 

Q. 59 A) 1971 war changed the political geography of the subcontinent

(B) Despite the significance of the event, there has been no serious book about the conflict

(C) Surrender at Dhaka aims to fill this gap

(D) It also profoundly altered the geo-strategic situation in South-East Asia

A. ACBD

B. CADB

C. BADC

D. ADBC

 

Q. 60 (A) Thus begins the search for relief: painkillers, ice, yoga, herbs, even surgery

(B) Most computer users develop disorders because they ignore warnings like tingling fingers, a numb hand or a sore shoulder

(C) They keep pointing and dragging until tendons chafe and scar tissue forms, along with bad habits that are almost impossible to change.

(D) But cures are elusive, because repetitive injuries present a bag of ills that often defy easy diagnosis.

A. BDAC

B. BADC

C. BCAD

D. ABCD

 

 

Answer Sheet
Question 1 2 3 4 5 6 7 8 9 10
Answer B D D D B B A C A B
Question 11 12 13 14 15 16 17 18 19 20
Answer D D C A D D A D B B
Question 21 22 23 24 25 26 27 28 29 30
Answer B C B A D B D C D B
Question 31 32 33 34 35 36 37 38 39 40
Answer B C B C A A D A B D
Question 41 42 43 44 45 46 47 48 49 50
Answer C C A C C C B C A B
Question 51 52 53 54 55 56 57 58 59 60
Answer D C C C B B D B D C
×

Hello!

Click one of our representatives below to chat on WhatsApp or send us an email to info@vidhyarthidarpan.com

×